Sunteți pe pagina 1din 140

2019 - Test 28-

Exam Title :
History & C...
Email : narenips6@gmail.com
Contact :

Note: If the exam is multi-lingual i.e. English and Hindi. Hindi solutions will be
after the completion of English solutions.
QUESTION 1. MTU4Njk4K0d1bnR1cnUgTmFyZW4gS3VtYXIrbmFyZW5pcHM2QGdtYWlsLmNvbStRVU
VTVElPTiAw
Consider the following statements

1. The prehistoric paintings found at Lakhudiyar consists of man and animal paintings only

2. The human figures found at Lakhudiyar are represented in stick- like forms

Choose the correct code

a) 1 only
b) 2 only
c) Both 1 and 2
d) None of the above
Correct Answer: B
Your Answer: Unanswered
Explanation

Solution (b)

Remnants of rock paintings have been found on the walls of the caves situated in several
districts of Madhya Pradesh, Uttar Pradesh, Andhra Pradesh, Karnataka and Bihar. Some
paintings have been reported from the Kumaon hills in Uttarakhand also. The rock shelters on
banks of the River Suyal at Lakhudiyar, about twenty kilometres on the Almora– Barechina
road, bear these prehistoric paintings. Lakhudiyar literally means one lakh caves. The paintings
here can be divided into three categories: man, animal and geometric patterns in white,
black and red ochre . Humans are represented in stick-like forms . A long-snouted animal,
a fox and a multiple legged lizard are the main animal motifs. Wavy lines, rectangle-filled
geometric designs, and groups of dots can also be seen here. One of the interesting scenes
depicted here is of hand-linked dancing human figures. There is some superimposition of
paintings. The earliest are in black; over these are red ochre paintings and the last group
comprises white paintings.

QUESTION 2. MTU4Njk4K0d1bnR1cnUgTmFyZW4gS3VtYXIrbmFyZW5pcHM2QGdtYWlsLmNvbStRVU
VTVElPTiAx
Consider the following statements

1. In upper Palaeolithic period the paintings were in the form of linear representation where
green paintings were of dancers and the red ones of hunters.

2. Hunting scenes predominate in the Mesolithic paintings

3. In Mesolithic paintings humans were depicted only in a stylistic manner

Choose the correct code

a) 1 and 2 only
b) 2 and 3 only

IASbaba
Score:
Web: http://ilp.iasbaba.com/
47.00 /
Email: ilp@iasbaba.com
Page 1 200
2019 - Test 28-
Exam Title :
History & C...
Email : narenips6@gmail.com
Contact :

c) 1 and 3 only
d) All of the above
Correct Answer: D
Your Answer: Unanswered
Explanation

Solution (d)

The paintings of the Upper Palaeolithic phase are linear representations, in green and dark red,
of huge animal figures, such as bisons, elephants, tigers, rhinos and boars besides stick-like
human figures. A few are wash paintings but mostly they are filled with geometric patterns. The
green paintings are of dancers and the red ones of hunters.

The largest number of paintings belongs to Period II that covers the Mesolithic paintings.
During this period the themes are multiple but the paintings are smaller in size. Hunting scenes
predominate. The hunting scenes depict people hunting in groups, armed with barbed spears,
pointed sticks, arrows and bows. Though animals were painted in a naturalistic style, humans
were depicted only in a stylistic manner. Women are painted both in the nude and clothed. The
young and the old equally find place in these paintings. Children are painted running, jumping
and playing. Community dances provide a common theme.

QUESTION 3. MTU4Njk4K0d1bnR1cnUgTmFyZW4gS3VtYXIrbmFyZW5pcHM2QGdtYWlsLmNvbStRVU
VTVElPTiAy
Consider the following statements about Indus valley civilization

1. The terracotta images of humans are in crude form compared to the stone and bronze statues

2. During Indus valley civilization, the bronze casting technique was practiced only in Harappa

3. The bronze figures of only humans are found during this time

Choose the correct code

a) 1 and 2 only
b) 2 and 3 only
c) 1 and 3 only
d) None of the above
Correct Answer: D
Your Answer: Unanswered
Explanation

Solution (d)

Statues whether in stone, bronze or terracotta found in Harappan sites are not abundant, but
refined. The stone statuaries found at Harappa and Mohenjodaro are excellent examples of
handling three-dimensional volumes. In stone are two male figures—one is a torso in red
sandstone and the other is a bust of a bearded man in soapstone—which are extensively
discussed.

The art of bronze-casting was practised on a wide scale by the Harappans. In bronze we find
human as well as animal figures, the best example of the former being the statue of a girl
popularly titled ‘Dancing Girl’. Amongst animal figures in bronze the buffalo with its uplifted

IASbaba
Score:
Web: http://ilp.iasbaba.com/
47.00 /
Email: ilp@iasbaba.com
Page 2 200
2019 - Test 28-
Exam Title :
History & C...
Email : narenips6@gmail.com
Contact :
head, back and sweeping horns and the goat are of artistic merit. Bronze casting was popular at
all the major centres of the Indus Valley Civilisation. The copper dog and bird of Lothal and the
bronze figure of a bull from Kalibangan are in no way inferior to the human figures of copper
and bronze from Harappa and Mohenjo-Daro.

The Indus Valley people made terracotta images also but compared to the stone and bronze
statues the terracotta representations of human form are crude in the Indus Valley. They are
more realistic in Gujarat sites and Kalibangan. The most important among the Indus figures are
those representing the mother goddess.

QUESTION 4. MTU4Njk4K0d1bnR1cnUgTmFyZW4gS3VtYXIrbmFyZW5pcHM2QGdtYWlsLmNvbStRVU
VTVElPTiAz
Which of the following are correctly matched?

Place Speciality

1. Lothal A) dead bodies were buried with ornaments

2. Dholavira B) Stone structural remains

3. Farmana in Haryana C) bead factory

Choose the correct code

a) 1 and 2 only
b) 2 only
c) 1 and 3 only
d) None of the above
Correct Answer: B
Your Answer: B
Explanation

Solution (b)

The Harappan men and women decorated themselves with a large variety of ornaments
produced from every conceivable material ranging from precious metals and gemstones to bone
and baked clay. All ornaments are well crafted. It may be noted that a cemetery has been found
at Farmana in Haryana where dead bodies were buried with ornaments.

The bead industry seems to have been well developed as evident from the factories discovered
at Chanhudaro and Lothal. Beads were made of carnelian, amethyst, jasper, crystal, quartz,
steatite, turquoise, lapis lazuli, etc. Metals like copper, bronze and gold, and shell, faience and
terracotta or burnt clay were also used for manufacturing beads.

From archaeological finds it appears that the people of the Indus Valley were conscious of
fashion. Different hairstyles were in vogue and wearing of a beard was popular among all.
Cinnabar was used as a cosmetic and facepaint, lipstick and collyrium (eyeliner) were also
known to them. Many stone structural remains are also found at Dholavira which show how the
Indus Valley people used stone in construction.

IASbaba
Score:
Web: http://ilp.iasbaba.com/
47.00 /
Email: ilp@iasbaba.com
Page 3 200
2019 - Test 28-
Exam Title :
History & C...
Email : narenips6@gmail.com
Contact :
QUESTION 5. MTU4Njk4K0d1bnR1cnUgTmFyZW4gS3VtYXIrbmFyZW5pcHM2QGdtYWlsLmNvbStRVU
VTVElPTiA0
Consider the following statements about the famous dancing girl statue of Indus valley
civilization.

1. It is found at Mohenjo-daro

2. It is made from broze by using lost wax technique

3. The figure is full of expression and bodily vigour and conveys a lot of information

Choose the correct code

a) 1 and 2 only
b) 2 only
c) 1 and 3 only
d) None of the above
Correct Answer: C
Your Answer: Unanswered
Explanation

Solution (c)

One of the best known artefacts from the Indus Valley is this approximately four-inch-high
copper figure of a dancing girl. Found in Mohenjo-Daro, this exquisite casting depicts a girl
whose long hair is tied in a bun. Bangles cover her left arm, a bracelet and an amulet or bangle
adorns her right arm, and a cowry shell necklace is seen around her neck. Her right hand is on
her hip and her left hand is clasped in a traditional Indian dance gesture. She has large eyes
and flat nose. This figure is full of expression and bodily vigour and conveys a lot of information.

QUESTION 6. MTU4Njk4K0d1bnR1cnUgTmFyZW4gS3VtYXIrbmFyZW5pcHM2QGdtYWlsLmNvbStRVU
VTVElPTiA1
consider the following statements

1. Yaksha worship gained prominence after the advent of Buddhism

2. Lomus rishi cave is a rock cut cave donated by Ashoka for the Ajivika sect

3. The Mauryan pillars were constructed in pieces by a mason displaying the carver’s skills

Choose the correct code

a) 1 and 2 only
b) 2 only
c) 1 and 3 only
d) None of the above
Correct Answer: B
Your Answer: Unanswered
Explanation

Solution (b)

IASbaba
Score:
Web: http://ilp.iasbaba.com/
47.00 /
Email: ilp@iasbaba.com
Page 4 200
2019 - Test 28-
Exam Title :
History & C...
Email : narenips6@gmail.com
Contact :

Yaksha worship was very popular before and after the advent of Buddhism and it was
assimilated in Buddhism and Jainism. Construction of stupas and viharas as part of monastic
establishments became part of the Buddhist tradition.

However, in this period, apart from stupas and viharas, stone pillars, rock-cut caves and
monumental figure sculptures were carved at several places. The tradition of constructing
pillars is very old and it may be observed that erection of pillars was prevalent in the
Achamenian Empire as well. But the Mauryan pillars are different from the Achamenian pillars.
The Mauryan pillars are rockcut pillars thus displaying the carver’s skills, whereas the
Achamenian pillars are constructed in pieces by a mason.

The rock-cut cave carved at Barabar hills near Gaya in Bihar is known as the Lomus Rishi cave.
The facade of the cave is decorated with the semicircular chaitya arch as the entrance. The
elephant frieze carved in high relief on the chaitya arch shows considerable movement. The
interior hall of this cave is rectangular with a circular chamber at the back. The entrance is
located on the side wall of the hall. The cave was donated by Ashoka for the Ajivika sect

QUESTION 7. MTU4Njk4K0d1bnR1cnUgTmFyZW4gS3VtYXIrbmFyZW5pcHM2QGdtYWlsLmNvbStRVU
VTVElPTiA2
Which of the following stupas built on the relics of the Buddha are situated outside the Gangetic
valley?

1. Rajagriha

2. Vaishali

3. Avanthi

4. Gandhara

Choose the correct code

a) 1 and 2 only
b) 2 and 4only
c) 3 and 4 only
d) 1 and 3 only
Correct Answer: C
Your Answer: Unanswered
Explanation

Solution (c)

Due to the popularity of Buddhism and Jainism, stupas and viharas were constructed on a large
scale. However, there are also examples of a few Brahmanical gods in the sculptural
representations. It is important to note that the stupas were constructed over the relics of the
Buddha at Rajagraha, Vaishali, Vethadipa and Pava in Bihar, Kapilavastu, Allakappa and
Ramagrama in Nepal, Kushinagar and Pippalvina in Uttar Pradesh. The textual tradition also
mentions construction of various other stupas on the relics of the Buddha at several places
including Avanti and Gandhara which are outside the Gangetic valley. Stupa, vihara and chaitya
are part of Buddhist and Jaina monastic complexes but the largest number belongs to the
Buddhist religion.

IASbaba
Score:
Web: http://ilp.iasbaba.com/
47.00 /
Email: ilp@iasbaba.com
Page 5 200
2019 - Test 28-
Exam Title :
History & C...
Email : narenips6@gmail.com
Contact :

QUESTION 8. MTU4Njk4K0d1bnR1cnUgTmFyZW4gS3VtYXIrbmFyZW5pcHM2QGdtYWlsLmNvbStRVU
VTVElPTiA3
Consider the following statements about lion capital at Sarnath

1. It was built in commemoration of the historical event of the first sermon or the
Dhammachakrapravartana by the Buddha

2. The abacus has the depiction of four animals namely a bull, a horse, an elephant and a lion

3. The capital without the crowning wheel has been adopted as the National Emblem of
Independent India

Choose the correct code

a) 1 and 2 only
b) 2 and 4only
c) 3 and 4 only
d) 1 and 3 only
Correct Answer: A
Your Answer: Unanswered
Explanation

Solution (a)

The Lion Capital discovered more than a hundred years ago at Sarnath, near Varanasi, is
generally referred to as Sarnath Lion Capital. This is one of the finest examples of sculpture
from the Mauryan period. Built in commemoration of the historical event of the first sermon or
the Dhammachakrapravartana by the Buddha at Sarnath, the capital was built by Ashoka.

The capital originally consisted of five component parts: (i) the shaft (which is broken in many
parts now), (ii) a lotus bell base, (iii) a drum on the bell base with four animals proceeding
clockwise, (iv) the figures of four majestic addorsed lions, and (v) the crowning element,
Dharamchakra, a large wheel, was also a part of this pillar. However, this wheel is lying in a
broken condition and is displayed in the site museum at Sarnath. The capital without the
crowning wheel and the lotus base has been adopted as the National Emblem of Independent
India. The abacus has the depiction of a chakra (wheel) having twenty-four spokes in all the
four directions and a bull, a horse, an elephant and a lion between every chakra is finely carved.

QUESTION 9. MTU4Njk4K0d1bnR1cnUgTmFyZW4gS3VtYXIrbmFyZW5pcHM2QGdtYWlsLmNvbStRVU
VTVElPTiA4
Consider the following statements about Madhubani paintings

1. It is a popular form of folk style painting practiced in Bihar

2. Objects like twigs, matchsticks and even fingers are used to create the paintings instead of
contemporary brushes

3. Tantric, Bharni and Godna were different styles associated with Madhubani painting

Choose the correct code

IASbaba
Score:
Web: http://ilp.iasbaba.com/
47.00 /
Email: ilp@iasbaba.com
Page 6 200
2019 - Test 28-
Exam Title :
History & C...
Email : narenips6@gmail.com
Contact :

a) 1 and 2 only
b) 2 and 4only
c) 3 and 4 only
d) All of the above
Correct Answer: D
Your Answer: D
Explanation

Solution (d)

Madhubani painting is one of the many famous Indian art forms. As it is practiced in the Mithila
region of Bihar and Nepal, it is called Mithila or Madhubani art. Often characterized by
complex geometrical patterns, these paintings are known for representing ritual content for
particular occasions, including festivals, religious rituals, etc. The colours used in Madhubani
paintings are usually derived from plants and other natural sources. These colours are often
bright and pigments like lampblack and ochre are used to create black and brown respectively.
Instead of contemporary brushes, objects like twigs, matchsticks and even fingers are used to
create the paintings. Madhubani paintings were initially practiced by different sects of people
and hence the paintings were categorized into five different styles, such as Tantrik, Kohbar,
Bharni, Godna, Katchni. But today, these five different styles have been merged by
contemporary artists. The themes used in these paintings often revolve around Hindu deities
like Krishna, Rama, Lakshmi, Shiva, Durga and Saraswati. Also, heavenly bodies like the Sun
and the Moon often form the centrepiece of Madhubani paintings.

QUESTION 10. MTU4Njk4K0d1bnR1cnUgTmFyZW4gS3VtYXIrbmFyZW5pcHM2QGdtYWlsLmNvbStRVU


VTVElPTiA5
Consider the following pairs

1. Arunachal Pradesh – Losar festival

2. Maharashtra – Ganesh Chathurthi

3. Goa – carnival

4. Haryana - Bastar Dusheera

Choose the correct code

a) 1, 2 and 3 only
b) 2, 3 and 4only
c) 3 and 4 only
d) All of the above
Correct Answer: A
Your Answer: A
Explanation

Solution (a)

Ganesh Chaturthi, the most important festival of Maharashtra falls in months of August — 
September. Ganesh images are kept in houses as a divine guest for five to ten days by people.

IASbaba
Score:
Web: http://ilp.iasbaba.com/
47.00 /
Email: ilp@iasbaba.com
Page 7 200
2019 - Test 28-
Exam Title :
History & C...
Email : narenips6@gmail.com
Contact :
The image is then taken out ceremoniously and immersed in the river, sea or well. It is called
Visarjan. Modak is special cuisine for Ganesh Chaturthi celebrations.

The exciting and fun-filled three-day non-stop extravaganza of fun, frolic, song, music and dance
that is uniquely Goan! Celebrated since the 18th century, it was meant to be a feasting-drinking-
merrymaking orgy just before the 40 days of Lent; a time of abstinence and spirituality.

In the Carnival huge parades through the cities are organized with bands, floats and dances and
balls in the evenings. The final day concludes with the famous Red-and-Black dance held by the
Clube Nacional in Panjim. The carnival is presided over by King Momo, who on the opening day
orders his subjects to party.

Bastar Dussehra, the longest Dussehra celebrations in the world is celebrated in Bastar region
of Chattisgarh and spans over 75 days starting around August and ending at October. Diverse
tribes participate with traditional customs.

Bastar Dussehra begins with several customs like ‘Pata jatra’, or worship of wood followed by
‘Deri Gadhai’- Posting of the pillars, Kachan Gaadi- Throne for goddess Kachan Devi, Kalash
Sthapana- Installation of urns, Jogi Bithai- Jogi’s penance, Rath Parikrama- The chariot circuit,
Nisha Jatra- The nocturnal festival, Jogi Uthai- Raising of the jogi, Maoli Parghav- Reception of
Devi Maoli, Bheetar Raini- The inner circuit, Baahar Raini- The outer circuit, Kachan Jatra-
Thanksgiving ceremony, Muria Durbar- The tribal chieftains’ conference and finally on the last
day, Ohadi- A farewell to deities.

The Tibetan New year is also known as Losar is practiced in Arunachal Pradesh and is mainly
celebrated for 3 days in late January or February. Marked with ancient ceremonies that
represent the struggle between good and evil Lossar has people chanting and passing fire
torches through the crowds.

Events like the dance of the deer and the amusing battles between the King and his various
ministers provide the cheerful side of festivities. Losar Festival is characterized especially by
dancing, music, and a general spirit of merry making.

QUESTION 11. MTU4Njk4K0d1bnR1cnUgTmFyZW4gS3VtYXIrbmFyZW5pcHM2QGdtYWlsLmNvbStRVUVTVE


PTiAxMA==
Losoong festival is held in which of the following states?

a) Sikkim
b) Manipur
c) Himachal Pradesh
d) Mizoram
Correct Answer: A
Your Answer: D
Explanation

Solution (a)

Losoong is the Sikkimese New Year. It is also called ‘Sonam Losar’ (or the Farmer’s New Year)
for the farmer’s rejoice and celebrate their harvest. Losoong marks the end of the harvest
season and also the end of the tenth month of lunar calendar. It is a traditional celebration to
thank for the good harvest and to offer prayers for better in the next season. Although Losoong
is celebrated privately among family members and friends, there is an air of festivity and the

IASbaba
Score:
Web: http://ilp.iasbaba.com/
47.00 /
Email: ilp@iasbaba.com
Page 8 200
2019 - Test 28-
Exam Title :
History & C...
Email : narenips6@gmail.com
Contact :
youth all over Sikkim have archery competitions. The festival is marked by Chaam (masked)
dancing at Tsu-La-Khang Palace temple, in Phodong and in Rumtek. The dances symbolize the
exorcizing of the evil spirits of the year and the welcoming of the good spirit of the New Year. It
is best to see this festival in rural areas of Sikkim, where it is celebrated more pompously with
traditional archery competitions and other events.

QUESTION 12. MTU4Njk4K0d1bnR1cnUgTmFyZW4gS3VtYXIrbmFyZW5pcHM2QGdtYWlsLmNvbStRVUVTVE


PTiAxMQ==
The event in the picture frame of this Jataka panel depicts the King standing with his army and
about to shoot an arrow at the deer, and the man who was rescued by the deer is also shown
along with the king pointing a finger at the deer.

Which of the following Jataka panels is being referred to here?

a) Jataka panel,Ajanta
b) Jataka pane, Ellora
c) Jataka panel, Bharuht
d) Jataka panel, Sanchi
Correct Answer: C
Your Answer: C
Explanation

Solution (c)

The depiction of Jataka story is very simple—narrated by clubbing the events according to the
geographical location of the story like the depiction of Ruru Jataka where the Boddhisattva deer
is rescuing a man on his back. The other event in the same picture frame depicts the King
standing with his army and about to shoot an arrow at the deer, and the man who was rescued
by the deer is also shown along with the king pointing a finger at the deer. According to the
story, the man promised the deer after his rescue that he would not disclose his identity to
anybody. But when the king makes a proclamation of reward for disclosing the identity of the
deer, he turns hostile and takes the king to the same jungle where he had seen the deer. Such
Jataka stories became part of stupa decoration. Interestingly, with the rise in the construction of
stupas in various parts of the country, regional stylistic variations also began to emerge. One
main characteristic in all the male images of first–second centuries BCE is the knotted
headgear. In many sculptures it is very consistent. Some of the sculptures found at Bharhut are
displayed in Indian Museum, Kolkata.

QUESTION 13. MTU4Njk4K0d1bnR1cnUgTmFyZW4gS3VtYXIrbmFyZW5pcHM2QGdtYWlsLmNvbStRVUVTVE


PTiAxMg==
Which of the following are correctly matched?

Place Cave

1. Ajanta A) Apsidal vault-roof chaitya halls

2. Thana-Nadsur B) Apsidal vault-roof pillar less halls

3. Kondavite C) flat-roofed quadrangular hall with a circular chamber at the back

IASbaba
Score:
Web: http://ilp.iasbaba.com/
47.00 /
Email: ilp@iasbaba.com
Page 9 200
2019 - Test 28-
Exam Title :
History & C...
Email : narenips6@gmail.com
Contact :

Choose the correct code

a) 1 and 2 only
b) 2 and 3 only
c) 1 and 3 only
d) All of the above
Correct Answer: D
Your Answer: Unanswered
Explanation

Solution (d)

In western India, many Buddhist caves dating back to the second century BCE onwards have
been excavated. Mainly three architectural types were executed—(i) apsidal vault-roof chaitya
halls (found at Ajanta, Pitalkhora, Bhaja); (ii) apsidal vault-roof pillarless hall (found at Thana-
Nadsur in Maharashtra); and (iii) flat-roofed quadrangular hall with a circular chamber at the
back (found at Kondivite in Maharashtra). The front of the chaitya hall is dominated by the
motif of a semi-circular chaitya arch with an open front which has a wooden façade and, in
some cases, there is no dominating chaitya arch window such as found at Kondivite. In all the
chaitya caves a stupa at the back is common.

QUESTION 14. MTU4Njk4K0d1bnR1cnUgTmFyZW4gS3VtYXIrbmFyZW5pcHM2QGdtYWlsLmNvbStRVUVTVE


PTiAxMw==
Consider the following statements

1. Ajanta is the only surviving example of painting of the first century BCE and the fifth century
CE

2. Karla has the largest cave excavations

3. The biggest rock-cut chaitya hall was excavated in Junnar

Choose the correct code

a) 1 only
b) 2 and 3 only
c) 1 and 3 only
d) All of the above
Correct Answer: A
Your Answer: D
Explanation

Solution (a)

In Karla, the biggest rock-cut chaitya hall was excavated. The cave consists of an open
courtyard with two pillars, a stone screen wall to protect from rain, a veranda, a stone-screen
wall as facade, an apsidal vault-roof chaitya hall with pillars, and a stupa at the back. Karla
chaitya hall is decorated with human and animal figures.

The stupas in the fourth and fifth centuries CE have Buddha images attached. Junnar has the
largest cave excavations— more than two hundred caves around the hills of the town— whereas

IASbaba
Score:
Web: http://ilp.iasbaba.com/
47.00 /
Email: ilp@iasbaba.com
Page 10 200
2019 - Test 28-
Exam Title :
History & C...
Email : narenips6@gmail.com
Contact :
Kanheri in Mumbai has a hundred and eight excavated caves. The most important sites are
Ajanta, Pitalkhora, Ellora, Nashik, Bhaja, Junnar, Karla, Kanheri. Ajanta, Ellora, and Kanheri
continue to flourish.

The most famous cave site is Ajanta. Ajanta has twenty-nine caves. It has four chaitya caves
datable to the earlier phase, i.e., the second and the first century BCE (Cave Nos. 10 and 9) and
the later phase, i.e., the fifth century CE (Cave Nos. 19 and 26). It has large chaityaviharas and
is decorated with sculptures and paintings. Ajanta is the only surviving example of painting of
the first century BCE and the fifth century CE.

QUESTION 15. MTU4Njk4K0d1bnR1cnUgTmFyZW4gS3VtYXIrbmFyZW5pcHM2QGdtYWlsLmNvbStRVUVTVE


PTiAxNA==
Consider the following pairs of patrons with their caves at Ajanta

1. Varahadeva - patron of Cave No.26

2. Upendragupta - patron of Cave Nos.17–20

3. Buddhabhadra - patron of Cave No.16

4. Mathuradasa - patron of Cave No.4

Choose the correct code

a) 2 and 4 only
b) 1 and 3 only
c) 1,2 and 4 only
d) 1,2 and 3 only
Correct Answer: A
Your Answer: C
Explanation

Solution (a)

Shrine images at Ajanta are grand in size. Some of the vihara caves are unfinished such as Cave
Nos. 5, 14, 23 24, 28 and 29. Among the important patrons at Ajanta were Varahadeva (patron
of Cave No. 16), the prime minister of the Vakataka king, Harishena; Upendragupta (patron of
Cave Nos. 17–20) the local king of the region and feudatory of the Vakataka king, Harishena;
Buddhabhadra (patron of Cave No. 26); and Mathuradasa (patron of Cave No. 4). Many
paintings have survived in Cave Nos. 1, 2, 16 and 17.

QUESTION 16. MTU4Njk4K0d1bnR1cnUgTmFyZW4gS3VtYXIrbmFyZW5pcHM2QGdtYWlsLmNvbStRVUVTVE


PTiAxNQ==
In which of the following temples the first instance of Shikhar and Panchayatan architecture is
found?

a) Dashavatar temple at Deogarh


b) Maniyar Math temple at Rajgir
c) Sanchi temple

IASbaba
Score:
Web: http://ilp.iasbaba.com/
47.00 /
Email: ilp@iasbaba.com
Page 11 200
2019 - Test 28-
Exam Title :
History & C...
Email : narenips6@gmail.com
Contact :

d) Khandariya Mahadev Temple of Khajuraho


Correct Answer: A
Your Answer: A
Explanation

Solution (a)

Dasavatara Temple: It is situated in Deogarh in Uttar Pradesh. The temple is dedicated to


Vishnu. This is the first temple of India with a Shikhara and having Panchayatan style of
architecture.

A Hindu temple is a Panchayatana one when the main shrine is surrounded by four subsidiary
shrines.

QUESTION 17. MTU4Njk4K0d1bnR1cnUgTmFyZW4gS3VtYXIrbmFyZW5pcHM2QGdtYWlsLmNvbStRVUVTVE


lPTiAxNg==
Consider the following statements about Nagara style of temples

1. The complete structure is built on an elevated platform with the temple having a square base
with projections going outside

2. Shikhar, a tower built over the Garbhagriha is the main characteristic feature of these
temples

3. The remains of their architecture are mostly found at Mamallapuram and Kanchipuram

Choose the correct code

a) 1 and 2 only
b) 2 and 3 only
c) 1 and 3 only
d) All of the above
Correct Answer: A
Your Answer: Unanswered
Explanation

Solution (a)

General Characteristic of Nagara Style Temples:

Location: This time of architecture is found in Northern India from Himalayas up to Vindhyas.

Plan: It has a cruciform ground plan with projections coming out of it. The complete structure is
built on an elevated platform. The temple has a square base with projections going outside
called rathas.

Parts of a temple :

1) Garbhagriha: It is the Sanctum Sanctorum of the temple where the idol of chief deity is kept.

2) Mandapa: It is a platform in front of Garbha Griha where devotes can assemble for praying.

IASbaba
Score:
Web: http://ilp.iasbaba.com/
47.00 /
Email: ilp@iasbaba.com
Page 12 200
2019 - Test 28-
Exam Title :
History & C...
Email : narenips6@gmail.com
Contact :

3) Shikhar: It is the main tower built over the Garbhagriha. It is curvilinear. In many temples,
over mandaps there can be subsidiary Shikhars.

4) Amlaka: It is a circular notch built over the Shikhar.

5) Kalash: It is the top most structure over the Shikhar. It looks like a coconut kalash put on top
of Amlak.

6) Jagati: It is a raised platform in front of Garbhagriha from where aarti is done.

The remains of Dravida architecture are mostly found at Mamallapuram and Kanchipuram

QUESTION 18. MTU4Njk4K0d1bnR1cnUgTmFyZW4gS3VtYXIrbmFyZW5pcHM2QGdtYWlsLmNvbStRVUVTVE


PTiAxNw==
Consider the following statements

1. Brihadeshwara temple at Thanjavur was built by Rajendra Chola

2. Vijayanagar kings introduced the concept of enlarged high enclosure walls and more
decoration on them

3. Krishnadeva Raya installed life size sculptors of him and his wives in the temple

Choose the correct code

a) 1 and 2 only
b) 2 and 3 only
c) 1 and 3 only
d) All of the above
Correct Answer: B
Your Answer: A
Explanation

Solution (b)

Dravidan temples reached their artistic peak during the reign of the Chola Kings. Finest
examples are Brihadeshwara temple at Thanjavur, built about 1010 by Rajaraja 1, & temple at
Gangaikondacolapuram, built about 1025 by his son Rajendra Chola.

Vijayanagar Style:

1) They introduced the concept of enlarged high enclosure walls and more decoration on these
high enclosure walls and Gopuram’s.

2) Sculpture or motif of supernatural horses was used very frequently.

3) They also introduced the concept of secular buildings (Example-Lotus Mahal).

4) They added a Mandap in the left side of the main shrine which is called Kalyan Mandap. It is
used for wedding ceremonies of Gods and Goddesses.

5) Krishnadeva Raya installed life size sculptors of him and his wives in the temple.

IASbaba
Score:
Web: http://ilp.iasbaba.com/
47.00 /
Email: ilp@iasbaba.com
Page 13 200
2019 - Test 28-
Exam Title :
History & C...
Email : narenips6@gmail.com
Contact :

QUESTION 19. MTU4Njk4K0d1bnR1cnUgTmFyZW4gS3VtYXIrbmFyZW5pcHM2QGdtYWlsLmNvbStRVUVTVE


lPTiAxOA==
Consider the following statements

1. Under this temple architecture, the height of the temple towers was reduced even though the
numbers of tiers were retained.

2. The semi-circular structures of the Buddhist chaityas are also borrowed in this style

3. It does not have covered ambulatory around the sanctum

Which temple architecture is being referred to here?

a) Nagara style
b) Vesara style
c) Dravidian style
d) Gothic style
Correct Answer: B
Your Answer: Unanswered
Explanation

Solution (b)

Vesara style

1) It is an amalgamation of Nagara style and Dravidan style and emerged during early medieval
period.

2) Temples built in the Deccan under the later Chalukyas of Kalyani and Hoysalas are
considered examples of this style.

3) Vesara style reduces the height of the temple towers even though the numbers of tiers are
retained. This is accomplished by reducing the height of individual tiers.

4) The semi-circular structures of the Buddhist chaityas are also borrowed in this style, as in
the Durga temple of Aihole.

5) It consists of two principle components like Dravidian style i.e. Vimana & Mandap.

6) Departing from Dravidian style it does not have covered ambulatory around the sanctum.

7) Many temples in Central India and Deccan have used the Vesara style with regional
modifications. The Papanatha temple (680 AD) in particular and someo ther temples to a lesser
extent located at Pattadakal demonstrate panache for this stylistic overlap

QUESTION 20. MTU4Njk4K0d1bnR1cnUgTmFyZW4gS3VtYXIrbmFyZW5pcHM2QGdtYWlsLmNvbStRVUVTVE


PTiAxOQ==
Which of the following architectural features were introduced by Sultanate rulers?

1. Arches

IASbaba
Score:
Web: http://ilp.iasbaba.com/
47.00 /
Email: ilp@iasbaba.com
Page 14 200
2019 - Test 28-
Exam Title :
History & C...
Email : narenips6@gmail.com
Contact :

2. Domes

3. Minarets

4. Beams

5. Brackets

Choose the correct code

a) 1,2 and 3 only


b) 1,4 and 5 only
c) 2,3 and 4 only
d) 2,3 and 5 only
Correct Answer: A
Your Answer: A
Explanation

Solution (a)

The Sultanate architecture can be broadly classified into secular buildings and religious
building.

Secular buildings include forts, Palaces, Pillars, Sarais and Hauz (Artificial lakes) while
Religious structures include mosques, mausoleums, Dargahs and madarsas.

Sultans freely employed Indian architects, Masons and workers. They also introduced the ideas
of Central Asian Architecture. This made Sultanate architecture and amalgamation of Islamic
and Indian style.

Most of the initial buildings were made after destroying the previous structures and using the
raw material from them. Quwat-ul-Islam Mosque in Delhi is said to have been built by Qutub-ud-
Din Aibak by demolishing a Hindu temple, which was built over a Jain temple. Likewise ‘Adai-
din-ka Jhopra’ a mosque in Ajmer, built in 2 ½ days came into being on the ruins of a Hindu
building.

Arches (Mehrab), domes and minarets were introduced by Sultanate rulers. The previous
indigenous architecture was based on beams and brackets which was called ‘trabeated’ style.
On the otherhand, the Islamic architecture was ‘Arcuate’ which used arches and vaults to
bridge the spaces and supported the building of graceful domes instead of flat roofs.

QUESTION 21. MTU4Njk4K0d1bnR1cnUgTmFyZW4gS3VtYXIrbmFyZW5pcHM2QGdtYWlsLmNvbStRVUVTVE


PTiAyMA==
Which of the following are correctly matched?

Structure Speciality

1. Balban’s tomb A) First true arch

2. Jamat Khana Mosque B) First scientific dome

3. Tomb of Sikander Lodhi C) First double dome

IASbaba
Score:
Web: http://ilp.iasbaba.com/
47.00 /
Email: ilp@iasbaba.com
Page 15 200
2019 - Test 28-
Exam Title :
History & C...
Email : narenips6@gmail.com
Contact :

Choose the correct code

a) 1 and 2 only
b) 2 and 3 only
c) 1 and 3 only
d) All of the above
Correct Answer: D
Your Answer: Unanswered
Explanation

Solution (d)

Balban’s Tomb – Balban built his own tomb which is considered to have first true arch

Ala ud din Khilji built new city called Siri (in Delhi) and built Jamat Khana Mosque .He also built
the Alai Darwaza. Alai Darwaza is the entrance to the Jamat Khana mosque. This has the first
scientific dome. Horse shoe arch was used in the building.

Lodhis had a very stable tenure and hence they got a chance to build.

Important Features:

Buildings were made on a raised platform with gardens around them. This was made to copy
the theme of heaven, later, Mughals used the same concept in their buildings.

The tombs were made in octagonal shapes. This feature was also adopted by the Mughals.

For the first time a double dome was introduced in the tomb of Sikander Lodhi.

QUESTION 22. MTU4Njk4K0d1bnR1cnUgTmFyZW4gS3VtYXIrbmFyZW5pcHM2QGdtYWlsLmNvbStRVUVTVE


PTiAyMQ==
Consider the following statements

1. The Qutub Minar was constructed entirely by using red sandstone

2. Tughlaqabad fort was entirely made of granite as it was readily available

Choose the correct code

a) 1 only
b) 2 only
c) Both 1 and 2
d) None of the above
Correct Answer: B
Your Answer: B
Explanation

Solution (b)

Qutub Minar is probably the most important structure of the Sultanate period. It was built in
honour of Sufi Saint Qutub ud din Bakhtiyar Kaki. The construction was started by Qutub ud din

IASbaba
Score:
Web: http://ilp.iasbaba.com/
47.00 /
Email: ilp@iasbaba.com
Page 16 200
2019 - Test 28-
Exam Title :
History & C...
Email : narenips6@gmail.com
Contact :
Aibak, next three floors were made by Iltutmish and final two floors were reconstructed by
Firoz Shah Tughlaq after the top storey was destroyed by lightning.

Minar is 73m high and has got 5 storeys. They have beautiful geometric designs and Arabesque.

Balconies at each floor are intricately carved with flower and geometric designs. They are also
designed in such a fashion as if they are holding the next floor.

The first three storeys have been constructed by red sandstone and last two are made
of marble.

Ghayas ud din Tughlaq constructed the new city of Tughlaqabad. Tughlaqabad fort was built
during this period. It is made of granite as it was readily available . Since granite is
difficult to polish the texture of the building is rough. The boundary wall of this fort is sloping
instead of straight.

QUESTION 23. MTU4Njk4K0d1bnR1cnUgTmFyZW4gS3VtYXIrbmFyZW5pcHM2QGdtYWlsLmNvbStRVUVTVE


PTiAyMg==
Consider the following statements about Mughal Architecture

1. All the buildings of this Mughal architecture were theme based

2. Mughals buildings were a mix of Indo – Gothic style

3. Initially marble was used because of its easy availability. Later red sandstone was extensively
used

Choose the correct code

a) 1 only
b) 2 and 3 only
c) 1 and 3 only
d) All of the above
Correct Answer: A
Your Answer: Unanswered
Explanation

Solution (a)

Salient Features of Mughal Architecture

1) Mughals buildings were a mix of Indo – Islamic style. In Indian styles, Rajput and Buddhist
influence is more dominant. In Islamic style, Iranian and Central Asian style is dominant.

2) The structures are generally big and lofty displaying the wealth and strength of empire.

3) The king used to take personal interest right from design and construction. Hence usually a
centralized plan was made which artists were not allowed to change in between.

4) Pillars, dome, arches, squinches and plinth were main features.

5) All the buildings were theme based. Special emphasis was given on gardens, geometry and
symmetry of design.

IASbaba
Score:
Web: http://ilp.iasbaba.com/
47.00 /
Email: ilp@iasbaba.com
Page 17 200
2019 - Test 28-
Exam Title :
History & C...
Email : narenips6@gmail.com
Contact :

6) Initially red sandstone was used because of its easy availability. Later marble was

7) extensively used. Lime was used as binding material.

8) Precious and semi-precious stones were used for decoration. They were brought from Iran
and central Asia.

9) Calligraphic work and arabesque on the walls were used frequently. The one on Taj mahal is
most famous. Persian styled double domes (onion shaped) were made.

QUESTION 24. MTU4Njk4K0d1bnR1cnUgTmFyZW4gS3VtYXIrbmFyZW5pcHM2QGdtYWlsLmNvbStRVUVTVE


PTiAyMw==
Consider the following statements about Gandhara School of Art

1. It displays Greek artistic influence and the concept of the man-god which was essentially
inspired by Greek mythological culture.

2. The Gandhara art primarily depicted the Buddhist themes

3. Gandhara School of art enjoyed the liberal patronage of Kushana rulers

Choose the correct code

a) 1 only
b) 2 and 3 only
c) 1 and 3 only
d) All of the above
Correct Answer: D
Your Answer: Unanswered
Explanation

Solution (d)

Gandhara School of art enjoyed the liberal patronage of Kushana rulers especially Kanishka.
The Gandhara art primarily depicted the Buddhist themes. It displays Greek artistic influence
and the concept of the “man-god” was essentially inspired by Greek mythological culture.
Artistically, the Gandharan School of sculpture is said to have contributed wavy hair, drapery
covering shoulders, shoes and sandals etc

QUESTION 25. MTU4Njk4K0d1bnR1cnUgTmFyZW4gS3VtYXIrbmFyZW5pcHM2QGdtYWlsLmNvbStRVUVTVE


lPTiAyNA==
Consider the following statements

1. Gundecha brothers are well known exponents of Drupad style of Hindustani Classical Music

2. Thyagaraja, Shamashastri and Muthuswami Dikshitar are referred to as the "trinity" of


Carnatic music

Choose the correct code

IASbaba
Score:
Web: http://ilp.iasbaba.com/
47.00 /
Email: ilp@iasbaba.com
Page 18 200
2019 - Test 28-
Exam Title :
History & C...
Email : narenips6@gmail.com
Contact :

a) 1 only
b) 2 only
c) Both 1 and 2
d) None of the above
Correct Answer: C
Your Answer: Unanswered
Explanation

Solution (c)

Gundecha brothers are well known exponents of Drupad style of Hindustani Classical Music

Dhrupad is a genre in Hindustani classical music from the Indian subcontinent. It is the oldest
style of music major vocal styles associated with Hindustani classical music, Haveli Sangeet and
also related to the South Indian Carnatic tradition. The term denotes both the verse form of the
poetry and the style in which it is sung. It is spiritual, heroic, thoughtful, virtuous, embedding
moral wisdom or solemn form of song-music combination. A Dhrupad has at least four stanzas,
called Sthayi (or Asthayi), Antara, Sancari and Abhoga.

Carnatic music or Carnatic sangeet is the south Indian classical music. Carnatic music has a
rich history and tradition and is one of the gems of world music. Carnatic Sangeet has
developed in the south Indian states of Tamil Nadu, Kerala, Andhra Pradesh and Karnataka.
These states are known for their strong presentation of Dravidian culture. Purandardas
(1480-1564) is considered to be the father of Carnatic music. To him goes the credit of
codification of the method of Carnatic music. He is also credited with creation of several
thousand songs. Another great name associated with Carnatic music is that of Venkat Mukhi
Swami. He is regarded as the grand theorist of Carnatic music. He also developed "Melankara",
the system for classifying south Indian ragas.

It was in the 18th century that Carnatic music acquired its present form. This was the period
that saw the "trinity" of Carnatic music; Thyagaraja, Shamashastri and Muthuswami Dikshitar
compile their famous compositions.

QUESTION 26. MTU4Njk4K0d1bnR1cnUgTmFyZW4gS3VtYXIrbmFyZW5pcHM2QGdtYWlsLmNvbStRVUVTVE


PTiAyNQ==
Who among the following is revered as the second Budhha?

a) Padmasmbava
b) Samanthabhadra
c) Avalokiteshwara
d) Vajrapani
Correct Answer: A
Your Answer: A
Explanation

Solution (a)

Padmasambhava also known as Guru Rinpoche, was an 8th-century Buddhist master from the
Indian subcontinent. Although there was a historical Padmasambhava, little is known of him
apart from helping the construction of the first Buddhist monastery in Tibet at Samye, at the
behest of Trisong Detsen, and shortly thereafter leaving Tibet due to court intrigues.

IASbaba
Score:
Web: http://ilp.iasbaba.com/
47.00 /
Email: ilp@iasbaba.com
Page 19 200
2019 - Test 28-
Exam Title :
History & C...
Email : narenips6@gmail.com
Contact :

A number of legends have grown around Padmasambhava's life and deeds, and he is widely
venerated as a "second Buddha" by adherents of Tibetan Buddhism in Tibet, Nepal, Bhutan, the
Himalayan states of India, and elsewhere.

In Tibetan Buddhism, he is a character of a genre of literature called terma, an emanation of


Amitabha that is said to appear to tertons in visionary encounters and a focus of guru yoga
practice, particularly in the Rime schools. The Nyingma School considers Padmasambhava to be
a founder of their tradition.

QUESTION 27. MTU4Njk4K0d1bnR1cnUgTmFyZW4gS3VtYXIrbmFyZW5pcHM2QGdtYWlsLmNvbStRVUVTVE


lPTiAyNg==
Consider the following statements

1. Atharva veda prescribes the rituals for performing different sacrifices

2. Bharat’s Natyashashtra is based upon Gandharvaveda which itself is a upveda of Yajurveda

3. Yajurveda describes the popular beliefs and superstitions of the humble folk

Choose the correct code

a) 1 and 2 only
b) 2 and 3 only
c) 1 and 3 only
d) None of the above
Correct Answer: D
Your Answer: Unanswered
Explanation

Solution (d)

“Yajus” means "sacrificial formula" and Yajurveda is the book of sacrificial prayers. It contains
the rituals of the Yajnas. It is estimated to have been composed between 1,400 and 1000 BC.

It prescribes the rituals for performing different sacrifices.

Gandharvaveda is Sam Veda’s Upveda is is a technical treatise on Music, Dance and Drama.
Bharat’s Natyashashtra is based upon Gandharvaveda

Atharva-Veda is entirely different from the other three Vedas and is chronologically the last of
the four.

It is important and interesting as it describes the popular beliefs and superstitions of the
humble folk.

Atharvaveda contains the magic spells, incorporates much of early traditions of healing and
magic that are paralleled in other Indo-European literatures.

QUESTION 28. MTU4Njk4K0d1bnR1cnUgTmFyZW4gS3VtYXIrbmFyZW5pcHM2QGdtYWlsLmNvbStRVUVTVE


PTiAyNw==

IASbaba
Score:
Web: http://ilp.iasbaba.com/
47.00 /
Email: ilp@iasbaba.com
Page 20 200
2019 - Test 28-
Exam Title :
History & C...
Email : narenips6@gmail.com
Contact :

Consider the following statements

1. Shiksha is a vedanga which aims at the teaching of the correct pronunciation of the Vedic
hymns and mantras

2. Srautasutras, which are based on the Shruti teach the performance of the great sacrifices,
requiring three or five sacrificial fires

3. Chhanda vedanga measures and divides Vedic Mantras by number of padas in a verse

Choose the correct code

a) 1 and 2 only
b) 2 and 3 only
c) 1 and 3 only
d) All of the above
Correct Answer: D
Your Answer: Unanswered
Explanation

Solution (d)

Vedangas are the first series in the Smriti Literature. These refer to six auxiliary disciplines
associated with the study and understanding of the Vedas. They are as follows:

1. Shiksha (Phonetics) - Its aim is the teaching of the correct pronunciation of the Vedic hymns
and mantras.

2. Kalpa (Ritual Canon) - It contains the sacrificial practice and systematic sutras. Srautasutras,
which are based on the Shruti, and teach the performance of the great sacrifices, requiring
three or five sacrificial fires.

Smartasutras, or rules based on the Smriti or tradition.

The Dharmasutras are the first four texts of the Dharmasastra tradition and they focus on the
idea of dharma, the principal guide by which Hindus strive to live their lives.

3. Vyakaran (Grammar) - Vyakaran includes the Aṣṭadhyayi, of Panini. Most of the work of very
early Indian grammarians ranging to 8th century BC is lost.

4. Nirukta (explanation) - It is traditionally attributed to Yaska, an ancient Sanskrit grammarian.


It deals with etymology, particularly of obscure words, especially those occurring in the Veda.

5. Chhanda (Vedic meter) - It measures and divides Vedic Mantras by number of padas in a
verse. Number of padas divides each verse, hymn, or mantra and number of syllables divides
each pada.

6. Jyotisha (Astrology) - It describes rules for tracking the motions of the sun and the moon and
the foundation of Vedic Jyotish.

QUESTION 29. MTU4Njk4K0d1bnR1cnUgTmFyZW4gS3VtYXIrbmFyZW5pcHM2QGdtYWlsLmNvbStRVUVTVE


lPTiAyOA==
Which of the following are correctly matched?

IASbaba
Score:
Web: http://ilp.iasbaba.com/
47.00 /
Email: ilp@iasbaba.com
Page 21 200
2019 - Test 28-
Exam Title :
History & C...
Email : narenips6@gmail.com
Contact :

Art forms Place

1. Saura A) Arunachal Pradesh

2. Kalbelia B) Odisha

3. Daminda C) Rajasthan

4. Silambam D) Kerala

Choose the correct code

a) 1 and 2 only
b) 2 and 4 only
c) 3 and 4 only
d) None of the above
Correct Answer: D
Your Answer: A
Explanation

Solution (d)

Saura is a form of painting practiced by Saura tribe in Odisha which is similar to Warli but
larger and elongated. Natural colors are used

Silambam is a type of Martial art practiced in Tamil Nadu – Weapon based martial art. Uses
animal movements of tiger, snake, eagle forms and foot work patterns plays an important role;
Kuttuvarasai – part of silambam – unarmed version.

Kalbelia is dance form of Rajastha which consists of swirling, graceful movements that make
this dance a treat to behold. Women dance while men provide music for the performance.
Kalbelia dance and songs are a part of UNESCO’s representative list of Intangible Cultural
Heritage of Humanity. Musical instrument used – Pungi – the one used by snake charmers

Daminda is an art form of Apatani tribe in Arunachal Pradesh

Performed by girls during Dree festival

QUESTION 30. MTU4Njk4K0d1bnR1cnUgTmFyZW4gS3VtYXIrbmFyZW5pcHM2QGdtYWlsLmNvbStRVUVTVE


PTiAyOQ==
Which of the following are included under the UNESCO's Representative List of the Intangible
Cultural Heritage of Humanity from India?

1. Kutiyattam, Sanskrit theatre

2. Mudiyettu, ritual theatre and dance drama of Kerala

3. Nawrouz

4. Kumbha mela

5. Krishnattam

IASbaba
Score:
Web: http://ilp.iasbaba.com/
47.00 /
Email: ilp@iasbaba.com
Page 22 200
2019 - Test 28-
Exam Title :
History & C...
Email : narenips6@gmail.com
Contact :

6. Sankirtana, ritual singing, drumming and dancing of Manipur

Choose the correct code

a) 1,2,3 and 5 only


b) 1,2,3,4 and 6 only
c) 1,3,4,5 and 6 only
d) 1,2,4,5 and 6 only
Correct Answer: B
Your Answer: D
Explanation

Solution (b)

A total of 13 Intangible cultural heritage (ICH) elements from India have been inscribed till date
on the UNESCO’s Representative List of the Intangible Cultural Heritage of Humanity.

For inclusion of an element in the UNESCO’s Representative List of Intangible Cultural


Heritage, the state parties are required to submit nomination dossier on the relevant element
for evaluation and examination of the UNESCO Committee.

The Ministry of Culture has appointed the Sangeet Natak Akademi, an autonomous organisation
under the Ministry of Culture, as nodal office for matters relating to the intangible cultural
heritage including for preparation of the nomination dossiers for the Representative List of
UNESCO. The Sangeet Natak Akademi (SNA) accordingly makes necessary interaction with the
stakeholders, experts/officials etc. prior to finalization of the dossier in respect of element
identified for the nomination.

Representative List of the Intangible Cultural Heritage of Humanity

ICH Element - Year of Inscription

1) Tradition of Vedic chanting - 2008

2) Ramlila, the traditional performance of the Ramayana - 2008

3) Kutiyattam, Sanskrit theatre - 2008

4) Ramman, religious festival and ritual theatre of the Garhwal Himalayas, India 2009

5) Mudiyettu, ritual theatre and dance drama of Kerala - 2010

6) Kalbelia folk songs and dances of Rajasthan - 2010

7) Chhau dance - 2010

8) Buddhist chanting of Ladakh: recitation of sacred Buddhist texts in the trans-Himalayan


Ladakh region, Jammu and Kashmir, India - 2012

9) Sankirtana, ritual singing, drumming and dancing of Manipur - 2013

10) Traditional brass and copper craft of utensil making among the Thatheras of Jandiala Guru,
Punjab, India - 2014

11) Yoga - 2016

IASbaba
Score:
Web: http://ilp.iasbaba.com/
47.00 /
Email: ilp@iasbaba.com
Page 23 200
2019 - Test 28-
Exam Title :
History & C...
Email : narenips6@gmail.com
Contact :

12) Nawrouz, Novruz, Nowrouz, Nowrouz, Nawrouz, Nauryz, Nooruz, Nowruz, Navruz, Nevruz,
Nowruz, Navruz - 2016

13) Kumbh Mela – 2017

QUESTION 31. MTU4Njk4K0d1bnR1cnUgTmFyZW4gS3VtYXIrbmFyZW5pcHM2QGdtYWlsLmNvbStRVUVTVE


PTiAzMA==
Which of the following works are attributed to Asvaghosha?

1. Buddha Charita

2. Sariputra Prakaran

3. Saundaranandakavya

4. Swapnavasavadattam

Choose the correct code

a) 1,2 and 3 only


b) 2,3 and 4 only
c) 1,3 and 4 only
d) 1,2 and 4 only
Correct Answer: A
Your Answer: A
Explanation

Solution (a)

Asvaghoṣa (Circa. 80 – 150 AD) is considered to be the first Sanskrit Dramatist of the world. He
was a born Brahmin, but later turned into a Buddhist Monk.

He was the court poet of the Kushana king Kanishka.

Asvaghosha wrote in Classical Sanskrit. His work, Buddha Charita is an epic style Sanskrit
work. It mainly deals with Buddha's Life.

Asvaghosa also wrote a Sanskrit Drama “Sariputra Prakaran” which deals about Sariputta or
Sariputra the disciple of Buddha.

Asvaghosha also wrote the Saundaranandakavya, a kavya poem with the theme of conversion of
Nanda, Buddha‟s half-brother, so that he might reach salvation.

QUESTION 32. MTU4Njk4K0d1bnR1cnUgTmFyZW4gS3VtYXIrbmFyZW5pcHM2QGdtYWlsLmNvbStRVUVTVE


PTiAzMQ==
It is an epic poem which has 17 sirgs, out of which only 8 are accepted as his authorship. The
prince is Kartikeya and it refers to birth of Kartikeya, son of Shiva and Parvati after a lot of
Tapasya to win Shiva who had already won Kamdeva (God of Love). Kartikeya later killed

IASbaba
Score:
Web: http://ilp.iasbaba.com/
47.00 /
Email: ilp@iasbaba.com
Page 24 200
2019 - Test 28-
Exam Title :
History & C...
Email : narenips6@gmail.com
Contact :
Tarakasur demon who was blessed that he would not be killed by any other than son of Shiva
and Parvati.

Which of the following work is being referred to here?

a) Raghuvamsa
b) Kumarasambhava
c) Ritusamhara
d) Mricchakatika
Correct Answer: B
Your Answer: B
Explanation

Solution (b)

Raghuvamsa - Raghuvamsa is a Sanskrit epic poem that is a long (19 Sargas) narration of
genealogy of Lord Rama's Raghu Vamsa beginning with King Dileep up to Agnivarna.

Kumarasambhava - Kumarasambhava is an epic poem which has 17 sirgs, out of which only 8
are accepted as his authorship. Kumara or prince is Kartikeya and it refers to birth of Kartikeya,
son of shiva and Parvati after a lot of Tapasya to win Shiva who had already won Kamdeva (God
of Love).

Kartikeya later killed Tarakasur demon who was blessed that he would not be killed by any
other than son os Shiva and Parvati.

Ritusamhara - Ritusamhara is again a mini epic poem in Sanskrit which has 6 Sargas. These
Sargas refer to 6 seaosns (Ritu) viz, Grisma (Summer), varsha (Rains), Sharad (Autumn),
Hemanta (Cool), Sisira (Winter) and Vasantha (Spring). It mentions the feelings, emotions and
experiences of lovers in 6 seasons. Ritusamhara is considered to be the earliest work of
Kalidasa.

QUESTION 33. MTU4Njk4K0d1bnR1cnUgTmFyZW4gS3VtYXIrbmFyZW5pcHM2QGdtYWlsLmNvbStRVUVTVE


PTiAzMg==
Consider the following statements about the Bronze Nataraja statue of Cholas

1. In this sculpture Shiva has been shown balancing himself on his left leg and suppressing the
apasmara, the demon of ignorance with the foot of the same leg

2. His four arms are outstretched and the main right hand is posed in abhaya hasta

3. The upper right holds the damaru his favourite musical instrument to keep on the beat tala

Choose the correct code

a) 1 and 2 only
b) 2 and 3 only
c) 1 and 3 only
d) All of the above
Correct Answer: B
Your Answer: D

IASbaba
Score:
Web: http://ilp.iasbaba.com/
47.00 /
Email: ilp@iasbaba.com
Page 25 200
2019 - Test 28-
Exam Title :
History & C...
Email : narenips6@gmail.com
Contact :
Explanation

Solution (b)

Shiva is associated with the end of the cosmic world with which this dancing position is
associated.

In this Chola period bronze sculpture he has been shown balancing himself on his right leg and
suppressing the apasmara, the demon of ignorance or forgetfulness, with the foot of the same
leg.

At the same time he raises his left leg in bhujangatrasita stance, which represents tirobhava,
that is kicking away the veil of maya or illusion from the devotee’s mind.

His four arms are outstretched and the main right hand is posed in abhaya hasta or the gesture
suggesting.

The upper right holds the damaru his favourite musical instrument to keep on the beat tala.

The upper left hand carries a flame while the main left hand is held in dola hasta and connects
with the abhaya hasta of the right hand.

His hair locks fly on both the sides touching the circular jvala mala or the garland of flames
which surrounds the entire dancing figuration.

QUESTION 34. MTU4Njk4K0d1bnR1cnUgTmFyZW4gS3VtYXIrbmFyZW5pcHM2QGdtYWlsLmNvbStRVUVTVE


PTiAzMw==
Which of the following features are unique to Ellora caves?

1. It has monasteries associated with the three religions

2. Confluence of many styles at one place

3. The triple storeyed caves

4. A rock-cut temple carved out of a single rock

Choose the correct code

a) 1,2 and 3 only


b) 2,3 and 4 only
c) 1,3 and 4 only
d) All of the above
Correct Answer: D
Your Answer: D
Explanation

Solution (d)

Ellora is located a hundred kilometres from Ajanta and has thirty-four Buddhist, Brahmanical
and Jain caves.

IASbaba
Score:
Web: http://ilp.iasbaba.com/
47.00 /
Email: ilp@iasbaba.com
Page 26 200
2019 - Test 28-
Exam Title :
History & C...
Email : narenips6@gmail.com
Contact :

It is a unique art-historical site in the country as it has monastries associated with the three
religions dating from the fifth century CE onwards to the eleventh century CE.

It is also unique in terms of stylistic eclecticism, i.e., confluence of many styles at one place.
The caves of Ellora and Aurangabad show the ongoing differences between the two religions—
Buddhism and Brahmanical. There are twelve Buddhist caves having many images belonging to
Vajrayana Buddhism like Tara, Mahamayuri, Akshobhya, Avalokiteshwara, Maitrya, Amitabha,
etc. Buddhist caves Ajanta also has excavated double-storeyed caves but at Ellora, the triple
storey is a unique achievement. All the caves were plastered and painted but nothing visible is
left.

Cave No. 16 is known as Kailash leni. A rock-cut temple has been carved out of a single rock, a
unique achievement of the artisans.

QUESTION 35. MTU4Njk4K0d1bnR1cnUgTmFyZW4gS3VtYXIrbmFyZW5pcHM2QGdtYWlsLmNvbStRVUVTVE


PTiAzNA==
Consider the following statements about Mohiniyattam

1. It is a solo dance performed by female artists where the song is customarily in Manipravala
which is a mix of Sanskrit and Malayalam language

2. Traditionally the repertoire of Mohiniyattam follows two performance categories namely,


‘Nritta’ and ‘Nritya’

3. It follows the Lasya type of dance that showcases a more graceful, gentle and feminine form
of dancing and displays excellence in ‘Ekaharya Abhinaya’ form

Choose the correct code

a) 1 and 2 only
b) 2 and 3 only
c) 1 and 3 only
d) All of the above
Correct Answer: D
Your Answer: Unanswered
Explanation

Solution (d)

Mohiniattam or Mohiniyattam is an Indian classical dance form that evolved in the state of
Kerala, India, and is counted among the two popular dance arts of the state, the other being
Kathakali. Although its roots date back to the age-old Sanskrit Hindu text on performing arts
called ‘Natya Shastra’, similar to other Indian classical dance forms, Mohiniattam adheres to
the Lasya type that showcases a more graceful, gentle and feminine form of dancing.
Mohiniattam derives its name from the word ‘Mohini’, a female avatar of Lord Vishnu.
Conventionally a solo dance performed by female artists, it emotes a play through dancing and
singing where the song is customarily in Manipravala which is a mix of Sanskrit and Malayalam
language and the recitation may be either performed by the dancer herself or by a vocalist with
the music style being Carnatic.

IASbaba
Score:
Web: http://ilp.iasbaba.com/
47.00 /
Email: ilp@iasbaba.com
Page 27 200
2019 - Test 28-
Exam Title :
History & C...
Email : narenips6@gmail.com
Contact :

Traditionally the repertoire of Mohiniyattam follows two performance categories namely,


‘Nritta’ and ‘Nritya’ mentioned in ‘Natya Shastra’. It follows the Lasya type of dance that
showcases a more graceful, gentle and feminine form of dancing and displays excellence in
‘Ekaharya Abhinaya’ form in other words a solo and expressive dance art complimented with
music and songs.

QUESTION 36. MTU4Njk4K0d1bnR1cnUgTmFyZW4gS3VtYXIrbmFyZW5pcHM2QGdtYWlsLmNvbStRVUVTVE


PTiAzNQ==
Consider the following statements

1. This dance form includes themes from Vaishnavism and others associated with Hindu gods
and goddesses like Shiva, Surya and Shakti

2. All the 108 fundamental dance units elucidated in ‘Natya Shastra’ are similar to this art form

3. This dance form incorporates Indian ragas, both from south and north that indicate exchange
of concepts and performance arts between the two parts of India

Which of the following dance form is being referred to here?

a) Kuchipudi
b) Manipuri
c) Odissi
d) Kathak
Correct Answer: C
Your Answer: Unanswered
Explanation

Solution (c)

Odissi or Orissi is one of the pre-eminent classical dance forms of India which originated in the
Hindu temples of the eastern coastal state of Odisha in India. Its theoretical base trace back to
‘Natya Shastra’, the ancient Sanskrit Hindu text on the performing arts. Age-old tradition of
Odissi is manifested from Odisha Hindu temples and various sites of archaeological significance
that are associated with Hinduism, Jainism and Buddhism, the sculptures of which adorn dance
postures of this art form. A form of illustrative anecdote of mythical and religious stories,
devotional poems and spiritual ideas emoted by dancer with excellent body movements,
expressions, impressive gestures and sign languages, its performance repertoire includes
invocation, nrita, nritya, natya, and moksha. This dance form includes themes from Vaishnavism
and others associated with Hindu gods and goddesses like Shiva, Surya and Shakti.

The antiquity of this dance form is evident from its roots that trace back to the ancient Sanskrit
Hindu text called ‘Natya Shastra’ which deals with different performing arts. All the 108
fundamental dance units elucidated in ‘Natya Shastra’ are similar to this art form. It
encompasses thousands of verses that are structured in various chapters.

The unique feature of this dance form is that it incorporates Indian ragas, both from south and
north that indicate exchange of concepts and performance arts between the two parts of India.
‘Shokabaradi’, ‘Karnata’, ‘Bhairavee’, ‘Dhanashri’, ‘Panchama’, ‘Shree Gowda’, ‘Nata’, ‘Baradi’
and ‘Kalyana’ are the main ragas of Odissi. The musical instruments include tabla, pakhawaj,
harmonium, cymbals, violin, flute, sitar and Swarmandal.

IASbaba
Score:
Web: http://ilp.iasbaba.com/
47.00 /
Email: ilp@iasbaba.com
Page 28 200
2019 - Test 28-
Exam Title :
History & C...
Email : narenips6@gmail.com
Contact :

QUESTION 37. MTU4Njk4K0d1bnR1cnUgTmFyZW4gS3VtYXIrbmFyZW5pcHM2QGdtYWlsLmNvbStRVUVTVE


lPTiAzNg==
Consider the following statements:

1. Angkorwat temple was built in 12 th century in Thailand.

2. This is the largest of any world heritage site which has been digitally captured.

3. It is dedicated to lord Shiva.

4. It was built by kind Suryavarman II of Khmer dynasty.

Which of the above statements are correct:

a) All of the above.


b) 1,2 and 3
c) 2 and 4
d) 1 and 3
Correct Answer: C
Your Answer: C
Explanation

Solution (c)

Angkorwat temple was built by king Suryavarman II of Khmer dynasty in 12 th century in


Cambodia(Kamboja). It is dedicated to lord Vishnu.

Google Street View mapped >90000 images of Angkor Vat. You can view at home, 360 degree
tour. (Same available for Mt.Fiji and Tajhmahal) . So far this is the largest digital capturing of
any world heritage site.

QUESTION 38. MTU4Njk4K0d1bnR1cnUgTmFyZW4gS3VtYXIrbmFyZW5pcHM2QGdtYWlsLmNvbStRVUVTVE


PTiAzNw==
consider the following regarding ‘Bhagvatism’:

1. It centered on the worship of Vishnu.

2. It discouraged the idol worship.

3. It preached the doctrine of Incarnation.

4. It promoted the sacrifices of animals.

Choose the correct option:

a) 1 and 3
b) 1,2 and 3
c) 2,3 and 4

IASbaba
Score:
Web: http://ilp.iasbaba.com/
47.00 /
Email: ilp@iasbaba.com
Page 29 200
2019 - Test 28-
Exam Title :
History & C...
Email : narenips6@gmail.com
Contact :

d) all of the above


Correct Answer: A
Your Answer: A
Explanation

Solution (a)

· Bhagvatism centered around the worship of Vishnu or Bhagwat and originated in post Maurya
times. He was called Bhagvat and his worshippers were called bhagwatas.

· Bhagvatism was marked by Bhakti and Ahimsa. Bhakti meant the offer of loving devotion.
Ahimsa or the doctrine of non-killing of animals suited the agricultural society. Out of their
aversion to killing of animals some of them took only vegetarian food.

· It preached the doctrine of incarnation, or Avtara. History was presented as a cycle of ten
incarnations of Vishnu. It was believed that whenever the social order faced crisis, Vishnu
appeared in an appropriate form to save it.

· Idol worship in the temples became a common feature after Bhagvatism. Temples based on
modern lines were started during Gupta period.

QUESTION 39. MTU4Njk4K0d1bnR1cnUgTmFyZW4gS3VtYXIrbmFyZW5pcHM2QGdtYWlsLmNvbStRVUVTVE


lPTiAzOA==
Consider the following statements

1. The 2018 edition of the 'Mountain Echoes Literature Festival' was held at Himachal Pradesh

2. International water colour festival, 2018 was held in Ranchi

Choose the correct code

a) 1 only
b) 2 only
c) Both 1 and 2
d) None of the above
Correct Answer: B
Your Answer: C
Explanation

Solution (b)

In Bhutan, the ninth edition of Mountain Echoes Literature Festival was held in Thimphu. The
festival is an initiative of the India-Bhutan Foundation and India’s leading literary agency,
Siyahi. The 2018 edition of the Festival celebrated 50 years of formal diplomatic relations
between India and Bhutan.

The international watercolour festival was organised in Ranchi, Jharkhand. It was organised by
the Department of Tourism, Art-Culture, Sports & Youth Affairs.

The international watercolour festival aims to bring local artists in touch with the international
art and promote the rich art and culture of Jharkhand.

IASbaba
Score:
Web: http://ilp.iasbaba.com/
47.00 /
Email: ilp@iasbaba.com
Page 30 200
2019 - Test 28-
Exam Title :
History & C...
Email : narenips6@gmail.com
Contact :

QUESTION 40. MTU4Njk4K0d1bnR1cnUgTmFyZW4gS3VtYXIrbmFyZW5pcHM2QGdtYWlsLmNvbStRVUVTVE


PTiAzOQ==
Consider the following statements about Indian philosophy

1. The followers of Samkhya school of philosophy believe that obtaining valid knowledge is the
only way to gain release from suffering

2. Nyaya school postulates that everything in reality stems from Purusha and Prakriti and
liberation occurs with the realization that the soul and the dispositions of matter are same

3. Purva mimamsa school requires unquestionable faith in the Vedas and the regular
performance of the Vedic fire-sacrifices to sustain all the activity of the universe

Choose the correct code

a) 1 and 2 only
b) 2 and 3 only
c) 3 only
d) 1 and 3 only
Correct Answer: C
Your Answer: D
Explanation

Solution (c)

Indian Philosophy refers to any of several traditions of philosophical thought that originated in
the Indian subcontinent, including Hindu philosophy, Buddhist philosophy, and Jain philosophy.

Samkhya is the oldest of the orthodox philosophical systems, and it postulates that everything
in reality stems from Purusha (self or soul or mind) and Prakriti (matter, creative agency,
energy). It is a dualist philosophy, although between the self and matter rather than between
mind and body as in the Western dualist tradition, and liberation occurs with the realization
that the soul and the dispositions of matter (steadiness, activity and dullness) are different.

The Nyaya school is based on the Nyaya Sutras, written by Aksapada Gautama in the 2nd
Century B.C. Its methodology is based on a system of logic that has subsequently been adopted
by the majority of the Indian schools, in much the same way as Aristotelian logic has influenced
Western philosophy. Its followers believe that obtaining valid knowledge (the four sources of
which are perception, inference, comparison and testimony) is the only way to gain release
from suffering. Nyaya developed several criteria by which the knowledge thus obtained was to
be considered valid or invalid (equivalent in some ways to Western analytic philosophy).

The main objective of the Purva Mimamsa school is to interpret and establish the authority of
the Vedas. It requires unquestionable faith in the Vedas and the regular performance of the
Vedic fire-sacrifices to sustain all the activity of the universe. Although in general the Mimamsa
accept the logical and philosophical teachings of the other schools, they insist that salvation can
only be attained by acting in accordance with the prescriptions of the Vedas. The school later
shifted its views and began to teach the doctrines of Brahman and freedom, allowing for the
release or escape of the soul from its constraints through enlightened activity.

IASbaba
Score:
Web: http://ilp.iasbaba.com/
47.00 /
Email: ilp@iasbaba.com
Page 31 200
2019 - Test 28-
Exam Title :
History & C...
Email : narenips6@gmail.com
Contact :
QUESTION 41. MTU4Njk4K0d1bnR1cnUgTmFyZW4gS3VtYXIrbmFyZW5pcHM2QGdtYWlsLmNvbStRVUVTVE
PTiA0MA==
Pravasi Bhartiya Diwas was shifted to coincide with Prayag Kumbh mela. At which of the
following places, Kumbh Mela is not held?

a) Haridwar
b) Nashik
c) Ujjain
d) Puri
Correct Answer: D
Your Answer: D
Explanation

Solution (d)

Kumbh Mela or Kumbha Mela is a mass Hindu pilgrimage of faith in which Hindus gather to
bathe in a sacred or holy river.

Traditionally, four fairs are widely recognized as the Kumbh Melas: the Prayagraj Kumbh Mela,
Haridwar Kumbh Mela, the Nashik-Trimbakeshwar Simhastha, and Ujjain Simhastha.

These four fairs are held periodically at one of the following places by rotation: Allahabad
(Prayagraj), Haridwar, Nashik district (Nashik and Trimbak), and Ujjain. The main festival site
is located on the banks of a river: the Ganges (Ganga) at Haridwar; the confluence (Sangam) of
the Ganges and the Yamuna and the invisible Sarasvati at Allahabad; the Godavari at Nashik;
and the Shipra at Ujjain. Bathing in these rivers is thought to cleanse a person of all their sins.

At any given place, the Kumbh Mela is held every 12 years. There is a difference of around 3
years between the Kumbh Melas at Haridwar and Nashik; the fairs at Nashik and Ujjain are
celebrated in the same year or one year apart. The exact date is determined, following the
Vikram Samvat calendar and the principles of Jyotisha, according to a combination of zodiac
positions of Jupiter, the Sun and the Moon.

QUESTION 42. MTU4Njk4K0d1bnR1cnUgTmFyZW4gS3VtYXIrbmFyZW5pcHM2QGdtYWlsLmNvbStRVUVTVE


PTiA0MQ==
Which of the following leaders got the title – Rahbar - E - Azam?

a) Sir Syed Ahmad Khan


b) Sir Chhotu Ram
c) Baba Ramchander
d) Din Bandhu Mitra
Correct Answer: B
Your Answer: A
Explanation

Solution (b)

Prime Minister Narendra Modi unveiled a statue of Jat leader Sir Chhotu Ram (1881-1945) in
Haryana.

His reforms

IASbaba
Score:
Web: http://ilp.iasbaba.com/
47.00 /
Email: ilp@iasbaba.com
Page 32 200
2019 - Test 28-
Exam Title :
History & C...
Email : narenips6@gmail.com
Contact :

· As a member of the pre-Partition Punjab Legislative Council, his first major achievement was
the passage of the Punjab Land Revenue (Amendment) Act, 1929, which remains a landmark
social legislation till date.

· The exploitation of the peasantry by moneylenders was brought to an end with a series of
measures, starting with the Punjab Regulation of Accounts Act, 1930.

· It was followed by the Punjab Debtors Protection Act of 1936 and the Punjab Relief of
Indebtedness Act, 1943.

· The Punjab Agricultural Produce Markets Act was passed in 1939. Popularly called the Mandi
Act, it provided for the constitution of market committees in notified areas, and helped free the
farmer from exploitation

· A consolidation of land holdings was undertaken after passing the Consolidations Holding Act,
1936, amended in 1945. Not only were all these laws passed; Chhotu Ram also ensured their
implementation.

· A grateful peasantry rewarded him with the titles of Deen-bandhu and Rahbar-e-Azam. The
British honoured him with a knighthood in 1937.

QUESTION 43. MTU4Njk4K0d1bnR1cnUgTmFyZW4gS3VtYXIrbmFyZW5pcHM2QGdtYWlsLmNvbStRVUVTVE


PTiA0Mg==
Around 1000 petroglyphs have been discovered in and around Ratnagiri and Rajapur district in
Konkan region of Maharashtra. Which of the following statements are correct regarding
Petroglyphs?

1. Petroglyphs are ancient rock painting drawn on the rocks using natural colours.

2. Petroglyphs are rarely found in the world and only known specimens have been discovered in
India and Pakistan.

Select the code from following:

a) 1 only
b) 2 only
c) Both 1 and 2
d) Neither 1 nor 2
Correct Answer: D
Your Answer: Unanswered
Explanation

Solution (d)

Different sites with petroglyphs in Ratnagiri and Rajapur districts have been found over the last
two or three years.

Petroglyphs are images created by removing part of a rock surface by incising, picking, carving,
or abrading, as a form of rock art.

The term petroglyph should not be confused with petrograph, which is an image drawn or
painted on a rock face. Both types of image belong to the wider and more general category of

IASbaba
Score:
Web: http://ilp.iasbaba.com/
47.00 /
Email: ilp@iasbaba.com
Page 33 200
2019 - Test 28-
Exam Title :
History & C...
Email : narenips6@gmail.com
Contact :
rock art or parietal art. Petroforms, or patterns and shapes made by many large rocks and
boulders over the ground, are also quite different.

Petroglyphs are common and found in different parts across the world.

QUESTION 44. MTU4Njk4K0d1bnR1cnUgTmFyZW4gS3VtYXIrbmFyZW5pcHM2QGdtYWlsLmNvbStRVUVTVE


PTiA0Mw==
Which of the following statements are correct regarding Bathukamma Festival?

a) It is a floral festival celebrated predominantly by the Hindu women of Telangana.


b) It is a theatre form depicting the life of Lord Rama according to Ramayana.
c) It is a martial art festival celebrated by the tribals of western ghats in Kerala.
d) It is a month long festival celebrated in Andhra Pradesh to pay respects to goddess
Lakshmi.
Correct Answer: A
Your Answer: A
Explanation

Solution (a)

Bathukamma is floral festival celebrated predominantly by the Hindu women of Telangana and
some parts of Andhra Pradesh. Every year this festival is celebrated as per Shathavahana
calendar for nine days starting Bhadrapada Pournami till Durgashtami, usually in September–
October of Gregorian calendar.

QUESTION 45. MTU4Njk4K0d1bnR1cnUgTmFyZW4gS3VtYXIrbmFyZW5pcHM2QGdtYWlsLmNvbStRVUVTVE


PTiA0NA==
World’s largest Bird sculpture - ’Jatayu’, has been made in India. In which of the following
states is it located?

a) Tamil Nadu
b) Uttar Pradesh
c) Kerala
d) Odisha
Correct Answer: C
Your Answer: B
Explanation

Solution (c)

Kerala tourism will unveil the largest bird sculpture in Nature Park at Thiruvananthapuram.

The replica of the bird 'Jatayu' -- a character in the Ramayana -- is 200 feet long, 150 feet wide,
65 feet tall and is positioned right at the top of a rock situated 1,000 feet above sea level.

According to the legend the rock, 'Jatayupara', where the structure is based, is the place where
the mythical bird 'Jatayu' fell after its wings were slain by Ravana.

IASbaba
Score:
Web: http://ilp.iasbaba.com/
47.00 /
Email: ilp@iasbaba.com
Page 34 200
2019 - Test 28-
Exam Title :
History & C...
Email : narenips6@gmail.com
Contact :

QUESTION 46. MTU4Njk4K0d1bnR1cnUgTmFyZW4gS3VtYXIrbmFyZW5pcHM2QGdtYWlsLmNvbStRVUVTVE


PTiA0NQ==

25 th International Camel Festival was celebrated in India. Which of the following statements
are correct regarding it?

1. It is held every year for two days at Rann of Kutch.

2. It is organized by department of Tourism, art and culture of Rajasthan.

3. Folk artists from different regions come and perform here.

Select the code from following:

a) 1 only
b) 2 and 3
c) 1 and 3
d) All of the above
Correct Answer: B
Your Answer: D
Explanation

Solution (b)

International Camel Festival

It is a two day long festival in the honour of camel organized at Bikaner, Rajasthan.

It is organized by Department of Tourism, Art and Culture, Rajasthan.

Activities such as fur cutting, camel decoration, designing among other are held. Folk artists
from different regions come and perform in the festival.

QUESTION 47. MTU4Njk4K0d1bnR1cnUgTmFyZW4gS3VtYXIrbmFyZW5pcHM2QGdtYWlsLmNvbStRVUVTVE


PTiA0Ng==
Annual Classical festival, ‘Sur Festival’, held in Kolkata has been renamed as Sur Girija Festival
in tribute to Dr Girija Devi. Girija Devi was known as the Queen of

a) Dadra
b) Thumri
c) Ghazal
d) Bhajan
Correct Answer: B
Your Answer: B
Explanation

Solution (b)

Sur Festival

IASbaba
Score:
Web: http://ilp.iasbaba.com/
47.00 /
Email: ilp@iasbaba.com
Page 35 200
2019 - Test 28-
Exam Title :
History & C...
Email : narenips6@gmail.com
Contact :

· An annual classical music festival in the city will now be christened as 'Sur Girija Devi Festival'
in tribute to the late Thumri queen.

· The festival, to be held on February 4 is organised by Debapriya Adhikary and Samanwaya


Sarkar, disciples of Girija Devi.

· In the past three editions, it was called 'Sur Festival, from now on, it will be 'Sur Girija Devi
Festival'.

QUESTION 48. MTU4Njk4K0d1bnR1cnUgTmFyZW4gS3VtYXIrbmFyZW5pcHM2QGdtYWlsLmNvbStRVUVTVE


PTiA0Nw==
‘Kashmir Daily’ is the first ever Kashmiri feature film shot and produced entirely in Kashmir
valley. Which of the following statements is/are correct regarding the film?

1. It explores the themes of Militancy and atrocities of army in Kashmir.

2. The film will not see a theatrical release in Kashmir as there are no functioning cinema halls
in the Valley.

Select the code from following:

a) 1 only
b) 2 only
c) Both 1 and 2
d) Neither 1 nor 2
Correct Answer: B
Your Answer: Unanswered
Explanation

Solution (b)

For the first time in 45 years, a Kashmiri feature film, shot and produced entirely in the Valley,
will hit cinema halls outside Kashmir.

Kashmir Daily explores the themes of drug addiction and unemployment in Kashmir.

Ironically, the film will not see a theatrical release in Kashmir as there are no functioning
cinema halls in the Valley.

QUESTION 49. MTU4Njk4K0d1bnR1cnUgTmFyZW4gS3VtYXIrbmFyZW5pcHM2QGdtYWlsLmNvbStRVUVTVE


PTiA0OA==
In order to celebrate the art and culture of tribal communities ‘Aadi Mahotsava’ was celebrated.
In which of the following states was it celebrated?

a) Madhya Pradesh
b) Nagaland
c) Assam
d) Rajasthan

IASbaba
Score:
Web: http://ilp.iasbaba.com/
47.00 /
Email: ilp@iasbaba.com
Page 36 200
2019 - Test 28-
Exam Title :
History & C...
Email : narenips6@gmail.com
Contact :
Correct Answer: C
Your Answer: A
Explanation

Solution (c)

‘Aadi Mahotsava’ was held for 10 days in Guwahati, Assam to celebrated the art and culture of
the tribal communities.

QUESTION 50. MTU4Njk4K0d1bnR1cnUgTmFyZW4gS3VtYXIrbmFyZW5pcHM2QGdtYWlsLmNvbStRVUVTVE


PTiA0OQ==
Mahamastakabhiseka Festival is held in every 12 years. Which of the following statements
regarding Mahamastakabhisha are correct?

1. Mahamastakabhiseka of Gommateshwara, who is also known as Bahubali takes place once in


12 years.

2. It is held at Sharavanbelagola in Karnantak.

3. It is one of the biggest Buddhist festival held in the world.

Select the code from following:

a) 1 and 2
b) 2 and 3
c) 1 and 3
d) All of the above
Correct Answer: A
Your Answer: D
Explanation

Solution (a)

President Ram Nath Kovind will inaugurate the 88th Mahamastakabhisheka Utsav of Lord
Gommateshwara in Sharavanabelagola in Hassan district of Karnataka.

Mahamastakabhiseka of Gommateshwara, who is also known as Bahubali takes place once in 12


years.

It is an important Jain piligramage centre and lakhs of tourists visit Shravanbelagola to see 57
feet tall monolithic stone statue.

QUESTION 51. MTU4Njk4K0d1bnR1cnUgTmFyZW4gS3VtYXIrbmFyZW5pcHM2QGdtYWlsLmNvbStRVUVTVE


PTiA1MA==
"Longpi Hampai pottery" has started gaining prominence in the international markets owing to
its intricate designs and technology used to produce them. Which of the following statements
are correct regarding this?

1. The ancient art has originated from two Longpi villages in Manipur namely Longpi Khullen
and Longpi Kajui.

IASbaba
Score:
Web: http://ilp.iasbaba.com/
47.00 /
Email: ilp@iasbaba.com
Page 37 200
2019 - Test 28-
Exam Title :
History & C...
Email : narenips6@gmail.com
Contact :

2. It is practised by Tangkhul community residing in the hill district of Manipur.

Select the code from following:

a) 1 only
b) 2 only
c) Both 1 and 2
d) Neither 1 nor 2
Correct Answer: C
Your Answer: C
Explanation

Solution (c)

The Manipur-based indigenous Longpi Hampai pottery has started gaining prominence in the
international markets owing to its intricate designs and technology used to produce them.

The ancient art has originated from two Longpi villages in Manipur, namely Longpi Khullen and
Longpi Kajui near Ukhrul district. It is mainly practised by Tangkhul community residing in the
hill district of Manipur.

Longpi pottery is made from a mixed paste of ground black serpentine stone and special brown
clay, which is found only in Longpi village. The pots are manually shaped, polished and sun-
dried, potters collect the products and heat it in a bonfire which takes a total of six days to be
completed.

QUESTION 52. MTU4Njk4K0d1bnR1cnUgTmFyZW4gS3VtYXIrbmFyZW5pcHM2QGdtYWlsLmNvbStRVUVTVE


PTiA1MQ==
Consider the following statements regarding Indo – Islamic Architecture:

1. Mughals introduced Islamic architecture in India

2. Muslim architecture forbids replication of any living forms on any surface

IASbaba
Score:
Web: http://ilp.iasbaba.com/
47.00 /
Email: ilp@iasbaba.com
Page 38 200
2019 - Test 28-
Exam Title :
History & C...
Email : narenips6@gmail.com
Contact :

3. Qutub Minar has been constructed by Qutub-ud-din Aibak, Iltutmish and Firozshah Tughlaq

Which of the above statements are correct?

a) 1 and 2
b) 2 and 3
c) 1 and 3
d) All of the above
Correct Answer: B
Your Answer: A
Explanation

Solution (b)

Though Islam came to India over a period of 600 years through merchants, traders, holy men, it
was only in thirteenth century that large scale building activity was begun by Turkish state,
established after Turkish conquest of northern India.

Unlike Hindus who conceived manifestation of god everywhere in multiple forms as part of their
religious faith, Muslims forbade to replicate living forms on any surface.

In 1200 AD, Qutub al-Din Aibak, the founder of the Delhi Sultanate started construction of the
Qutub Minar. In 1220, Aibak's successor and son-in-law Iltutmish added three storeys to the
tower. In 1369, lightning struck the top storey, destroying it completely. So, Firoz Shah Tughlaq
carried out restoration work replacing the damaged storey with two new storeys every year.

QUESTION 53. MTU4Njk4K0d1bnR1cnUgTmFyZW4gS3VtYXIrbmFyZW5pcHM2QGdtYWlsLmNvbStRVUVTVE


PTiA1Mg==
Which of the following statements are correct regarding ‘Torans’?

1. They are entrance gates to a Stupa.

2. They are independent structures having two pillars and three horizontal beams connecting
the two.

3. They are intricately carved with life stories of Buddha.

4. Stupas have four Toranas, one in each direction.

Select the code from following:

a) 1,2 and 3
b) 2,3 and 4
c) 1,3 and 4
d) All of the above
Correct Answer: D
Your Answer: C
Explanation

Solution (d)

IASbaba
Score:
Web: http://ilp.iasbaba.com/
47.00 /
Email: ilp@iasbaba.com
Page 39 200
2019 - Test 28-
Exam Title :
History & C...
Email : narenips6@gmail.com
Contact :

Torana, Indian gateway, usually of stone, marking the entrance to a Buddhist shrine or stupa or
to a Hindu temple. Toranas typically consist of two pillars carrying two or three transverse
beams that extend beyond the pillars on either side. Strongly reminiscent of wooden
construction, toranas are often covered from top to bottom with exquisite sculpture. The four
toranas of the Great Stupa at Sanchi (see also Sanchi sculpture) are superb examples. 158698

QUESTION 54. MTU4Njk4K0d1bnR1cnUgTmFyZW4gS3VtYXIrbmFyZW5pcHM2QGdtYWlsLmNvbStRVUVTVE


PTiA1Mw==
Consider the following statements in regard to Mahavira and Jainism. Which of the statements
given below is not correct?

a) Mahavira, the 24th Tirthankara is considered to be the founder of Jainism


b) Mahavira was associated with Makari Gosala Putta for 6 years, but later departed due to
serious philosophical differences

IASbaba
Score:
Web: http://ilp.iasbaba.com/
47.00 /
Email: ilp@iasbaba.com
Page 40 200
2019 - Test 28-
Exam Title :
History & C...
Email : narenips6@gmail.com
Contact :

c) Then Mahavira joined Nigrantha sect, while Makari Gosala Putta started Ajivika religion.
d) After Mahavira, Jainism came under the control of 11 disciples of Mahavira, namely
Ganadharas.
Correct Answer: A
Your Answer: B
Explanation

Solution (a)

· Parsvanatha, the 23rd Tirthankara is considered to be the founder of Jainism.

· Except 1 st statement, remaining all statements are correct.

S. No. Tirthankara Associated Symbol

1 st Tirthankara Rishaba / Adinatha Bull

21 st Neminatha Blue Lotus

22 nd Aristanemi Conch

23 rd Parsvanatha Snake

24 th Mahavira Lion

QUESTION 55. MTU4Njk4K0d1bnR1cnUgTmFyZW4gS3VtYXIrbmFyZW5pcHM2QGdtYWlsLmNvbStRVUVTVE


PTiA1NA==
Consider the following statements:

1. During Ashoka’s regime (3 rd century BC), Sanskrit served as the lingua franca of the
country.

2. Pali acquired the same position and served as the state language, especially in the post-
Gupta period.

Which of the statements given above is/are correct?

a) 1 only
b) 2 only
c) Both 1 and 2
d) Neither 1 nor 2
Correct Answer: D

IASbaba
Score:
Web: http://ilp.iasbaba.com/
47.00 /
Email: ilp@iasbaba.com
Page 41 200
2019 - Test 28-
Exam Title :
History & C...
Email : narenips6@gmail.com
Contact :
Your Answer: Unanswered
Explanation

Solution (d)

· In the 3 rd century BC, Prakrit (not Sanskrit) served as the lingua franca of the country
. Throughout the major portion of India, Ashoka’s inscriptions were written in the Prakrit
language and Brahmi script. Bramhi continued to be the main script till the end of Gupta times.

· Later Sanskrit (not Pali) acquired the same position and served as the state language
in the remotest parts of the country. The process became prominent in the Gupta period in the
fourth century A.D.

· Although politically the country witnessed numerous small states in the post-Gupta period, the
official documents were written in Sanskrit.

QUESTION 56. MTU4Njk4K0d1bnR1cnUgTmFyZW4gS3VtYXIrbmFyZW5pcHM2QGdtYWlsLmNvbStRVUVTVE


PTiA1NQ==
The seal of Pashupati Mahadeva of Harappan culture (found in Mahenjo-daro) - a male deity
with three horned heads, in the sitting posture of a yogi, placing one foot on the other – is
surrounded by which of the following animals?

1. Buffalo

2. Elephant

3. Rhinoceros

4. Tiger

5. Deer

Select the appropriate code:

a) 1, 2 and 5
b) 1, 2, 4 and 5
c) 1, 3, 4 and 5
d) All of the above
Correct Answer: D
Your Answer: D
Explanation

Solution (d)

Explanation:

· Observe the figure/seal carefully and identify the animals

IASbaba
Score:
Web: http://ilp.iasbaba.com/
47.00 /
Email: ilp@iasbaba.com
Page 42 200
2019 - Test 28-
Exam Title :
History & C...
Email : narenips6@gmail.com
Contact :

QUESTION 57. MTU4Njk4K0d1bnR1cnUgTmFyZW4gS3VtYXIrbmFyZW5pcHM2QGdtYWlsLmNvbStRVUVTVE


PTiA1Ng==
Which among the following is/are the principles of Jain Panchasutras?

1. Ahimsa

2. Satya

3. Nyaya

IASbaba
Score:
Web: http://ilp.iasbaba.com/
47.00 /
Email: ilp@iasbaba.com
Page 43 200
2019 - Test 28-
Exam Title :
History & C...
Email : narenips6@gmail.com
Contact :

4. Dharma

5. Brahmacharya

Select the appropriate code:

a) All of the above


b) 1, 2 and 3 only
c) 1, 2 and 5 only
d) 1, 2, 4 and 5 only
Correct Answer: C
Your Answer: Unanswered
Explanation

Solution (c)

Jain Principles

Panchasutras : Basic principles of Jainism, namely Ahimsa, Satya , Aparigraha , Asteya ,


Brahmacharya.

Triratnas / Ratnatraya : Three Jewels of Jainism , namely

1. Right Action ( samyak kriya )

2. Right Knowledge ( samyak gnana)

3. Right Faith (samyak viswas)

QUESTION 58. MTU4Njk4K0d1bnR1cnUgTmFyZW4gS3VtYXIrbmFyZW5pcHM2QGdtYWlsLmNvbStRVUVTVE


PTiA1Nw==
Consider the literary works and select the incorrect pair from the given below:

Literary works written by

a) Harshacharita Banabhatta
b) Ramacharita Sandhyakara Nandi
c) Vikramanakadevacharita Kalhana
d) Mushika Vamsa Atula
Correct Answer: C
Your Answer: B
Explanation

Solution (c)

· Vikramanakadevacharita is written by Bilhana, which recounts the achievements of his patron,


Vikramadithya VI, the Chalukya king

· Rajatarangini or ‘The Stream of Kings’ is written by Kalhana. It is a string of biographies of


the kings of Kashmir.

IASbaba
Score:
Web: http://ilp.iasbaba.com/
47.00 /
Email: ilp@iasbaba.com
Page 44 200
2019 - Test 28-
Exam Title :
History & C...
Email : narenips6@gmail.com
Contact :

QUESTION 59. MTU4Njk4K0d1bnR1cnUgTmFyZW4gS3VtYXIrbmFyZW5pcHM2QGdtYWlsLmNvbStRVUVTVE


PTiA1OA==
Which among the following statements is/are true in regard to Rig Veda :

1. The Rig veda is the earliest text of the Indo-European languages. It is a collection of prayers
offered to Agni, Indra, Mitra, Varuna and other gods by various families of poets or sages.

2. The Rig veda has many things common with the Avesta , which is the oldest text in the
Iranian language

Which of the statements given above is/are correct?

a) 1 only
b) 2 only
c) Both 1 and 2
d) Neither 1 nor 2
Correct Answer: C
Your Answer: Unanswered
Explanation

Solution (c)

· Both the given statements are correct and self-explanatory.

· The Rig veda has many things common with the Avesta, which is the oldest text in the Iranian
language. The two texts use the same names for several gods and even for social classes.

QUESTION 60. MTU4Njk4K0d1bnR1cnUgTmFyZW4gS3VtYXIrbmFyZW5pcHM2QGdtYWlsLmNvbStRVUVTVE


PTiA1OQ==
The below given contains the PANCHAMAHAKALYANA/FIVE EVENTS IN LIFE OF BUDDHA.
Select the correctly matched pair/s from the Lists given below

List I (EVENT NAME) List II (ASSOCIATED SYMBOLS)

1. Mahabhinishkramana Bodhi/ peepal tree

2. Dharmachakraparivarthana Dharmachakra with 24 spokes

3. Mahaparinirvarna Stupa

4. Birth of buddha Lotus

Choose the appropriate code:

a) 1, 3 and 4 only
b) 3 and 4 only
c) 2 and 4 only
d) 4 only
Correct Answer: B

IASbaba
Score:
Web: http://ilp.iasbaba.com/
47.00 /
Email: ilp@iasbaba.com
Page 45 200
2019 - Test 28-
Exam Title :
History & C...
Email : narenips6@gmail.com
Contact :
Your Answer: C
Explanation

Solution (b)

PANCHAMAHAKALYANA/FIVE EVENTS IN LIFE OF BUDDHA :

QUESTION 61. MTU4Njk4K0d1bnR1cnUgTmFyZW4gS3VtYXIrbmFyZW5pcHM2QGdtYWlsLmNvbStRVUVTVE


PTiA2MA==
Choose the correct pair(s) from the below:

1. Study of coins :: Numismatics

2. Study of inscriptions : : Epigraphy

3. Study of old writings and old records in inscriptions : : Palaeography

Select the correct code:

a) All of the above


b) None of the above
c) 1 only
d) 2 and 3 only
Correct Answer: A
Your Answer: A
Explanation

IASbaba
Score:
Web: http://ilp.iasbaba.com/
47.00 /
Email: ilp@iasbaba.com
Page 46 200
2019 - Test 28-
Exam Title :
History & C...
Email : narenips6@gmail.com
Contact :

Solution (a)

· Factual question – no explanations needed

QUESTION 62. MTU4Njk4K0d1bnR1cnUgTmFyZW4gS3VtYXIrbmFyZW5pcHM2QGdtYWlsLmNvbStRVUVTVE


PTiA2MQ==
Consider the following similarities and difference between the Hindustani and the Carnatic
styles of classic music. Which of the following statements is incorrect regarding this?

a) Origin of Hindustani music is earlier than Carnatic music. It synthesizes with Islamic
traditions and Persian Musiqu-e-Assil style and has no association with religion.
b) Hindustani music originated in the Vedic period, while Carnatic music originated during the
Bhakti movement.
c) Main emphasis in Carnatic music is on vocal music and most compositions are to be sung,
involving gayaki style.
d) Although there are stylistic differences, the basic elements of swara, raga and tala as
foundation of both Carnatic and Hindustani are same.
Correct Answer: A
Your Answer: D
Explanation

Solution (a)

Explanation:

· As both Hindustani and the Carnatic styles of classic music has great association with religion.

Carnatic music is a system of music associated with the southern part of Indian sub
continent. Hindustani music is associated with North India and is deeply influenced by
Persian and Islamic music system.

Similarities

(i) Although there are stylistic differences, the basic elements of swara, raga and tala as
foundation of both Carnatic and Hindustani are same.

(ii) Hindustani music originated in the Vedic period, while Carnatic music originated during the
Bhakti movement. Thus both are having great association with religion.

(iii) Carnatic is one of two main sub-genres of India classical music that evolved from ancient
Hindu traditions, the other sub-genre being Hindustani music, which emerged as a distinct form
due to Persian and Islamic influences in North India.

(iv) Both the music evolved with Sanskrit language scripts in itself and through Vedic traditions.

(v) The central notions in both these system is that of a melodic mode or raga, sug to a rhythmic
cycle or tala.

Differences

(i) Origin of Hindustani music is earlier than Carnatic music. It synthesizes with Vedic chants,
Islamic traditions and Persian Musiqu-e-Assil style. Carnatic is Comparatively pure and was

IASbaba
Score:
Web: http://ilp.iasbaba.com/
47.00 /
Email: ilp@iasbaba.com
Page 47 200
2019 - Test 28-
Exam Title :
History & C...
Email : narenips6@gmail.com
Contact :
developed in 15th 16th century during Bhakti movement and also get boost in 19th -20th
century.

(ii) Main emphasis in Carnatic music is on vocal music and most compositions are to be sung,
involving gayaki style. In Hindustani music vocal- centric group is required. Many instruments
are designed to accompany the singer.

(iii) The major vocal forms of Hindustani music are Dhrupad, khayal, Tarana, Thumri, Dadra and
Gazals. While Carnatic music embraces several varieties of improvisation consist of Alpana,
Niraval, kalpnaswaram and Ragam Thana pallavi.

Hindustani Classical Music: Indian classical music found throughout North India. The style is
sometimes called North Indian classical music or Shāstriya Sangīt. It is a tradition that
originated in Vedic ritual chants and has been evolving since the 12th century CE, in North
India and to some extent in Nepal and Afghanistan.

Carnatic music (Karnataka Sangita): A system of music commonly associated with the
southern part of the Indian subcontinent, with its area roughly confined to four modern states
of India: Andhra Pradesh, Karnataka, Kerala, and Tamil Nadu. It is one of two main sub-genres
of Indian classical music that evolved from ancient Hindu traditions; the other sub-genre being
Hindustani music, which emerged as a distinct form because of Persian and Islamic influences
in North India.

QUESTION 63. MTU4Njk4K0d1bnR1cnUgTmFyZW4gS3VtYXIrbmFyZW5pcHM2QGdtYWlsLmNvbStRVUVTVE


PTiA2Mg==
Which of the following statements are correct about ‘PanchaSiddhanta’?

1. It was written by Aryabhatta.

2. It is a detailed account of Greek and Roman Astrology.

Select the correct code from the following:

a) 1 only
b) 2 only
c) Both 1 and 2
d) Neither 1 nor 2
Correct Answer: D
Your Answer: A
Explanation

Solution (d)

Panchasiddhanta was written by Vrahamihira. It is a treatise on mathematical astronomy.

QUESTION 64. MTU4Njk4K0d1bnR1cnUgTmFyZW4gS3VtYXIrbmFyZW5pcHM2QGdtYWlsLmNvbStRVUVTVE


PTiA2Mw==
Consider the following statements—

1. This dance form has been introduced by Mahapurusha Sankaradeva.

IASbaba
Score:
Web: http://ilp.iasbaba.com/
47.00 /
Email: ilp@iasbaba.com
Page 48 200
2019 - Test 28-
Exam Title :
History & C...
Email : narenips6@gmail.com
Contact :

2. The musical compositions accompanying the dance form are called borgeets.

3. The costumes used are made up of pat.

Identify the dance-form from the following options:

a) Mohiniyattam
b) Thang Ta
c) Sattriya
d) Kuchipudi
Correct Answer: C
Your Answer: C
Explanation

Solution (c)

Mahapurusha Sankaradeva: A great Vaishnava saint and reformer of Assam

Pat:

· A silk produced in Assam

· Derived from the mulberry plant

QUESTION 65. MTU4Njk4K0d1bnR1cnUgTmFyZW4gS3VtYXIrbmFyZW5pcHM2QGdtYWlsLmNvbStRVUVTVE


PTiA2NA==
Which among the following statements is not true in regard to Paitkar Paintings – an
endangered art form which is in decline and may soon become extinct?

a) Paitkar paintings are scroll paintings practiced by the tribal people of Assam.
b) The cultural heritage of this old form of painting has associations with one of the most well-
known goddess in the Bengali household, Ma Manasa.
c) The Paitkar paintings have links with the socio-religious custom of holding yajnas and
giving alms.
d) The paintings have a common subject – what happens to human life post death.
Correct Answer: A
Your Answer: Unanswered
Explanation

Solution (a)

· Paitkar paintings are scroll paintings practiced by the tribal people of Jharkhand state.

IASbaba
Score:
Web: http://ilp.iasbaba.com/
47.00 /
Email: ilp@iasbaba.com
Page 49 200
2019 - Test 28-
Exam Title :
History & C...
Email : narenips6@gmail.com
Contact :

Paitkar painting is one of the earliest forms of tribal paintings in India, and are also called scroll
paintings due to its appearance. These paintings are the pictures depicted in the way of
storytelling. Paitkar paintings of Amadubi (Jharkhand) comprise one of the most sought-after
paintings in the state and the country. Amadubi is also known as the village of Paitkar. This
mode of painting is a creative diction of the folk art of Jharkhand and the adjoining states. The
paintings that belong to this form have a common theme of what happens to human life after
death. While on the other hand, this type of painting also paints the Bengali and Jharkhandi
everyday life. The usual subjects of these paintings are-

· Flora and Fauna

· Legend and Folktale

· Traditional Hindu Epic

· Festivals and Fairs

QUESTION 66. MTU4Njk4K0d1bnR1cnUgTmFyZW4gS3VtYXIrbmFyZW5pcHM2QGdtYWlsLmNvbStRVUVTVE


PTiA2NQ==
Chikankari is

a) An intricate and fine shadow-work type of embroidery traditionally done with white yarn on
colourless muslins called tanzeb.
b) A carpet, akin to Persian carpets of Iranian origin, which is completely hand-made, hand-
knotted and are primarily made in pure wool, pure silk and occasionally wool and silk
blends.
c) Literally a ‘pen craft’ – is an art that involves hand block printing apart from painting on a
saree.

IASbaba
Score:
Web: http://ilp.iasbaba.com/
47.00 /
Email: ilp@iasbaba.com
Page 50 200
2019 - Test 28-
Exam Title :
History & C...
Email : narenips6@gmail.com
Contact :

d) A delicious food item of Ambur, Tamil Nadu, which recently got GI tag under Geographical
Indications of Goods (Registration and Protection Act 1999) Act.
Correct Answer: A
Your Answer: C
Explanation

Solution (a)

· Chikankari is an intricate and fine shadow-work type of embroidery traditionally done with
white yarn on colourless muslins called tanzeb.

QUESTION 67. MTU4Njk4K0d1bnR1cnUgTmFyZW4gS3VtYXIrbmFyZW5pcHM2QGdtYWlsLmNvbStRVUVTVE


PTiA2Ng==
Conisder the following statements in regard to Thangka art:

1. Thangka is one of the Manipur’s most ancient martial arts.


2. Thangka involves using a sword or sphere against one or more opponents.

IASbaba
Score:
Web: http://ilp.iasbaba.com/
47.00 /
Email: ilp@iasbaba.com
Page 51 200
2019 - Test 28-
Exam Title :
History & C...
Email : narenips6@gmail.com
Contact :

Which of the above statement(s) is/are correct?

a) 1 only
b) 2 only
c) Both
d) None
Correct Answer: D
Your Answer: C
Explanation

Solution (d)

· Thangkas are typical paintings of Sikkim (as well as Tibet), originally a medium of reverence
through which the highest ideals of Buddhism were evoked.

· Thangkas are created on cotton canvas and often framed with silk. They depict the images of
different deities and philosophies related to Buddhism.

· Thangkas were traditionally made by Buddhist priests and monks and specific ethnic groups,
the skills being passed from generation to generation. (Now Thangkas are an endangered art
form which is on decline)

· The colours used in making a thangka are all vegetable or mineral dyes extracted from nature.

NOTE: Thang-ta is one of the Manipur’s most ancient martial arts. Thang-ta involves using a
sword or sphere against one or more opponents.

QUESTION 68. MTU4Njk4K0d1bnR1cnUgTmFyZW4gS3VtYXIrbmFyZW5pcHM2QGdtYWlsLmNvbStRVUVTVE


PTiA2Nw==
With regard to Tuti-nama consider the following statements.

1. It is an illustrated compilation of stories in miniature paintings.

2. The work was commissioned by Akbar.

3. The themes and stories are derived from the 12th century Sanskrit anthology titled
Śukasaptati.

Which of the above statements is/are correct?

a) 1 and 2 only
b) 2 only
c) 1, 2 and 3
d) 1 only
Correct Answer: C
Your Answer: C
Explanation

Solution (c)

Tuti-nama is considered to be the first work of the Mughal School. Tuti-Nama literally
means the “Tales of a Parrot”. It is an illustrated compilation of 52 stories in 250 miniature

IASbaba
Score:
Web: http://ilp.iasbaba.com/
47.00 /
Email: ilp@iasbaba.com
Page 52 200
2019 - Test 28-
Exam Title :
History & C...
Email : narenips6@gmail.com
Contact :
paintings . The work was commissioned by Akbar. The themes and stories are derived from
the 12th century Sanskrit anthology titled Śukasaptati or “Seventy Tales of Parrot”. Th
e parrot tells the 52 stories in the consecutive 52 nights and in these stories he teaches some
moral stories to his owner. The work was completed in a span of five years under Mir Sayyid
Ali and Abdus Samad . The text was written by Nakhshabi, an ethnic Persian physician and a
Sufi saint who had migrated to Badayun. It was wrote in Persian.

Do you know?

· Hamza-nama, which contains the illustrations on cloth, originally consisting of 1400 leaves in
seventeen volumes. Each leaf measured about 27″x20″. These paintings were based upon a
Persian Hamzanama or Dastan-e-Amir Hamza.

QUESTION 69. MTU4Njk4K0d1bnR1cnUgTmFyZW4gS3VtYXIrbmFyZW5pcHM2QGdtYWlsLmNvbStRVUVTVE


PTiA2OA==
Which of the following statements are correct regarding the Ratha Temples of Mahabalipuram?

1. They are rock cut temples made by Pallava rulers.

2. There are five rathas, one each for five pandavas.

3. It is part of the UNESCO World Heritage site inscribed by UNESCO as Group of Monuments
at Mahabalipuram.

Select the code from following:

a) 1 and 2
b) 2 and 3
c) 1 and 3
d) All of the above
Correct Answer: C
Your Answer: C
Explanation

Solution (c)

Pancha Rathas

Pancha Rathas (also known as Five Rathas or Pandava Rathas) is a monument complex at
Mahabalipuram, on the Coromandel Coast of the Bay of Bengal, in the Kancheepuram district of
the state of Tamil Nadu, India. Pancha Rathas is an example of monolithic Indian rock-cut
architecture. Dating from the late 7th century, it is attributed to the reign of King
Mahendravarman I and his son Narasimhavarman I (630–680 AD; also called Mamalla, or "great
warrior") of the Pallava Kingdom. An innovation of Narasimhavarman, the structures are
without any precedent in Indian architecture. The complex is under the auspices of the
Archaeological Survey of India (ASI) and is part of the UNESCO World Heritage site inscribed
by UNESCO as Group of Monuments at Mahabalipuram.

Each of the five monuments in the Pancha Rathas complex resembles a chariot (ratha), and
each is carved over a single, long stone or monolith, of granite which slopes in north-south
direction with a slight incline.Though sometimes mistakenly referred to as temples, the
structures were never consecrated because they were never completed following the death of

IASbaba
Score:
Web: http://ilp.iasbaba.com/
47.00 /
Email: ilp@iasbaba.com
Page 53 200
2019 - Test 28-
Exam Title :
History & C...
Email : narenips6@gmail.com
Contact :
Narasimhavarman I. The structures are named after the Pancha Pandavas and their common
wife Draupadi, of epic Mahabharata fame. In order of their size, they include the Dharmaraja
Ratha, Bhima Ratha, Arjuna Ratha, Nakula Sahadeva Ratha, and Draupadi Ratha.

Note: Second statement is wrong because there are four Rathas for 5 pandavas and one Ratha
for Draupadi.

QUESTION 70. MTU4Njk4K0d1bnR1cnUgTmFyZW4gS3VtYXIrbmFyZW5pcHM2QGdtYWlsLmNvbStRVUVTVE


PTiA2OQ==
Chachnama was written by Ali Kufi. It deals with history of

a) Delhi Sultanate
b) Sind
c) Kashmir
d) Bahmani Kingdom
Correct Answer: B
Your Answer: D
Explanation

Solution (b)

Chach Nama

The Chach Nama is one of the main historical sources for the history of Sindh in the seventh to
eighth centuries CE, written in Persian.

The text, with the stories of early 8th-century conquests of Muhammad bin Qasim, has been
long considered to be a 13th-century translation into Persian by `Ali Kufi of an undated, original
but unavailable Arabic text.

Note: King Dahir of sind, who was defeated by Mohammad bin Qasim, belonged to Chach
Dynasty.

QUESTION 71. MTU4Njk4K0d1bnR1cnUgTmFyZW4gS3VtYXIrbmFyZW5pcHM2QGdtYWlsLmNvbStRVUVTVE


PTiA3MA==
Consider the following pairs about Sufi philosophy:

Philosophy Meaning

1. Waḥdat al-wujūd Unity of Existence

2. Waḥdat ash-shuhūd Unity of appearance

3. Al-Wujūd Al-Munbasiṭ Self-unfolding Being

Which of the above pairs is/are correctly matched?

a) 1 only

IASbaba
Score:
Web: http://ilp.iasbaba.com/
47.00 /
Email: ilp@iasbaba.com
Page 54 200
2019 - Test 28-
Exam Title :
History & C...
Email : narenips6@gmail.com
Contact :

b) 1 and 2 only
c) 1, 2 and 3
d) 3 only
Correct Answer: C
Your Answer: Unanswered
Explanation

Solution (c)

Major ideas in Sufi metaphysics have surrounded the concept of weḥdah meaning "unity", or in
Arabic tawhid. Two main Sufi philosophies prevail on this topic. waḥdat al-wujūd literally
means the "Unity of Existence" or "Unity of Being" but better translation would be
Monotheism of Existence. Wujud (i.e. existence) here refers to Allah's Wujud - implication is
Wahdat/Tawheed Of Wujud Of Allah. On the other hand, waḥdat ash-shuhūd, meaning
"Apparentism" or "Monotheism of Witness", holds that God and his creation are
entirely separate.

Al-Wujūd Al-Munbasiṭ (Self-unfolding Being)

Shah Waliullah Dehlawi tried to reconcile the two (apparently) contradictory doctrines of
waḥdat al-wujūd (unity of being) of Ibn Arabi and waḥdat ash-shuhūd (unity in conscience) of
Shaykh Ahmad Sirhindi. Shah Waliullah neatly resolved the conflict, calling these differences
'verbal controversies' which have come about because of ambiguous language. If we leave, he
says, all the metaphors and similes used for the expression of ideas aside, the apparently
opposite views of the two metaphysicians will agree.

Do you know?

· While orthodox Muslims emphasise external conduct, the Sufis lay stress on inner purity.

· While the orthodox believe in blind observance of rituals, the Sufis consider love and devotion
as the only means of attaining salvation.

· According to Sufis one must have the guidance of a pir or guru, without which spiritual
development is impossible.

· Sufism also inculcated a spirit of tolerance among its followers.

QUESTION 72. MTU4Njk4K0d1bnR1cnUgTmFyZW4gS3VtYXIrbmFyZW5pcHM2QGdtYWlsLmNvbStRVUVTVE


PTiA3MQ==
Consider the following statements about Gol Gumbaz:

1. It is built of Marble and decorated plaster work.

2. The dome is famous for whispering gallery.

Which of the above statements is/are correct?

a) 1 only
b) 2 only
c) Both 1 and 2
d) Neither 1 nor 2

IASbaba
Score:
Web: http://ilp.iasbaba.com/
47.00 /
Email: ilp@iasbaba.com
Page 55 200
2019 - Test 28-
Exam Title :
History & C...
Email : narenips6@gmail.com
Contact :
Correct Answer: B
Your Answer: Unanswered
Explanation

Solution (b)

Gumbad is a square building topped with a circular drum over which rests a majestic dome,
giving the building its nomenclature.

It is built of dark grey basalt and decorated plaster work.

The building has an amazing acoustical system. Along with the drum of the dome, there is a whi
spering gallery where sounds get magnified and echoed many times over.

Gol Gumbad is a fine convergence of many styles located in medieval India.

Do you know?

The Hindola Mahal, Mandu

· It looks like a railway viaduct bridge with its disproportionately large buttresses supporting
the walls.

· This was the audience hall of the Sultan and the place where he showed himself to his
subjects.

· Batter was used very effectively to give an impression of swinging (Hindola) walls.

QUESTION 73. MTU4Njk4K0d1bnR1cnUgTmFyZW4gS3VtYXIrbmFyZW5pcHM2QGdtYWlsLmNvbStRVUVTVE


PTiA3Mg==
Consider the following statements

1. Chahar bagh pattern was used in Humayun’s tomb.

2. Terraced chahar bagh pattern is used at Lal mahal Bari.

3. River front chahar bagh pattern is used at Shalimar garden.

Select the correct statements

a) Only 1
b) 1 and 2 only
c) 1 and 3 only
d) All of the above
Correct Answer: A
Your Answer: Unanswered
Explanation

Solution (a)

· Babur was interested in planning and laying out formal gardens, placed within rectangular
walled enclosures and divided into four quarters by artificial channels.

IASbaba
Score:
Web: http://ilp.iasbaba.com/
47.00 /
Email: ilp@iasbaba.com
Page 56 200
2019 - Test 28-
Exam Title :
History & C...
Email : narenips6@gmail.com
Contact :

· These gardens were called chahar bagh, four gardens, because of their symmetrical
division into quarters. Beginning with Akbar, some of the most beautiful chahar baghs were
constructed by Jahangir and Shah Jahan in Kashmir, Agra and Delhi.

QUESTION 74. MTU4Njk4K0d1bnR1cnUgTmFyZW4gS3VtYXIrbmFyZW5pcHM2QGdtYWlsLmNvbStRVUVTVE


PTiA3Mw==
Which of the following are correctly matched?

1. Akbarnama – Abul Fazl

2. Kitab ul Hind – Al Biruni

IASbaba
Score:
Web: http://ilp.iasbaba.com/
47.00 /
Email: ilp@iasbaba.com
Page 57 200
2019 - Test 28-
Exam Title :
History & C...
Email : narenips6@gmail.com
Contact :

3. Tabaqat I Nasiri – Minhaj Siraj

Select the code from following:

a) 1 and 2
b) 2 and 3
c) 1 and 3
d) All of the above
Correct Answer: D
Your Answer: D
Explanation

Solution (d)

Abul Fazl

· Abul Fazl was the vizier of the Mughal emperor Akbar, and author of the Akbarnama , the
official history of Akbar's reign in 3 volumes, (the 3 rd volume is known as the Ain-i-Akbari )
and a Persian translation of the Bible.

· He was also one of the Nine Jewels (Hindi: Navaratnas) of Akbar's royal.

Al Biruni

· Born in 973, in Khwarizm in present day Uzbekistan.

· In 1017, when Sultan Mahmud invaded Khwarizm, he took several scholars and poets back to
his capital, Ghazni; Al-Biruni was one of them.

· Al-Biruni spent years in the company of Brahmana priests and scholars, learning Sanskrit, and
studying religious and philosophical texts.

· He wrote “The Kitab-ul- Hind”.

o It was written in Arabic

o It is a voluminous text, divided into 80 chapters on subjects such as religion and philosophy,
festivals, astronomy, alchemy, manners and customs, social life, weights and measures,
iconography, laws and metrology of India.

Minhaj-i-Siraj

· A 13th-century Persian historian born in the Ghurid capital city of Firuzkuh.

· He was contemporary of Shamsuddin Iltutmish.

· He wrote 'Tabaqat-i-Nasiri'.

QUESTION 75. MTU4Njk4K0d1bnR1cnUgTmFyZW4gS3VtYXIrbmFyZW5pcHM2QGdtYWlsLmNvbStRVUVTVE


PTiA3NA==
Who invented sarangi and rabab?

IASbaba
Score:
Web: http://ilp.iasbaba.com/
47.00 /
Email: ilp@iasbaba.com
Page 58 200
2019 - Test 28-
Exam Title :
History & C...
Email : narenips6@gmail.com
Contact :

a) Amir Khusrau
b) Pir Bhodan
c) Abul Fazl
d) Abul Faizi
Correct Answer: A
Your Answer: A
Explanation

Solution (a)

Amir Khusru

· The famous Persian writer of this period. He wrote a number of poems.

· He experimented with several poetical forms and created a new style of Persian poetry called ‘
Sabaq-i-Hind or the Indian style ’.

· He also wrote some Hindi verses.

· Amir Khusrau was author of Khazain-ul-Futuh and Tughlaq Nama .

· He was Sufi musician, poet and scholar.

· Alauddin Khalji patronized poets like Amir Khusrau and Amir Hasan.

· Amir Khusru is considered as the father of Urdu language and also known as the ‘Parrot of
India’.

· Amizr Khusru was the inventor of Sarangi and the inventor of Sitar.

· Laila Majnu and Tughlaq Nama are the famous works of Amir Khusru.

· Amir Khusrau introduced many new ragas such as Ghora and Sanam.

· He evolved a new style of light music known as “Qwwali” by blending the Hindu and Iranian
systems.

· The invention of Sitar was also attributed to him.

Pir Bhodan or Budan

· He was also called as Baba Budhan Ali Shah

· He was a venerated Sufi pir who had religious discourse with Guru Nanak in Rawalpindi and
later accepted Gurmat thought during times of Guru Har Gobind.

· He was one of the great musicians of Delhi sultanate period.

Abul Fazl and Abul Faizi

· Shaikh Mubarak, was a scholar in the philosophy and literature of Greece as well as in Islamic
theology. He was associated with Akbar in the religious discussions.

· Abul fazal and Abul Faizi were son of Shaikh Mubarak.

IASbaba
Score:
Web: http://ilp.iasbaba.com/
47.00 /
Email: ilp@iasbaba.com
Page 59 200
2019 - Test 28-
Exam Title :
History & C...
Email : narenips6@gmail.com
Contact :

· Abul Faizi was the elder brother of Akbar's historian Abul Fazl. Akbar highly recognised the
genius in Faizi and appointed him tutor for his sons and gave place to him among his decorative
'Navaratnas’. The translation of Mahabharata into the Persian language was done under his
supervision.

· Abul Fazl was the vizier of the Mughal emperor Akbar, and author of the Akbarnama , the
official history of Akbar's reign in 3 volumes, (the 3 rd volume is known as the Ain-i-Akbari )
and a Persian translation of the Bible. He was his friend and autobiographer, helped Akbar in
framing a vision of governance around idea of sulh-i kul.

· Abul Fazl set a style of prose writing and it was followed by many generations.

· He was also one of the Nine Jewels (Hindi: Navaratnas) of Akbar's royal.

QUESTION 76. MTU4Njk4K0d1bnR1cnUgTmFyZW4gS3VtYXIrbmFyZW5pcHM2QGdtYWlsLmNvbStRVUVTVE


PTiA3NQ==
‘Shiroka Polyana’ was recently in news. Where is it located?

a) Russia
b) Greece
c) Bulgaria
d) Ukraine
Correct Answer: C
Your Answer: D
Explanation

Solution (c)

XVI World Ice Fishing Championship was held at the Shiroka Polyana.

It is a reservoir lake situated in the Western Rhodopes mountains in Bulgaria.

QUESTION 77. MTU4Njk4K0d1bnR1cnUgTmFyZW4gS3VtYXIrbmFyZW5pcHM2QGdtYWlsLmNvbStRVUVTVE


PTiA3Ng==
Consider the following statements with respect to ‘H5N1’

1. It does not affect humans

2. It is a subtype of the influenza A virus

3. It is sensitive to heat

Select the correct code:

a) 1 and 2
b) 2 and 3
c) 1, 2 and 3
d) 2 Only

IASbaba
Score:
Web: http://ilp.iasbaba.com/
47.00 /
Email: ilp@iasbaba.com
Page 60 200
2019 - Test 28-
Exam Title :
History & C...
Email : narenips6@gmail.com
Contact :
Correct Answer: B
Your Answer: Unanswered
Explanation

Solution (b)

Influenza A virus subtype H5N1, also known as A(H5N1) or simply H5N1, is a subtype of the
influenza A virus which can cause illness in humans and many other animal species.

H5N1 is a type of influenza virus that causes a highly infectious, severe respiratory disease in
birds called avian influenza (or "bird flu"). Human cases of H5N1 avian influenza occur
occasionally, but it is difficult to transmit the infection from person to person. When people do
become infected, the mortality rate is about 60%.

The virus is sensitive to heat. Normal temperatures used for cooking (so that food reaches 70°C
in all parts) will kill the virus. As a standard precaution, WHO recommends that poultry, poultry
products and wild game birds should always be prepared following good hygienic practices, and
that poultry meat should be properly cooked.

To date, a large number of human infections with the H5N1 virus have been linked to the home
slaughter and subsequent handling of diseased or dead birds prior to cooking. These practices
represent the highest risk of human infection and are the most important to avoid.

QUESTION 78. MTU4Njk4K0d1bnR1cnUgTmFyZW4gS3VtYXIrbmFyZW5pcHM2QGdtYWlsLmNvbStRVUVTVE


PTiA3Nw==
Consider the following statements with respect to ‘Programme for International Student
Assessment (PISA)’

1. It was introduced by the Organisation for Economic Cooperation Development (OECD) in


2012

2. It tests the learning levels of 15-year-olds in reading, mathematics and science

3. It is conducted every three years

Select the correct code:

a) 1 and 2
b) 2 and 3
c) 1 Only
d) 1, 2 and 3
Correct Answer: B
Your Answer: Unanswered
Explanation

Solution (b)

CONFIRMING INDIA’s participation in the Programme for International Student Assessment


(PISA) in 2021, the HRD Ministry signed an agreement with the Organisation for Economic
Cooperation Development (OECD).

PISA — introduced in 2000 by OECD — tests the learning levels of 15-year-olds in reading,
mathematics and science. The test is conducted every three years. India stayed away from PISA

IASbaba
Score:
Web: http://ilp.iasbaba.com/
47.00 /
Email: ilp@iasbaba.com
Page 61 200
2019 - Test 28-
Exam Title :
History & C...
Email : narenips6@gmail.com
Contact :
in 2012 and 2015 on account of its dismal performance in 2009, when it was placed 72nd
among the 74 participating countries. The then UPA government had blamed “out of context”
questions for the poor show in 2009. The country, subsequently, chose to not participate in the
2012 and 2015 cycle.

QUESTION 79. MTU4Njk4K0d1bnR1cnUgTmFyZW4gS3VtYXIrbmFyZW5pcHM2QGdtYWlsLmNvbStRVUVTVE


PTiA3OA==
The term ‘Virakta’ is associated with which of the following?

a) Jainism
b) Buddhism
c) Lingayatism
d) Vaishnavism
Correct Answer: C
Your Answer: C
Explanation

Solution (c)

Read More - https://en.wikipedia.org/wiki/Lingayatism

QUESTION 80. MTU4Njk4K0d1bnR1cnUgTmFyZW4gS3VtYXIrbmFyZW5pcHM2QGdtYWlsLmNvbStRVUVTVE


PTiA3OQ==
Consider the following statements with respect to ‘NGARM’

1. It is indigenously developed by the DRDO

2. It is designed to destroy a variety of surveillance and radar

Select the correct statements

a) 1 Only
b) 2 Only
c) Both 1 and 2
d) Neither 1 nor 2
Correct Answer: C
Your Answer: Unanswered
Explanation

Solution (c)

The missile is designed to destroy a variety of surveillance and radar targets on the ground
after being fired from a Sukhoi-30MKI fighter

This new-generation anti-radiation missile (NGARM), with a strike range of around 100-km, is
the first indigenous air-to-ground missile to be developed by the DRDO, after the supersonic
BrahMos cruise missile + developed jointly with Russia.

IASbaba
Score:
Web: http://ilp.iasbaba.com/
47.00 /
Email: ilp@iasbaba.com
Page 62 200
2019 - Test 28-
Exam Title :
History & C...
Email : narenips6@gmail.com
Contact :

Source: https://timesofindia.indiatimes.com/india/india-tests-new-anti-radiation-
missile-to-destroy-enemy-radars/articleshow/67676064.cms

QUESTION 81. MTU4Njk4K0d1bnR1cnUgTmFyZW4gS3VtYXIrbmFyZW5pcHM2QGdtYWlsLmNvbStRVUVTVE


PTiA4MA==
Which of the following is/are correctly matched?

Ports Country

1. Gwadar - Pakistan

2. Hambantota - Maldives

3. Kyaukpyu – Myanmar

Select the correct code:

a) 1 and 2
b) 1 and 3
c) 1, 2 and 3
d) 1 Only
Correct Answer: B
Your Answer: B
Explanation

Solution (b)

Gwadar - Pakistan

Hambantota – Sri Lanka

Kyaukpyu – Myanmar

QUESTION 82. MTU4Njk4K0d1bnR1cnUgTmFyZW4gS3VtYXIrbmFyZW5pcHM2QGdtYWlsLmNvbStRVUVTVE


PTiA4MQ==
Consider the following statements with respect to ‘Great Indian bustard’

1. It is only found in India

2. It is the state bird of Maharashtra

3. It is listed as ‘Critically Endangered’ by the IUCN

Select the correct statements:

a) 1 and 2
b) 2 and 3
c) 3 Only
d) None of the above

IASbaba
Score:
Web: http://ilp.iasbaba.com/
47.00 /
Email: ilp@iasbaba.com
Page 63 200
2019 - Test 28-
Exam Title :
History & C...
Email : narenips6@gmail.com
Contact :
Correct Answer: C
Your Answer: C
Explanation

Solution (c)

The great Indian Bustard (Ardeotis nigriceps) or Indian bustard is a bustard found on the Indian
subcontinent. It is a state bird of Rajasthan.

IUCN Status – CR

QUESTION 83. MTU4Njk4K0d1bnR1cnUgTmFyZW4gS3VtYXIrbmFyZW5pcHM2QGdtYWlsLmNvbStRVUVTVE


PTiA4Mg==
Which of the following National Parks is unique in being a swamp with floating vegetation that
supports a rich biodiversity?

a) Bhitarkanika National Park


b) Keibul Lamjao National Park
c) Keoladeo Ghana National Park
d) Sultanpur National Park
Correct Answer: B
Your Answer: B
Explanation

Solution (b)

The Keibul Lamjao National Park is a national park in the Bishnupur district of the state of
Manipur in India. It is 40 sq. km in area, the only floating park in the world, located in North
East India, and an integral part of Loktak Lake.

QUESTION 84. MTU4Njk4K0d1bnR1cnUgTmFyZW4gS3VtYXIrbmFyZW5pcHM2QGdtYWlsLmNvbStRVUVTVE


PTiA4Mw==
The ‘Chin’ people are primarily from

a) China
b) Myanmar
c) Bangladesh
d) Bhutan
Correct Answer: B
Your Answer: B
Explanation

Solution (b)

The Chin people are one of the major ethnic nationalities in Burma. The Chin are one of the
founding groups (Chin, Kachin, Shan and Myanmar) of the Union of Burma. Chin is the primary
ethnic group of the Chin State, who have many related languages, cultures and traditions. The
largest ethnic group of the Chin people are the Zomi. These people predominantly live in the
Chin State, Rakhine State and Sagaing Region of Myanmar, but are also spread throughout

IASbaba
Score:
Web: http://ilp.iasbaba.com/
47.00 /
Email: ilp@iasbaba.com
Page 64 200
2019 - Test 28-
Exam Title :
History & C...
Email : narenips6@gmail.com
Contact :
Burma, Bangladesh and India as refugee. In the 2014 Burmese ethnic census, the Chin ethnicity
was again dismissed by the people of the Chin State.

QUESTION 85. MTU4Njk4K0d1bnR1cnUgTmFyZW4gS3VtYXIrbmFyZW5pcHM2QGdtYWlsLmNvbStRVUVTVE


PTiA4NA==
Consider the following statements with respect to ‘All India Rural Financial Inclusion Survey
2016-17’

1. The coverage of NAFIS spans across various financial inclusion aspects ranging from loans,
savings, investments, pension, remittances and insurance.

2. It was conducted by National Bank for Agriculture and Rural Development

3. Over 50% of agricultural households were found to have been covered under insurance

Select the correct statements

a) 1 and 2
b) 2 and 3
c) 1 and 3
d) All of the above
Correct Answer: A
Your Answer: Unanswered
Explanation

Solution (a)

NAFIS was launched in 2016-17 as a national level survey that offers a comprehensive overview
of the rural population in terms of their status of livelihoods and level of financial inclusion.

The coverage of NAFIS spans across various financial inclusion aspects ranging from loans,
savings, investments, pension, remittances and insurance. The survey also involved assessing
the financial knowledge, attitude & behaviour of individuals and captured their experience with
the financial products & services that they utilized.

Incidence of Indebtedness (IOI), which is a proportion of households having outstanding debt


on the date of the survey, was 52.5 per cent and 42.8 per cent for agricultural and non-
agricultural households

All India IOI taking rural households together stood at 47.4 per cent

While 88.1 per cent rural households and 55 per cent agricultural households reported having a
bank account, average savings per annum per household was Rs 17,488

About 26 per cent of agricultural households and 25 per cent of non-agricultural households
were found to have been covered under insurance, it said.

Similarly, 20.1 per cent agricultural households as against 18.9 per cent non-agricultural
household shave subscribed to pension schemes.

Read More - https://www.nabard.org/auth/writereaddata/tender/1608180417NABARD-


Repo-16_Web_P.pdf

IASbaba
Score:
Web: http://ilp.iasbaba.com/
47.00 /
Email: ilp@iasbaba.com
Page 65 200
2019 - Test 28-
Exam Title :
History & C...
Email : narenips6@gmail.com
Contact :

Source: https://economictimes.indiatimes.com/news/economy/agriculture/farm-loan-
waiver-how-to-nip-it-in-the-bud/articleshow/67410788.cms

QUESTION 86. MTU4Njk4K0d1bnR1cnUgTmFyZW4gS3VtYXIrbmFyZW5pcHM2QGdtYWlsLmNvbStRVUVTVE


PTiA4NQ==
‘Rythu Bandhu Scheme of Telangana’ and ‘KALIA scheme of Odisha’ is often seen in news. What
is it related to?

a) Free Solar Pumps for Farmers


b) Free Electricity for Farmers
c) Subsidised Fertilizers and Pesticides for Farmers
d) Income/investment support scheme for Farmers
Correct Answer: D
Your Answer: D
Explanation

Solution (d)

Rythu Bandhu scheme also Farmers’ Investment Support Scheme (FISS) is a welfare program
to support farmer’s investment for two crops a year by the Government of Telangana.[1] The
government is providing 58.33 lakh farmers, ₹4000 per acre per season to support the farm
investment, twice a year, for rabi and kharif seasons. This is a first direct farmer investment
support scheme in India, where the cash is paid directly.

Krushak Assistance for Livelihood and Income Augmentation, KALIA

All farmers will be provided Rs 10,000 per family as assistance for cultivation. Each family will
get Rs 5,000 separately in the kharif and rabi seasons, for five cropping seasons between
2018-19 and 2021-22.

Read More - https://indianexpress.com/article/explained/kalia-how-odisha-new-scheme-


supports-farm-community-with-payments-5540259/

https://en.wikipedia.org/wiki/Rythu_Bandhu_scheme

QUESTION 87. MTU4Njk4K0d1bnR1cnUgTmFyZW4gS3VtYXIrbmFyZW5pcHM2QGdtYWlsLmNvbStRVUVTVE


PTiA4Ng==
‘Global Talent Competitiveness Index (GTCI)’ is released by

a) INSEAD
b) World Economic Forum
c) UNDP
d) IMF
Correct Answer: A
Your Answer: A
Explanation

Solution (a)

IASbaba
Score:
Web: http://ilp.iasbaba.com/
47.00 /
Email: ilp@iasbaba.com
Page 66 200
2019 - Test 28-
Exam Title :
History & C...
Email : narenips6@gmail.com
Contact :

Launched for the first time in 2013, the Global Talent Competitiveness Index (GTCI) is an
annual benchmarking report that measures the ability of countries to compete for talent.

India ranks 80th on Global Talent Competitive Index 2019.

QUESTION 88. MTU4Njk4K0d1bnR1cnUgTmFyZW4gS3VtYXIrbmFyZW5pcHM2QGdtYWlsLmNvbStRVUVTVE


PTiA4Nw==
Which of the following statements explains the term ‘Tax Evasion’ most appropriately?

a) It is the artful and knowledgeable filing and reporting of income and write-offs
b) It is a deliberate attempt to avoid paying a true tax liability
c) It is the act of withholding assets for the purpose of conversion (theft) of such assets, by
one or more persons to whom the assets were entrusted, either to be held or to be used for
specific purposes.
d) It is the process of concealing the origins of money obtained illegally by passing it through
a complex sequence of banking transfers or commercial transactions.
Correct Answer: B
Your Answer: A
Explanation

Solution (b)

Tax evasion is an illegal action in which a person or entity deliberately avoids paying a true tax
liability.

QUESTION 89. MTU4Njk4K0d1bnR1cnUgTmFyZW4gS3VtYXIrbmFyZW5pcHM2QGdtYWlsLmNvbStRVUVTVE


PTiA4OA==
‘Beveridge curve’ is associated with

a) Income distribution
b) Unemployment
c) Tax to GDP Ratio
d) Monetary Policy
Correct Answer: B
Your Answer: D
Explanation

Solution (b)

Beveridge curve

· This refers to a graphical representation that shows the relationship between the
unemployment rate (on the horizontal axis) and the job vacancy rate (on the vertical axis) in an
economy.

· It is named after British economist William Beveridge.

IASbaba
Score:
Web: http://ilp.iasbaba.com/
47.00 /
Email: ilp@iasbaba.com
Page 67 200
2019 - Test 28-
Exam Title :
History & C...
Email : narenips6@gmail.com
Contact :

· The Beveridge curve usually slopes downwards because times when there is high job vacancy
in an economy are also marked by relatively low unemployment since companies may actually
be actively looking to hire new people.

· By the same logic, a low job vacancy rate usually corresponds with high unemployment as
companies may not be looking to hire many people in new jobs.

QUESTION 90. MTU4Njk4K0d1bnR1cnUgTmFyZW4gS3VtYXIrbmFyZW5pcHM2QGdtYWlsLmNvbStRVUVTVE


PTiA4OQ==
Which of the following are possible reasons for ‘GST revenues going off target’?

1. Reducing number of items in the 28% slab

2. Businesses generating fake invoices to claim higher input tax credits

Select the correct code:

a) 1 Only
b) 2 Only
c) Both 1 and 2
d) Neither 1 nor 2
Correct Answer: C
Your Answer: A
Explanation

Solution (c)

GST revenues going off target – Reasons

· The GST Council, in its 31st meeting, cut rates on almost 20 categories of goods and a number
of services. As a result, only one common-use item — cement— was left in the highest tax slab
of 28%.

· Naturally, a reducing number of items in the 28% slab will result in lower collections. The way
to offset lower rates is to increase the number of people paying tax. That is, compliance has to
increase.

· Businesses generating fake invoices to claim higher input tax credits than they should be
receiving.

QUESTION 91. MTU4Njk4K0d1bnR1cnUgTmFyZW4gS3VtYXIrbmFyZW5pcHM2QGdtYWlsLmNvbStRVUVTVE


PTiA5MA==
‘The Future of Rail’ report is released by

a) World Economic Forum


b) International Rail Transport Union
c) International Energy Agency
d) International Union of Railways

IASbaba
Score:
Web: http://ilp.iasbaba.com/
47.00 /
Email: ilp@iasbaba.com
Page 68 200
2019 - Test 28-
Exam Title :
History & C...
Email : narenips6@gmail.com
Contact :
Correct Answer: C
Your Answer: Unanswered
Explanation

Solution (c)

The Future of Rail

· “The Future of Rail” of International Energy Agency (IEA)

· It is the first-of-a-kind report analyses the current and future importance of rail around the
world through the perspective of its energy and environmental implications.

· The report reviews the impact of existing plans and regulations on the future of rail, and
explores the key policies that could help to realise an enhanced future rail.

· This first ever global report has a focus on India, elaborating on the unique social and
economic role of rail in India, together with its great enduring potential, to show how India can
extend and update its networks to harness rail at a scope and scale that is unparalleled.

QUESTION 92. MTU4Njk4K0d1bnR1cnUgTmFyZW4gS3VtYXIrbmFyZW5pcHM2QGdtYWlsLmNvbStRVUVTVE


PTiA5MQ==
Consider the following statements with respect to ‘Golden Langur’

1. It is found in Western Ghats

2. It is listed as ‘Critically Endangered’ by the IUCN

Select the correct statements

a) 1 Only
b) 2 Only
c) Both 1 and 2
d) Neither 1 nor 2
Correct Answer: D
Your Answer: C
Explanation

Solution (d)

Gee's golden langur (Trachypithecus geei), or simply the golden langur, is an Old World monkey
found in a small region of western Assam, India and in the neighboring foothills of the Black
Mountains of Bhutan. It is one of the most endangered primate species of India. Long
considered sacred by many Himalayan people, the golden langur was first brought to the
attention of the western world by the naturalist E. P. Gee in the 1950s. In a part of Bhutan, it
has hybridised with the capped langur T. pileatus.

IUCN – Endagered

IASbaba
Score:
Web: http://ilp.iasbaba.com/
47.00 /
Email: ilp@iasbaba.com
Page 69 200
2019 - Test 28-
Exam Title :
History & C...
Email : narenips6@gmail.com
Contact :
QUESTION 93. MTU4Njk4K0d1bnR1cnUgTmFyZW4gS3VtYXIrbmFyZW5pcHM2QGdtYWlsLmNvbStRVUVTVE
PTiA5Mg==
‘Cholecystokinin’ is a hormone associated with

a) Blood Clotting
b) Breast Milk
c) Hunger suppressant
d) Cell reproduction and cell regeneration
Correct Answer: C
Your Answer: C
Explanation

Solution (c)

Cholecystokinin (CCK)

· It is a peptide hormone of the gastrointestinal system responsible for stimulating the digestion
of fat and protein.

· Cholecystokinin, officially called pancreozymin, is synthesized and secreted by


enteroendocrine cells in the duodenum, the first segment of the small intestine.

· Its presence causes the release of digestive enzymes and bile from the pancreas and
gallbladder, respectively, and also acts as a hunger suppressant.

· It is a satiety hormone which is highly expressed in memory formation, could, at higher levels,
decrease a person’s likelihood of developing Alzheimer’s disease by 65%.

· CCK is found in both the small intestines and the brain. In the small intestines, CCK allows for
the absorption of fats and proteins.

· In the brain, CCK is located in the hippocampus, which is the memory-forming region of the
brain.

QUESTION 94. MTU4Njk4K0d1bnR1cnUgTmFyZW4gS3VtYXIrbmFyZW5pcHM2QGdtYWlsLmNvbStRVUVTVE


PTiA5Mw==
‘Jamaat-ul-Mujahideen’, a terrorist organisation is from

a) Bangladesh
b) Pakistan
c) Afghanistan
d) Myanmar
Correct Answer: A
Your Answer: B
Explanation

Solution (a)

Jamaat-ul-Mujahideen

· It is an Islamic terrorist organisation operating in Bangladesh.

IASbaba
Score:
Web: http://ilp.iasbaba.com/
47.00 /
Email: ilp@iasbaba.com
Page 70 200
2019 - Test 28-
Exam Title :
History & C...
Email : narenips6@gmail.com
Contact :

· It is responsible for hatching the conspiracy to attack the Bodh Gaya temple and other symbols
of Buddhist faith in 2018.

QUESTION 95. MTU4Njk4K0d1bnR1cnUgTmFyZW4gS3VtYXIrbmFyZW5pcHM2QGdtYWlsLmNvbStRVUVTVE


PTiA5NA==
‘Karakkatam’ is associated with which of the following states?

a) Tamil Nadu
b) Karnataka
c) Telangana
d) Andhra Pradesh
Correct Answer: A
Your Answer: A
Explanation

Solution (a)

It is an ancient folk dance of Tamil Nadu performed in praise of the rain goddess Mariamman.
The ancient Tamil epic says that this type of dance has derived from Bharatham and a mixture
of multiple forms of Tamil Dance forms like Bharatanatyam postures, mudras. The offering of
this dance is to the goddess to bless Rain.

QUESTION 96. MTU4Njk4K0d1bnR1cnUgTmFyZW4gS3VtYXIrbmFyZW5pcHM2QGdtYWlsLmNvbStRVUVTVE


PTiA5NQ==
Consider the following statements with respect to ‘National Agricultural Higher Education
Project (NAHEP)’

1. It is launched by the Indian Council of Agricultural Research (ICAR)

2. It will be funded by the World Bank and the Indian Government on a 50:50 basis

Select the correct statements

a) 1 Only
b) 2 Only
c) Both 1 and 2
d) Neither 1 nor 2
Correct Answer: C
Your Answer: Unanswered
Explanation

Solution (c)

National Agricultural Higher Education Project (NAHEP)

· The Project supports the Country Partnership Strategy and addresses the three engagement
areas of integration, transformation and inclusion.

IASbaba
Score:
Web: http://ilp.iasbaba.com/
47.00 /
Email: ilp@iasbaba.com
Page 71 200
2019 - Test 28-
Exam Title :
History & C...
Email : narenips6@gmail.com
Contact :

· These engagement areas foresee increased agricultural productivity and support quality
improvements of higher education to create a more skilled workforce that continuously
improves the productivity of key sectors, including agriculture.

· The proposed Project is also a multi-Global Practice collaboration (Agriculture and Education)
and is expected to support activities and results directly related to cross-cutting strategic areas
of climate change, jobs and gender.

· ICAR has recently launched Rs 1100 crore ambitious National Agricultural Higher Education
Project (NAHEP) to attract talent and strengthen higher agricultural education in the country.

· This project will be funded by the World Bank and the Indian Government on a 50:50 basis.

· In addition, a four year degree in Agriculture, Horticulture, Fisheries and Forestry has been
declared a professional degree.

Objectives

· The objective of the Project is to support Participating Agricultural Universities and ICAR in
providing more relevant and higher quality education to Agricultural University students.

· NAHEP addresses quality by supporting interested AUs to propose and implement technically
sound and verifiable investments that increase faculty performance, attract better students to
these AUs, improve student learning outcomes and raise their prospects for future
employability, particularly in the private sector.

· NAHEP would target institutions that form the ICAR-AU System, consisting of State
Agricultural Universities, Deemed Universities, Central Universities with Agricultural Faculty
and Central Agricultural Universities.

DO YOU KNOW?

· In order to promote the participation of students in agricultural business, Student READY


(Rural Entrepreneurship Awareness Development Yojana) scheme is being run, under which
practical experience of agriculture and entrepreneurship is provided to undergraduate
students.

QUESTION 97. MTU4Njk4K0d1bnR1cnUgTmFyZW4gS3VtYXIrbmFyZW5pcHM2QGdtYWlsLmNvbStRVUVTVE


PTiA5Ng==
Which of the following have been awarded Conservation Assured |Tiger Standards (CA|TS)
approved status?

a) Bandipur Tiger Reserve


b) Buxa Tiger Reserve
c) Kaziranga Tiger Reserve
d) None of the above
Correct Answer: D
Your Answer: B
Explanation

Solution (d)

IASbaba
Score:
Web: http://ilp.iasbaba.com/
47.00 /
Email: ilp@iasbaba.com
Page 72 200
2019 - Test 28-
Exam Title :
History & C...
Email : narenips6@gmail.com
Contact :

Lansdowne Forest Division in Uttarakhand, India, Chitwan National Park in Nepal and Sikhote-
Alin Nature Reserve in Russia have been awarded CA|TS Approved status.

QUESTION 98. MTU4Njk4K0d1bnR1cnUgTmFyZW4gS3VtYXIrbmFyZW5pcHM2QGdtYWlsLmNvbStRVUVTVE


PTiA5Nw==
Which of the following tool, allows developing countries to raise tariffs temporarily to deal with
import surges or price falls?

a) Special Drawing Rights (SDR)


b) Common Risk Mitigation Mechanism (CRMM)
c) National Treatment Obligation (NTO)
d) Special Safeguard Mechanism (SSM)
Correct Answer: D
Your Answer: B
Explanation

Solution (d)

WTO’s Special Safeguard Mechanism (SSM) is a protection measure allowed for developing
countries to take contingency restrictions against agricultural imports that are causing injuries
to domestic farmers. The contingency measure is imposition of tariff if the import surge causes
welfare loss to the domestic poor farmers. The design and use of the SSM is an area of conflict
under the WTO.

In WTO’s terms, safeguards are contingency or emergency restrictions on imports taken


temporarily to deal with special circumstances such as a surge in imports. Contingency
restriction means imposition of an import tax if the imports are causing injuries to domestic
agricultural sector. The original GATT itself allows such restrictions to protect domestic
economy.

At the Doha Ministerial Conference, the developing countries were given a concession to adopt
a Special Safeguard Mechanism (SSM) besides the existing safeguards (like the Special
Agricultural Safeguard or the SSG). This SSM constituted an important part of the promises
offered to the developing world at Doha (known as Doha Development Agenda) and the Doha
MC became known as a development round.

As mentioned, the Special Safeguard Mechanism (SSM) allowed developing countries to raise
import duties on agricultural products in response to import surges.

The SSG was available to all countries- both developing and developed whereas the SSM is
allowable only to the developing countries. It is to be mentioned that the SSG was available as it
was inducted under the GATT agreement; whereas the SSM was the invention of the Doha MC.

QUESTION 99. MTU4Njk4K0d1bnR1cnUgTmFyZW4gS3VtYXIrbmFyZW5pcHM2QGdtYWlsLmNvbStRVUVTVE


PTiA5OA==
‘DAMAN’ acronym for ’Durgama Anchalare Malaria Nirakaran (malaria control in inaccessible
areas) is launched by which of the following states?

IASbaba
Score:
Web: http://ilp.iasbaba.com/
47.00 /
Email: ilp@iasbaba.com
Page 73 200
2019 - Test 28-
Exam Title :
History & C...
Email : narenips6@gmail.com
Contact :

a) Assam
b) Odisha
c) Kerala
d) Karnataka
Correct Answer: B
Your Answer: D
Explanation

Solution (b)

Durgama Anchalare Malaria Nirakaran (DAMaN)

· It is an initiative by the Odisha State Government aimed at strengthening malaria control


interventions

· It is led by Comprehensive Case Management Programme (CCMP)

· The programme is jointly implemented by Indian Council of Medical Research-National


Institute of Malaria Research (ICMR-NIMR), National Vector Borne Disease Control Programme
(NVBDCP), Odisha and Medicines for Malaria Venture (MMV).

· This programme includes mass screening for malaria with treatment of positive cases along
with intensified surveillance, mosquito control measures and regular health education activities
throughout the year.

QUESTION 100. MTU4Njk4K0d1bnR1cnUgTmFyZW4gS3VtYXIrbmFyZW5pcHM2QGdtYWlsLmNvbStRVUVTV


PTiA5OQ==
Which of the following is/are correctly matched?

Diseases Crop

1. Yellow Rust – Wheat

2. Quick wilt – Pepper

3. Stigmatomycosis – Cotton

Select the correct code:

a) 1 and 2
b) 2 and 3
c) 1 and 3
d) All of the above
Correct Answer: D
Your Answer: D
Explanation

Solution (d)

Quick wilt, also known as Phytophthora foot rot disease, causes sudden death of black pepper
vines.

IASbaba
Score:
Web: http://ilp.iasbaba.com/
47.00 /
Email: ilp@iasbaba.com
Page 74 200
2019 - Test 28-
Exam Title :
History & C...
Email : narenips6@gmail.com
Contact :

Stigmatomycosis is a fungal disease that occurs in a number of crops, such as cotton, soybean,
pecan, pomegranate, citrus, and pistachio.

Wheat yellow rust, also known as wheat stripe rust, is one of the three wheat rust diseases
principally found in wheat grown in cooler environments. Such locations are generally
associated with northern latitudes or cooler seasons.

IASbaba
Score:
Web: http://ilp.iasbaba.com/
47.00 /
Email: ilp@iasbaba.com
Page 75 200
2019 - Test 28-
Exam Title :
History & C...
Email : narenips6@gmail.com
Contact :
Review in Hindi
QUESTION 1.
�न�न�ल�खत�कथन��पर��वचार�क��जए�

1. लखु�दयार( Lakhudiyar ) म��पाए�गए��ागै�तहा�सक��च���म��केवल�मनु�य�और�पशु��के��च��ह��

2. लखु�दयार�म��पाए�गए��च���म��मानव�आकृ�तयाँ�का�छड़ी�क��तरह��न�पण��कया�गया�ह��

उपयु��कथन�का�चयन�क��जए�

a) केवल�1
b) केवल�2
c) 1 तथा�2 दोन��
d) इनम��से�कोई�नह��
Correct Answer: B
Your Answer:
Explanation

Solution (b)

म�य��दे श�, उ�र��दे श�, आं���दे श�, कना�टक�और��बहार�के�कई��जल��म����थत�गुफा��क��द�वार��पर�शैल��च���के�अवशेष�पाए�गए�ह�।


उ�राखंड�म��कुमाऊं�क��पहा�ड़य��से�भी�कुछ��च���क��सूचना��मली�है।�अ�मोड़ा-बरछ�ना�माग��पर�लगभग�बीस��कलोमीटर�क���री�पर
लखु�दयार�म��सुआल�नद��के�तट�पर���थत�ये�शैल�आ�य�इन��ागै�तहा�सक��च���को�धारण�करते�ह�।�लखु�दयार�का�शा��दक�अथ��है�एक�लाख
गुफाएँ।�यहां��च���को�तीन��े�णय��म���वभा�जत��कया�जा�सकता�है: मनु�य�, पशु�और��या�मतीय�पैटन��सफेद�, काले�और�लाल�गे�
रंग�म�।�मनु�य�को�छड़ी�जैसे��प��म��दशा�या�जाता�है।�एक�लॉ�ग-�नोऊटे ड( long-snouted ) जानवर�, एक�लोमड़ी�और�एक�ब�-पैर�वाली
�छपकली�मु�य�पशु��पांकन��ह�।�लहरदार�रेखाएं�, आयत�से�भरे��या�मतीय��डज़ाइन�और�डॉट् स�के�समूह�भी�यहाँ�दे खे�जा�सकते�ह�।�यहां
दशा�या�गया�एक��दलच�प���य�हाथ�से�जुड़े�नृ�य�मानव�आकृ�तय��का�है।��च���क��कुछ�सुप�र�पो�जशन�है।�सबसे�पहले�काले�रंग�म��ह��; इन�पर
लाल�गे��के��च��ह��और�अं�तम�समूह�म��सफेद��च��ह�।�

QUESTION 2.
�न�न�ल�खत�कथन��पर��वचार�क��जए�

1. ऊपरी�पुरापाषाण�काल�म���च��का��न�पण�रेखीय��प�म��था�जहाँ�नत�क��के��च��हरे�रंग�म��तथा��शका�रय��के��च��लाल�रंग�म��थे।�

2. म�यपाषाण�युग�म���शकार�के���य��च���क���धानता�थी�

3. म�यपाषाण�युग�के��च���म��मनु�य��को�केवल�शैलीगत�तरीके( stylistic manner ) से��च��त��कया�गया�था�

उपयु��कथन�का�चयन�क��जए�

a) केवल�1 तथा�2
b) केवल�2 तथा�3
c) केवल�1 तथा�3
d) उपरो��सभी�
Correct Answer: D
Your Answer:
Explanation

Solution (d)

ऊपरी�पुरापाषाण�काल�क��प��ट�ग�हरे�रंग�और�गहरे�लाल�रंग�म��, �वशाल�जानवर��के�आंकड़े�, जैसे��क�बाइसन�, हाथी�, बाघ�, ग�डे�और�सूअर


के�अलावा�छड़ी�जैसे�मानव�आंकड़े�ह�।�कुछ�वाश�प��ट�ग�ह��, ले�कन��यादातर�वे��या�मतीय�पैटन��से�भरे��ए�ह�।�हरे�रंग�के��च��नत��कय��के�ह�
और��शकारी�के�लाल�ह�।�

IASbaba
Score:
Web: http://ilp.iasbaba.com/
47.00 /
Email: ilp@iasbaba.com
Page 76 200
2019 - Test 28-
Exam Title :
History & C...
Email : narenips6@gmail.com
Contact :

�च���क��सबसे�बड़ी�सं�या�अव�ध�II क��है�जो�मेसो�ल�थक��च���को�कवर�करती�है।�इस�अव�ध�के�दौरान�थीम�कई�ह��ले�कन�प��ट�ग�आकार�म�
छोट��ह�।��शकार�के���य��बल�होते�ह�।��शकार�के���य�समूह��म���शकार�करने�वाले�लोग��को��च��त�करते�ह��, जो�कांटेदार�भाले�, नुक�ले�डंडे�,
तीर�और�धनुष�से�लैस�होते�ह�।�हालां�क�जानवर��को�एक��ाकृ�तक�शैली�म���च��त��कया�गया�था�, मानव��को�केवल�एक�शैलीगत�तरीके�से
�च��त��कया�गया�था।�म�हला��को�न�न�और�कपड़े�पहने�दोन��म���च��त��कया�जाता�है।�युवा�और�बूढ़े�समान��प�से�इन��च���म��जगह�पाते
ह�।�ब�च��को�दौड़ते�, कूदते�और�खेलते��ए��च��त��कया�जाता�है।�सामुदा�यक�नृ�य�एक�सामा�य��वषय��दान�करते�ह�।�

QUESTION 3.
�स�धु�घाट��स�यता�के�बारे�म���न�न�ल�खत�कथन��पर��वचार�क��जए�

1. प�थर�और�कां�य�क��मू�त�य��क��तुलना�म��मनु�य��क��टे राकोटा�मू�त�याँ�क�चे��प�म��ह��

2. �स�धु�घाट��स�यता�के�दौरान�, कां�य�का��टं ग�तकनीक�केवल�हड़�पा�म���च�लत�थी�

3. इस�दौरान�कां�य�क��मू�त�याँ�केवल�मनु�य�क��पायी�जाती�ह��

सही�कूट�का�चयन�क��जए�

a) केवल�1 तथा�2
b) केवल�2 तथा�3
c) केवल�1 तथा�3
d) उपरो��म��से�कोई�नह��
Correct Answer: D
Your Answer:
Explanation

Solution (d)

हड़�पा��थल��म��प�थर�, कां�य�या�टे राकोटा�क��मू�त�याँ��चुर�मा�ा�म��नह��ह��, ले�कन�प�र�कृत�ह�।�हड़�पा�और�मोहनजोदड़ो�म��पाए�जाने�वाले


प�थर�क��मू�त�याँ�तीन�आयामी�खंड��को�संभालने�के�उ�कृ��उदाहरण�ह�।�प�थर�म��दो�पु�ष�आकृ�तयाँ�ह��- एक�लाल�बलुआ�प�थर�म��एक�धड़�है
और��सरा�लाल�बलुआ�प�थर�म��एक�दाढ़��वाला�आदमी�है�- �जसक��बड़े�पैमाने�पर�चचा��क��जाती�है।�

हड़�पा�वा�सय���ारा�कां�य-ढलाई�क��कला�का��ापक��तर�पर�अ�यास��कया�गया�था।�कां�य�म��हम�मनु�य��के�साथ-साथ�जानवर��क�
आकृ�तयाँ�भी�दे खते�ह��, पूव��का�सबसे�अ�छा�उदाहरण�एक�लड़क��क��मू�त��है��जसका�नाम�' डां�स�ग�गल��' है।��स�धु�घाट��स�यता�के�सभी
�मुख�क����म��कां�य�का��टं ग�लोक��य�थी।�तांबे�के�कु�े�और�लोथल�के�प�ी�और�कालीबंगन�के�एक�बैल�क��कां�य�आकृ�त��कसी�भी�तरह�से
हड़�पा�और�मोहनजो-दारो�के�तांबे�और�कां�य�क��मानव�आकृ�तय��से�हीन�नह��है।�

�स�धु�घाट��के�लोग��ने�टे राकोटा�क��छ�वयां�भी�बना��, ले�कन�प�थर�और�कां�य�क��मू�त�य��क��तुलना�म���स�धु�घाट��म��मानव��प�के�टे राकोटा


��त�न�ध�व�क�चे�ह�।�वे�गुजरात��थल��और�कालीबंगन�म��अ�धक�यथाथ�वाद��ह�।��स�धु�के�आंकड़��म��सबसे�मह�वपूण��दे वी�मां�का���त�न�ध�व
करती�ह�।�

QUESTION 4.
�न�न�ल�खत�म��से�कौन-सा/से�सही�सुमे�लत�है�?

�थान��वशेषता�

1. लोथल�A) शव��को�आभूषण��के�साथ�दफनाया�गया�था�

2. धोलावीरा�B) प�थर�संरचना�के�अवशेष�

3. ह�रयाणा�म��फरमाना�C) मनका�फै����

सही�कूट�का�चयन�क��जए�

IASbaba
Score:
Web: http://ilp.iasbaba.com/
47.00 /
Email: ilp@iasbaba.com
Page 77 200
2019 - Test 28-
Exam Title :
History & C...
Email : narenips6@gmail.com
Contact :

a) केवल�1 तथा�2
b) केवल�2
c) केवल�1 तथा�3
d) उपरो��सभी�
Correct Answer: B
Your Answer:
Explanation

Solution (b)

हड़�पा�के�पु�ष��और�म�हला��ने�क�मती�धातु��और�र�न��से�लेकर�ह�ी�और�पके��ए��म���से�लेकर�हर�क�पनीय�साम�ी�से��न�म�त�कई
�कार�के�आभूषण��से��वयं�को�सजाया।�सभी�गहने�अ�छ��तरह�से�गढ़े �गए�ह�।�यह��यान��दया�जा�सकता�है��क�ह�रयाणा�के�फरमाना�म��एक
क���तान�पाया�गया�है�जहाँ�शव��को�गहन��के�साथ�दफनाया�गया�था।�

मनका�उ�ोग�अ�छ��तरह�से��वक�सत��कया�गया�है�जैसा��क�च��दडो�और�लोथल�म��खोजी�गई�फै���य��से��प��है।�बीड् स�कारे�लयन�, नीलम


, जै�पर�, ���टल�, �वाट्� ज�, �ट�टाइट�, �फ़रोज़ा�, लै�पस�लज�ली�आ�द�धातु��से�बने�होते�थे।�तांबा�, कां�य�और�सोने�जैसी�धातुए�ँ और�शेल�
, फ़ाइनेस�और�टे राकोटा�और�जली��म���का�उपयोग��व�नमा�ण�मो�तय��के��लए�भी��कया�जाता�था।�

पुराता��वक�खोज��से�ऐसा��तीत�होता�है��क��स�धु�घाट��के�लोग�फैशन�के���त�सचेत�थे।��व�भ��हेयर��टाइल��चलन�म��थे�और�दाढ़��पहनना
सभी�के�बीच�लोक��य�था।��स�ाबार�को�एक�कॉ�मे�टक�के��प�म��इ�तेमाल��कया�गया�था�और�फेसप�ट�, �लप��टक�और�कोली�रयम
(आईलाइनर) भी�उनके��लए�जाने�जाते�थे।�धोलावीरा�म��कई�प�थर�के�संरचना�मक�अवशेष�भी��मले�ह��, जो�बताते�ह���क��स�धु�घाट��के�लोग
�कस�तरह�से��नमा�ण�म��प�थर�का�इ�तेमाल�करते�थे।�

QUESTION 5.
�स�धु�घाट��स�यता�क����स��नृतक��क��मू�त��के�बारे�म���न�न�ल�खत�कथन��पर��वचार�क��जए।�

1. यह�मोहनजोदड़ो�म��पायी�गयी�है�

2. इसे�खोई��ई�मोम�तकनीक( lost wax technique ) के�उपयोग�से�कां�य��ारा��न�म�त��कया�गया�है�

3. यह�आकृ�त�अ�भ����तथा�शारी�रक�जोश�से�भरपूर�है�तथा�ब�त�सारी�जानकारी��दान�करती�है�

सही�कूट�का�चयन�क��जए�

a) केवल�1 तथा�2
b) केवल�2
c) केवल�1 तथा�3
d) उपरो��म��से�कोई�नह��
Correct Answer: C
Your Answer:
Explanation

Solution (c)

�स�धु�घाट��से�सबसे���स���ात�कलाकृ�तय��म��से�एक�यह�एक�नृ�य�करने�वाली�लड़क��क��लगभग�चार�इंच�ऊंची�तांबे�क��आकृ�त�है।
मोहनजो-दारो�म���मली�इस�ए�स�लू�सव�का��टं ग�म��एक�लड़क��को��दखाया�गया�है��जसके�लंबे�बाल�एक�जुड़े�म��बंधे�ह�।�चू�ड़याँ�उसक��बाय�
बाँह�को�ढँ कती�ह��, एक�कंगन�और�एक�ताबीज�या�चूड़ी�उसके�दा�हने�हाथ�म��सजी�है�, और�उसके�गले�म��एक�कावरी�शैल�हार��दखाई�दे ता�है।
उसका�दा�हना�हाथ�उसके�कू�हे�पर�है�और�उसका�बायाँ�हाथ�पारंप�रक�भारतीय�नृ�य�क��मु�ा�म���आ�है।�उसक��बड़ी�आँख��और�चपट��नाक�है।
यह�आंकड़ा�अ�भ����और�शारी�रक�श���से�भरा�है�और�ब�त�सारी�जानकारी�बताता�है।�

QUESTION 6.
�न�न�ल�खत�कथन��पर��वचार�क��जए�

IASbaba
Score:
Web: http://ilp.iasbaba.com/
47.00 /
Email: ilp@iasbaba.com
Page 78 200
2019 - Test 28-
Exam Title :
History & C...
Email : narenips6@gmail.com
Contact :

1. बौ��धम��के�आगमन�के�प�ात�य��पूजा�को��मुखता��ा�त��ई�

2. लोमस�ऋ�ष�क��गुफा�एक�रॉक�कट�गुफा�है�जो�अशोक��ारा�आजी�वक�सं�दाय�को�दान�क��गई�थी�

3. मौय���तंभ��का��नमा�ण�एक�राज�म��ी��ारा�टु कड़��म���कया�गया�था�जो�मू�त�कार�के�कौशल�को��द�श�त�करता�है�

सही�कूट�का�चयन�क��जए�

a) केवल�1 तथा�2
b) केवल�2
c) केवल�1 तथा�3
d) उपरो��म��से�कोई�नह��
Correct Answer: B
Your Answer:
Explanation

Solution (b)

बौ��धम��के�आगमन�के�पहले�और�बाद�म��य��पूजा�ब�त�लोक��य�थी�और�इसे�बौ��और�जैन�धम��म��आ�मसात��कया�गया�था।�मठवासी
��त�ान��के��ह�से�के��प�म���तूप��और��वहार��का��नमा�ण�बौ��परंपरा�का��ह�सा�बन�गया।�

हालां�क�, इस�अव�ध�म��, �तूप��और��वहार��के�अ�त�र��, प�थर�के��त�भ�, रॉक-कट�गुफाएं�और��मारक�क��मू�त�यां�कई��थान��पर�खुद���ई


थ�।��तंभ��का��नमा�ण�करने�क��परंपरा�ब�त�पुरानी�है�और�यह�दे खा�जा�सकता�है��क��तंभ��का��नमा�ण�ईरानी�सा�ा�य�म��भी��च�लत�था।
ले�कन�मौय���तंभ�, ईरानी��तंभ��से�अलग�ह�।�मौय���तंभ�च�ानी��तंभ�ह��, इस��कार�काव�र�के�कौशल�को��द�श�त�करते�ह��, जब�क�ईरानी��तंभ�
का��नमा�ण�राज�म��ी��ारा�टु कड़��म���कया�जाता�है।�

�बहार�म��गया�के�पास�बाराबर�पहा�ड़य��पर�न�काशीदार�रॉक-कट�गुफा�को�लोमस�ऋ�ष�गुफा�के��प�म��जाना�जाता�है।�गुफा�के�मोच��को��वेश
�ार�के��प�म��अध�वृ�ाकार�चै�य�मेहराब�से�सजाया�गया�है।�चै�य�आच��पर�उ�च�राहत�म��खुद���ई�हाथी���जी�काफ��आंदोलन�को�दशा�ती�है।
इस�गुफा�का�आंत�रक�हॉल�आयताकार�है��जसके�पीछे �एक�गोलाकार�क��है।��वेश��ार�हॉल�क��द�वार�पर���थत�है।�इस�गुफा�को�अशोक�ने
अजी�वका�सं�दाय�के��लए�दान��कया�था�

QUESTION 7.
बु��के�अवशेष��पर��न�म�त��न�न�ल�खत��तूप��म��से�कौन�सा�गंगा�घाट��से�बाहर���थत�है�?

1. राजगृह�

2. वैशाली�

3. अव�ती�

4. गंधार�

सही�कूट�का�चयन�क��जए�

a) केवल�1 तथा�2
b) केवल�2 तथा�4
c) केवल�3 तथा�4
d) केवल�1 तथा�3
Correct Answer: C
Your Answer:
Explanation

Solution (c)

IASbaba
Score:
Web: http://ilp.iasbaba.com/
47.00 /
Email: ilp@iasbaba.com
Page 79 200
2019 - Test 28-
Exam Title :
History & C...
Email : narenips6@gmail.com
Contact :

बौ��धम��और�जैन�धम��क��लोक��यता�के�कारण�, �तूप�और��वहार�का��नमा�ण�बड़े�पैमाने�पर��कया�गया�था।�हालां�क�, मू�त�कला�म��कुछ


�ा�णवाद��दे वता��के�उदाहरण�भी�ह�।�यह��यान�रखना�मह�वपूण��है��क��तूप��का��नमा�ण��बहार�म��राजगृह�, वैशाली�, वेथाद�पा�और�पावा�म��
, क�पलव�तु�, अ�लाक�पा�और�नेपाल�म��रामा�ाम�, उ�र��दे श�म��कुशीनगर�और��पपलवीना�म��बु��के�अवशेष��पर��कया�गया।��ंथ�क�
परंपरा�म��अवं�त�और�गांधार�स�हत�कई��थान��पर�बु��के�अवशेष��पर��व�भ��अ�य��तूप��के��नमा�ण�का�उ�लेख�है�जो�गंगा�क��घाट��के�बाहर
ह�।��तूप�, �वहार�और�चै�य�बौ��और�जैन�मठ�प�रसर�का��ह�सा�ह��ले�कन�सबसे�बड़ी�सं�या�बौ��धम��क��है।�

QUESTION 8.
सारनाथ�म���स�ह�क��राजधानी�के�बारे�म���न�न�ल�खत�कथन��पर��वचार�क��जए�

1. इसे�बु���ारा��थम�उपदे श�या�धम�च��वत�न�क��ऐ�तहा�सक�घटना�के��मरणो�सव�म���न�म�त��कया�गया�था।�

2. शीष��फलक�म��चार�जानवर��का��च�ण��कया�गया�है��जसम��एक�बैल�, एक�घोड़ा�, एक�हाथी�और�एक��स�ह�है�

3. �ाउ�न�ग��हील�के��बना�राजधानी�को��वतं��भारत�के�रा�ीय��तीक�के��प�म��अपनाया�गया�है�

सही�कूट�का�चयन�क��जए�

a) केवल�1 तथा�2
b) केवल�2 तथा�4
c) केवल�3 तथा�4
d) केवल�1 तथा�3
Correct Answer: A
Your Answer:
Explanation

Solution (a)

वाराणसी�के�पास�सारनाथ�म��सौ�साल�से�भी�पहले�क��खोज�क��गई�लायन�कै�पटल�को�आमतौर�पर�सारनाथ�लायन�कै�पटल�के��प�म��जाना
जाता�है।�यह�मौय��काल�से�मू�त�कला�के�बेहतरीन�उदाहरण��म��से�एक�है।�सारनाथ�म��बु���ारा��थम�उपदे श�या�ध�मच��वत�न�क��ऐ�तहा�सक
घटना�के��मरणो�सव�म���न�म�त�, राजधानी�का��नमा�ण�अशोक��ारा��कया�गया�था।�

राजधानी�म��मूल��प�से�पांच�घटक�भाग�होते�ह�: ( i) शा�ट�(जो�अब�कई�भाग��म��टू ट�गया�है) , (ii) कमल�बेल�आधार�, (iii) बेल�आधार�पर


�म��जसे�चार�जानवर�आगे�बढ़ते�ह��, द��णावत��( iv) चार�राजसी�और�( v) मुकुट�त�व�, धरमच��, एक�बड़ा�प�हया�, भी�इस��तंभ�का�एक
�ह�सा�था।�हालां�क�, यह�प�हया�टू ट��हालत�म��पड़ा��आ�है�और�सारनाथ�म��साइट�सं�हालय�म���द�श�त��कया�गया�है।�मुकुट�प�हया�और�कमल
आधार�के��बना�पूंजी�को��वतं��भारत�के�रा�ीय��तीक�के��प�म��अपनाया�गया�है।�अबेकस�म��एक�च��(प�हए) का��च�ण�होता�है�, �जसम�
चार���दशा��म��चौबीस��पोक�होते�ह��और�एक�बैल�, एक�घोड़ा�, एक�हाथी�और�हर�च��के�बीच�म��एक�शेर�होता�है।�

QUESTION 9.
मधुबनी��च�कला�के�बारे�म���न�न�ल�खत�कथन��पर��वचार�क��जए�

1. यह��बहार�म���च�लत�लोक�शैली��च�कला�का�एक�लोक��य��प�है�

2. समकालीन��श�के��थान�पर��च���को�बनाने�के��लए�टह�नयाँ�, मा�चस�क��तीलीय��और�यहाँ�तक��क�उंग�लय��जैसी�व�तु��का�उपयोग
�कया�जाता�है�

3. तां��क�, भरणी�और�गोधना�मधुबनी�प��ट�ग�से�जुड़ी�अलग-अलग�शै�लयाँ�थ��

सही�कूट�का�चयन�क��जए�

a) केवल�1 तथा�2
b) केवल�2 तथा�4
c) केवल�3 तथा�4

IASbaba
Score:
Web: http://ilp.iasbaba.com/
47.00 /
Email: ilp@iasbaba.com
Page 80 200
2019 - Test 28-
Exam Title :
History & C...
Email : narenips6@gmail.com
Contact :

d) उपरो��सभी�
Correct Answer: D
Your Answer:
Explanation

Solution (d)

मधुबनी�प��ट�ग�कई���स��भारतीय�कला��प��म��से�एक�है।�जैसा��क��बहार�और�नेपाल�के��म�थला��े��म���च�लत�है�, इसे��म�थला�या�मधुबनी
कला�कहा�जाता�है।�अ�सर�ज�टल��या�मतीय�पैटन��क���वशेषता�होती�है�, इन��च���को��योहार��, धा�म�क�अनु�ान��आ�द�स�हत��वशेष�अवसर�
के��लए�अनु�ान�साम�ी�का���त�न�ध�व�करने�के��लए�जाना�जाता�है।�मधुबनी��च���म���यु��रंग�आमतौर�पर�पौध��और�अ�य��ाकृ�तक��ोत�
से��ा�त�होते�ह�।�ये�रंग�अ�सर�चमक�ले�होते�ह��और�लै�प�लेक�और�गे��जैसे��पगम�ट��मशः�काले�और�भूरे�रंग�के�बनाने�के��लए�उपयोग��कए
जाते�ह�।�समकालीन��श�के�बजाय�, प��ट�ग�बनाने�के��लए�टह�नयाँ�, मा�चस�और�यहाँ�तक��क�उंग�लय��जैसी�व�तु��का�उपयोग��कया�जाता
है।�मधुबनी��च���को�शु��म��लोग��के��व�भ��सं�दाय���ारा�अ�यास��कया�गया�था�और�इस�लए��च���को�पांच�अलग-अलग�शै�लय��म��वग�कृत
�कया�गया�था�, जैसे��क�तां��क�, कोहबर�, भरणी�, गोडना�, कतचन।�ले�कन�आज�, इन�पांच�अलग-अलग�शै�लय��को�समकालीन�कलाकार�
�ारा��वलय�कर��दया�गया�है।�इन��च���म���यु���वषय�अ�सर�कृ�ण�, राम�, ल�मी�, �शव�, �गा��और�सर�वती�जैसे��ह���दे वता��के�इद� -�गद�
घूमते�ह�।�इसके�अलावा�, सूय��और�चं�मा�जैसे��वग�य��प�ड�अ�सर�मधुबनी��च���का�क����ब���बनते�ह�।�

QUESTION 10.
�न�न�ल�खत�यु�म��पर��वचार�क��जए�

1. अ�णाचल��दे श�– लोसर�फे��टवल�

2. महारा��- गणेश�चतुथ��

3. गोवा�- का�न�वल�

4. ह�रयाणा�- ब�तर�दशहरा�

सही�कूट�का�चयन�क��जए�

a) केवल�1, 2 तथा�3
b) केवल�2, 3 तथा�4
c) केवल�3 तथा�4
d) उपरो��सभी�
Correct Answer: A
Your Answer:
Explanation

Solution (a)

गणेश�चतुथ��, महारा��का�सबसे�मह�वपूण���योहार�अग�त�- �सतंबर�के�महीन��म��आता�है।�लोग���ारा�पांच�से�दस��दन��के��लए��द��अ�त�थ


के��प�म��घर��म��गणेश���तमाएं�रखी�जाती�ह�।�तब�छ�व�को�औपचा�रक��प�से��नकाल��लया�जाता�है�और�नद��, समु��या�कुएं�म���वस�ज�त�कर
�दया�जाता�है।�इसे��वसज�न�कहा�जाता�है।�गणेश�चतुथ��समारोह�के��लए�मोदक��वशेष��ंजन�है।�

का�न�वल�म��शहर��के�मा�यम�से��वशाल�परेड�ब�ड�, झां�कय��और�शाम�और�नृ�य�के�साथ�आयो�जत��कए�जाते�ह�।�अं�तम��दन�पं�जम�म���लुब
नेशनल��ारा�आयो�जत���स��रेड-एंड-�लैक�नृ�य�के�साथ�संप���आ।�का�न�वल�क��अ�य�ता��क�ग�मोमो��ारा�क��जाती�है�, जो�पहले��दन
अपने��वषय��को�पाट��करने�का�आदे श�दे ता�है।�

ब�तर�दशहरा�, ��नया�का�सबसे�लंबा�दशहरा�उ�सव�छ�ीसगढ़�के�ब�तर��े��म��मनाया�जाता�है�और�75 �दन��म��अग�त�के�आसपास�शु�


होता�है�और�अ�टू बर�म��समा�त�होता�है।��व�वध�जनजा�तयाँ�पारंप�रक�री�त-�रवाज��के�साथ�भाग�लेती�ह�।�

ब�तर�दशहरा�' पाट�जा�ा�' जैसे�कई�री�त-�रवाज��से�शु��होता�है�, या�लकड़ी�क��पूजा�होती�है�, इसके�बाद�' दारी�गढई�'- �तंभ��क��पो��टं ग�,


काचन�गाद�- दे वी�काचन�दे वी�के��लए��स�हासन�, कलश��थापन- कलश��थापना�, जोगी��बठाई- जोगी�क��तप�या�रथ�प�र�मा- रथ�स�क�ट�,

IASbaba
Score:
Web: http://ilp.iasbaba.com/
47.00 /
Email: ilp@iasbaba.com
Page 81 200
2019 - Test 28-
Exam Title :
History & C...
Email : narenips6@gmail.com
Contact :
�नशा�जा�ा- �नशाचर�पव��, जोगी�उथाई- जोगी�क��प�र�मा�, मौली�परघव- दे वी�मौली�का��वागत�, बठार�रेन- इनर�स�क�ट�, बाहार�रैनी- बाहरी
स�क�ट�, कचन�जा�ा- ध�यवाद�समारोह�, ।�मु�रया�दरबार- आ�दवासी�स�मेलन�और�अंत�म��अं�तम��दन�, ओहद�- दे वता��को��वदाई।�

�त�बती�नव�वष��को�अ�णाचल��दे श�म��लोसार�के��प�म��भी�जाना�जाता�है�और�मु�य��प�से�जनवरी�के�अंत�या�फरवरी�म��3 �दन��के��लए


मनाया�जाता�है।��ाचीन�समारोह��के�साथ��च��त�है�जो�अ�छे �और�बुरे�के�बीच�संघष��का���त�न�ध�व�करते�ह��हा�सार�म��लोग��को�भीड़�के
मा�यम�से�जप�और�आग�क��मशाल��गुजरती�ह�।�

�हरन�के�नृ�य�और�राजा�और�उनके��व�भ��मं��य��के�बीच�मनोरंजक�लड़ाई�जैसे�काय��म�उ�सव�का�आनंददायक�प���दान�करते�ह�।�लोसार
महो�सव�म���वशेष��प�से�नृ�य�, संगीत�और�मीरा�बनाने�क��एक�सामा�य�भावना�क���वशेषता�है।�

QUESTION 11.
लॉसो�ग�फे��टवल( Losoong festival ) �न�न�म��से�कौन-से�रा�य�म��आयो�जत��कया�जाता�है�?

a) �स��कम�
b) म�णपुर�
c) �हमाचल��दे श�
d) �मजोरम�
Correct Answer: A
Your Answer:
Explanation

Solution (a)

लॉसो�ग��स��कम�का�नया�साल�है।��कसान�क��खुशी�के��लए�इसे�' सोनम�लोसार�' ( या��कसान�का�नया�साल) भी�कहा�जाता�है।�लॉसंग�ने


फसल�के�मौसम�के�अंत�और�चं��कैल�डर�के�दसव��महीने�के�अंत�को�भी��च��त��कया�है।�यह�अ�छ��फसल�के��लए�ध�यवाद�और�अगले�सीजन
म��बेहतर�के��लए��ाथ�ना�क��पेशकश�करने�के��लए�एक�पारंप�रक�उ�सव�है।�हालाँ�क�, प�रवार�के�सद�य��और�दो�त��के�बीच�लोसॉ�ग�को��नजी
तौर�पर�मनाया�जाता�है�, ले�कन�उ�सव�क��हवा�है�और��स��कम�के�युवा��म��तीरंदाजी�क����तयो�गताएं�ह�।�यह��यौहार�चोम�(नकाबपोश)
�ारा��सुंग-ला-खांग�पैलेस�मं�दर�म��, फोड�ग�म��और��मटे क�म���च��त��कया�गया�है।�नृ�य�वष��क��बुरी�आ�मा��और�नए�साल�क��अ�छ�
आ�मा�के��वागत�का��तीक�है।�इस��यौहार�को��स��कम�के��ामीण��े���म��दे खना�सबसे�अ�छा�है�, जहाँ�यह�पारंप�रक�तीरंदाजी
��तयो�गता��और�अ�य�काय��म��के�साथ�अ�धक�धूमधाम�से�मनाया�जाता�है।�

QUESTION 12.
इस�जातक�पैनल�के��च���ेम�म��घटना�म��राजा�को�अपनी�सेना�के�साथ�और��हरण�पर�तीर�चलाने�के�बारे�म���दखाया�गया�है�, और��जस����
को��हरण��ारा�बचाया�गया�था�, उसे�राजा�के�साथ��हरण�पर�उंगली�से�इशारा�करते��ए�भी��दखाया�गया�है।�

�न�न�म��से�कौन�सा�जातक�पैनल�यहाँ�संद�भ�त��कया�जा�रहा�है�?

a) जातक�पैनल�, अजंता�
b) जातक�पैनल�, एलोरा�
c) जातक�पैनल�, भ�हट�
d) जातक�पैनल�, सांची�
Correct Answer: C
Your Answer:
Explanation

Solution (c)

जातक�कहानी�का��च�ण�ब�त�ही�सरल�है�- कहानी�क��भौगो�लक���थ�त�के�अनुसार�घटना��को��लब�करके��च��त��कया�जाता�है�जैसे��क
���जातक�का��च�ण�जहां�बो�धस�व�मृग�अपनी�पीठ�पर�एक�आदमी�को�बचाता�है।�उसी��च���ेम�म��अ�य�घटना�म��राजा�को�अपनी�सेना�के
साथ�खड़े�होने�और��हरण�पर�एक�तीर�चलाने�के�बारे�म���दखाया�गया�है�, और��जस�����को��हरण��ारा�बचाया�गया�था�, उसे�राजा�के�साथ
�हरण�पर�उंगली��दखाते��ए�भी��दखाया�गया�है।�कहानी�के�अनुसार�, आदमी�ने�अपने�बचाव�के�बाद��हरण�से�वादा��कया��क�वह�अपनी
पहचान��कसी�के�सामने�जा�हर�नह��करेगा।�ले�कन�जब�राजा��हरण�क��पहचान�का�खुलासा�करने�के��लए�इनाम�क��घोषणा�करता�है�, तो�वह

IASbaba
Score:
Web: http://ilp.iasbaba.com/
47.00 /
Email: ilp@iasbaba.com
Page 82 200
2019 - Test 28-
Exam Title :
History & C...
Email : narenips6@gmail.com
Contact :
श�ुतापूण��हो�जाता�है�और�राजा�को�उसी�जंगल�म��ले�जाता�है�जहां�उसने��हरण�को�दे खा�था।�ऐसी�जातक�कथाएँ��तूप�स�जा�का��ह�सा�बन�।
�दलच�प�है�, दे श�के��व�भ���ह�स��म���तूप��के��नमा�ण�म��वृ���के�साथ�, �े�ीय�शैलीगत��भ�ताएं�भी�उभरने�लग�।�पहली-�सरी�शता�द��ईसा
पूव��के�सभी�पु�ष��च���म��एक�मु�य��वशेषता�नॉटे ड�हेड�गयर�है।�कई�मू�त�य��म��यह�ब�त�सुसंगत�है।�भर�त�म���मली�कुछ�मू�त�यां�भारतीय
सं�हालय�, कोलकाता�म���द�श�त�ह�।�

QUESTION 13.
�न�न�ल�खत�म��से�कौन�सही��प�से�सुमे�लत�है�?

गुफा���������������������थान�

1. अजंता�A) गजपृ�ठाकार�वॉ�ट-�फ�चै�य�हॉल�

2. थाना-नंदस
् ौर�( Thana-Nadsur ) B) गजपृ�ठाकार�वॉ�ट-�फ��तंभ�के��बना�हॉल�

3. क�डा�वते�C) �लैट-छत�वाला�चतु�कोणीय�हॉल��जसम��पीछे �एक�गोलाकार�क��है�

सही�कूट�का�चयन�क��जए�

a) केवल�1 तथा�2
b) केवल�2 तथा�3
c) केवल�1 तथा�3
d) उपरो��सभी�
Correct Answer: D
Your Answer: Unanswered
Explanation

Solution (d)

प��मी�भारत�म��, ईसा�पूव���सरी�शता�द��के�बाद�क��कई�बौ��गुफा��क��खुदाई�क��गई�है।�मु�य��प�से�तीन�वा�तु�श�प��कार��को
�न�पा�दत��कया�गया�था- ( i) अपसाइडल�वॉ�ट-�फ�चै�य�हॉल�(अजंता�, �पटकुलोरा�, भजा�म��पाया�गया) ; (ii) अ�साइडल�वॉ�ट-�फ
�पलरलेस�हॉल�(महारा��म��थाना-नादसुर�म��पाया�जाता�है) ; और�( iii) पीछे �क��ओर�एक�गोलाकार�क��(महारा��म��क��डवाइट�म��पाया
जाता�है) के�साथ�सपाट�छत�वाला�चतुभु�ज�हॉल।�चै�य�हॉल�के�सामने�एक�अध�-गोलाकार�चै�य�आच��के��पांकन��का�वच��व�है�, �जसम��एक
खुला�मोचा��है�, �जसम��लकड़ी�का�अ�भाग�है�और�कुछ�मामल��म��, कोई�वच��व�वाले�चै�य�आच���व�डो�नह��है�जैसे��क�क�डाइ�वट�म��पाया�जाता
है।�सभी�चै�य�गुफा��म��पीठ�पर�एक��तूप�आम�है।�

QUESTION 14.
�न�न�ल�खत�कथन��पर��वचार�क��जए�

1. अजंता�पहली�शता�द��ईसा�पूव��और�पांचव��शता�द��ई�वी�क���च�कला�का�एकमा��जी�वत�उदाहरण�है�

2. काला��म��सबसे�बड़ी�गुफा�क��खुदाई��ई�है�

3. जु�ार�म��सबसे�बड़े�रॉक-कट�चै�य�हॉल�क��खुदाई�क��गयी�थी�

सही�कूट�का�चयन�क��जए�

a) केवल�1
b) केवल�2 तथा�3
c) केवल�1 तथा�3
d) उपरो��सभी�
Correct Answer: A
Your Answer:
Explanation

IASbaba
Score:
Web: http://ilp.iasbaba.com/
47.00 /
Email: ilp@iasbaba.com
Page 83 200
2019 - Test 28-
Exam Title :
History & C...
Email : narenips6@gmail.com
Contact :

Solution (a)

काल��म��, सबसे�बड़ा�रॉक-कट�चै�य�हॉल�खुदाई��कया�गया�था।�गुफा�म��दो�खंभ��के�साथ�एक�खुला��ांगण�है�, �जसम��बा�रश�से�बचाने�के��लए


एक�प�थर�क�����न�क��द�वार�, एक�बरामदा�, एक�प�थर�क�����न�द�वार�के��प�म��, �तंभ��के�साथ�एक�अ�सरा�मक�वॉ�ट-छत�चै�य�हॉल
और�पीछे �एक��तूप�है।�काल��चै�य�हॉल�को�मानव�और�जानवर��क��आकृ�तय��से�सजाया�गया�है।�

चौथी�और�पाँचव��शता�द��के��तूप��म��बु��क����तमाएँ�लगी��ई�ह�।�जु�ार�क��सबसे�बड़ी�गुफा�क��खुदाई�है- शहर�क��पहा�ड़य��के�चार��ओर
दो�सौ�से�अ�धक�गुफाएँ- जब�क�मुंबई�म��क�हेरी�म��एक�सौ�आठ�खुदाई�गुफाएँ�ह�।�सबसे�मह�वपूण���थल�अजंता�, �प�लखोरा�, एलोरा�, ना�स
क�, भजा�, जु�ार�, करला�, क�हेरी�ह�।�अजंता�, एलोरा�, और�क�हेरी�पनपना�जारी�है।�

सबसे���स��गुफा��थल�अजंता�है।�अजंता�म��उनतीस�गुफाएँ�ह�।�इसम��चार�चै�य�गुफाएं�ह��जो�पहले�चरण�म��, यानी�, �सरी�और�पहली�शता�द�


ईसा�पूव��(गुफा�नं।�10 और�9) और�बाद�क��अव�था�, यानी�पांचव��शता�द��सीई�(गुफा�नं।�19 और�26) ।�इसम��बड़े�चै�य�वहार�ह��और�यह
मू�त�य��और��च���से�सजाया�गया�है।�अजंता�पहली�शता�द��ईसा�पूव��और�पांचव��शता�द��ई�वी�क��प��ट�ग�का�एकमा��जी�वत�उदाहरण�है।�

QUESTION 15.
अजंता�म��गुफा��के�साथ�संर�क�के��न�न�ल�खत�यु�म�पर��वचार�क��जए�

1. वराहदे व�- गुफा�सं�या�26 के�संर�क�

2. उपे��गु�त�– गुफा�सं�या�17-20 के�संर�क�

3. बु�भ��– गुफा�सं�या�के�16 संर�क�

4. मथुरादास�- गुफा�सं�या�4 के�संर�क�

सही�कूट�का�चयन�क��जए�

a) केवल�2 तथा�4
b) केवल�1 तथा�3
c) केवल�1,2 तथा�4
d) केवल�1,2 तथा�3
Correct Answer: A
Your Answer:
Explanation

Solution (a)

अजंता�म��तीथ���च��आकार�म��भ��ह�।��वहार�गुफा��म��से�कुछ�अधूरी�ह��, जैसे�गुफा�सं�या�5, 14, 23 24, 28 और�29 अजंता�के


मह�वपूण��संर�क��म��वराहदे व�(गुफा�सं�या�16 के�संर�क) , वाकाटक�राजा�, हरीसेना�के��धान�मं�ी�थे�; उपे��गु�त�(गुफा�सं�या�17–20
के�संर�क) इस��े��के��थानीय�राजा�और�वाकाटक�राजा�, ह�रसेना�के�सामंत�थे।�बु�भ��(गुफा�सं�या�26 के�संर�क) ; और�मथुरादास
(गुफा�सं�या�4 के�संर�क)।�गुफा�सं�या�1, 2, 16 और�17 म��कई�प��ट�ग�बच�गई�ह�।�

QUESTION 16.
�न�न�ल�खत�म��से�कौन�- से�मं�दर�म���शखर�और�पंचायतन�वा�तुकला�का��थम�उदाहरण��मलता�है�?

a) दे वगढ़�म��दशावतार�मं�दर�
b) राजगीर�म��म�नयार�मठ�मं�दर�
c) सांची�मं�दर�
d) खजुराहो�का�कंद�रया�महादे व�मं�दर�
Correct Answer: A
Your Answer:
Explanation

IASbaba
Score:
Web: http://ilp.iasbaba.com/
47.00 /
Email: ilp@iasbaba.com
Page 84 200
2019 - Test 28-
Exam Title :
History & C...
Email : narenips6@gmail.com
Contact :

Solution (a)

दशावतार�मं�दर: यह�उ�र��दे श�के�दे वगढ़�म����थत�है।�मं�दर��व�णु�को�सम�प�त�है।�यह�भारत�का�पहला�मं�दर�है��जसम���शखर�है�और


वा�तुकला�क��पंचायतन�शैली�है।�

एक�ऐसा��ह���मं�दर�एक�पंचायत�है�जब�मु�य�मं�दर�चार�सहायक�मं�दर��से��घरा�होता�है।�

QUESTION 17.
नागर�मं�दर�शैली�के�बारे�म���न�न�ल�खत�कथन��पर��वचार�क��जए�

1. पूरा�ढांचा�एक�उठे ��ए�मंच�पर��न�म�त��कया�गया�है�, �जसम��एक�चौकोर�आधार�के�साथ�बाहर�क��ओर���ेपण�होता�ह��

2. �शखर�, गभ�गृह�के�ऊपर�बना�एक�टॉवर�इन�मं�दर��क��मु�य��वशेषता�है�

3. इस�वा�तुकला�के�अ�धकतर�अवशेष�माम�लापुरम�और�कांचीपुरम�म��पाए�जाते�ह��

सही�कूट�का�चयन�क��जए�

a) केवल�1 तथा�2
b) केवल�2 तथा�3
c) केवल�1 तथा�3
d) उपरो��सभी�
Correct Answer: A
Your Answer:
Explanation

Solution (a)

नागर�शैली�के�मं�दर��क��सामा�य��वशेषता:

�थान: वा�तुकला�का�यह�समय�उ�री�भारत�म���हमालय�से�लेकर��व��य�तक�पाया�जाता�है।�

योजना: इसम��एक��ू�सफ़ॉम���ाउंड��लान�ह�।�पूरा�ढांचा�एक�ऊंचे�मंच�पर�बनाया�गया�है।�मं�दर�का�एक�चौकोर�आधार�है��जसके�बाहर�के��ह�से
को�रथ�कहा�जाता�है।�

म��दर�के�भाग�:

1) गभ�गृह: यह�मं�दर�का�गभ�गृह�है�जहां�मु�य�दे वता�क��मू�त��रखी�जाती�है।�

2) मंडपा: यह�गरबा�गृह�के�सामने�एक�मंच�है�जहाँ�भ���ाथ�ना�के��लए�इक�ा�हो�सकते�ह�।�

3) �शखर: यह�गभ�गृह�के�ऊपर��न�म�त�मु�य�मीनार�है।�वहग�व��य�है।�कई�मं�दर��म��, मंडप��म��सहायक��शखर�हो�सकते�ह�।�

4) अमलाक: यह��शखर�के�ऊपर�बना�एक�गोलाकार��नशान�है।�

5) कलश: यह��शखर�के�ऊपर�सबसे�अ�धक�संरचना�है।�ऐसा�लग�रहा�है�जैसे�अमलक�के�ऊपर�ना�रयल�का�कलश�रखा�गया�हो।�

6) जगती: यह�गभ�गृह�के�सामने�एक�उठा��आ�मंच�है�जहाँ�से�आरती�क��जाती�है।�

��वड़�वा�तुकला�के�अवशेष��यादातर�मम�लापुरम�और�कांचीपुरम�म��पाए�जाते�ह��

QUESTION 18.
�न�न�ल�खत�कथन��पर��वचार�क��जए�

IASbaba
Score:
Web: http://ilp.iasbaba.com/
47.00 /
Email: ilp@iasbaba.com
Page 85 200
2019 - Test 28-
Exam Title :
History & C...
Email : narenips6@gmail.com
Contact :

1. तंजावुर�म��बृहदे �र�मं�दर�राज���चोल��ारा��न�म�त��कया�गया�था�

2. �वजयनगर�राजा��ने�अ�भव�ध�त�उ�च��ांगण�द�वार��और�उन�पर�अ�धक�सजावट�क��अवधारणा�को�पेश��कया�

3. कृ�णदे व�राय�ने�मं�दर��म���वयं�तथा�अपनी�प��नय��क��यथाथ��आकार�क��मू�त�कार��को��था�पत��कया�

सही�कूट�का�चयन�क��जए�

a) केवल�1 तथा�2
b) केवल�2 तथा�3
c) केवल�1 तथा�3
d) उपरो��सभी�
Correct Answer: B
Your Answer:
Explanation

Solution (b)

चोल�राजा��के�शासनकाल�के�दौरान���वड़�मं�दर�अपने�कला�मक��शखर�पर�प�ँच�गए।�सबसे�बड़ा�उदाहरण�तंजावुर�म��बृहदे �र�मं�दर�ह��, जो
राजाराज��थम��ारा�लगभग�1010, और�गंगईक�डकोलापुरम�म��मं�दर�, उनके�बेटे�राज���चोल��ारा�लगभग�1025 म��बनाया�गया�था।�

�वजयनगर�शैली�:

1) उ�ह�ने�इन�ऊँची�प�र�ध�क��द�वार��पर�बढ़े ��ए�उ�च�बाड़े�क��द�वार��और�अ�धक�सजावट�क��अवधारणा�और�गोपुरम�क��शु�आत�क�।�

2) अलौ�कक�घोड़��क��मू�त�कला�या�आकृ�त�ब�त�बार�उपयोग�क��जाती�थी।�

3) उ�ह�ने�धम��नरपे��इमारत��(उदाहरण-लोटस�महल) क��अवधारणा�को�भी�पेश��कया।�

4) उ�ह�ने�मु�य�मं�दर�के�बा��ओर�एक�मंडप�जोड़ा��जसे�क�याण�मंडप�कहा�जाता�है।�इसका�उपयोग�दे वता��और�दे वी�के��ववाह�समारोह�


के��लए��कया�जाता�है।�

5) कृ�णदे व�राय�ने�मं�दर�म��उनके�और�उनक��प��नय��के�जीवन�आकार�के�मू�त�कार��को��था�पत��कया।�

QUESTION 19.
�न�न�ल�खत�कथन��पर��वचार�क��जए�

1. इस�मं�दर�क��वा�तुकला�के�तहत�, �तर��क��सं�या�को�बरकरार�रखते��ए�भी�मं�दर�क��मीनार��क��ऊंचाई�कम�कर�द��गई�थी।�

2. बौ��चै�य�क��अ�� -वृ�ाकार�संरचना��का�भी�इस�शैली�म���योग��कया�गया�ह��

3. इस�शैली�म��गभ�गृह�के�चार��ओर�बंद�प�र�मा�क��नह��होता�है�

यहां�मं�दर�वा�तुकला�क��कौन-सी�शैली�को�संद�भ�त��कया�जा�रहा�है�?

a) नागर�शैली�
b) वेसर�शैली�
c) ��वड़�शैली�
d) गो�थक�शैली�
Correct Answer: B
Your Answer:
Explanation

Solution (b)

IASbaba
Score:
Web: http://ilp.iasbaba.com/
47.00 /
Email: ilp@iasbaba.com
Page 86 200
2019 - Test 28-
Exam Title :
History & C...
Email : narenips6@gmail.com
Contact :

वेसर�शैली�

1) यह�नागरा�शैली�और���वड़�शैली�का�एक�समामेलन�है�और�म�ययुगीन�काल�के�दौरान�उभरा।�

2) क�याणी�और�होयसल�के�बाद�के�चालु�य��के�तहत�द�कन�म���न�म�त�मं�दर�इस�शैली�के�उदाहरण�माने�जाते�ह�।�

3) वेसरा�शैली�मं�दर�क��मीनार��क��ऊँचाई�कम�कर�दे ती�है�, जब�क��टयर�क��सं�या�को�बरकरार�रखा�जाता�है।�यह����गत��तर��क��ऊंचाई


को�कम�करके�पूरा��कया�गया�है।�

4) बौ��शैली�के�अध�-वृ�ाकार�ढांचे�को�भी�इस�शैली�म��उधार��लया�गया�है�, जैसे��क�आइहोल�के��गा��मं�दर�म�।�

5) इसम��दो��स�ांत�घटक�जैसे���वड़�शैली�यानी��वमना�और�मंडप�शा�मल�ह�।�

6) ��व�ड़यन�शैली�से���थान�करने�के��लए�यह�गभ�गृह�के�चार��ओर�ए�बुलेटरी�कवर�नह��करता�है।�

7) म�य�भारत�और�डे�कन�म��कई�मं�दर��ने��े�ीय�संशोधन��के�साथ�वेसारा�शैली�का�उपयोग��कया�है।��वशेष��प�से�प�ादकल�म����थत�कुछ
हद�तक�पापनाथ�मं�दर�( 680 ई�वी) और�इस�शैलीगत�ओवरलैप�के��लए��च�ण�

QUESTION 20.
�न�न�ल�खत�म��से�कौन-से�वा�तु�श�प��वशेषता��को�स�तनत�शासक���ारा��ार�भ��कया�गया�था�?

1. मेहराब( Arches )

2. गुंबद�

3. मीनार��

4. बीम�

5. को�क( Brackets )

सही�कूट�का�चयन�क��जए�

a) केवल�1,2 तथा�3
b) केवल�1,4 तथा�5
c) केवल�2,3 तथा�4
d) केवल�2,3 तथा�5
Correct Answer: A
Your Answer:
Explanation

Solution (a)

स�तनत�वा�तुकला�को�मोटे �तौर�पर�धम��नरपे��इमारत��और�धा�म�क�भवन�म��वग�कृत��कया�जा�सकता�है।�

धम��नरपे��इमारत��म���कल��, महल��, �तंभ��, सराय�और�हौज़�(कृ��म�झील�) शा�मल�ह��जब�क�धा�म�क�संरचना��म��म��जद�, मकबरे�, दर


गाह�और�मदरसे�शा�मल�ह�।�

सु�तान��ने��वतं���प�से�भारतीय�आ�क�टे�ट�, राज�म��ी�और���मक��को�रोजगार��दया।�उ�ह�ने�म�य�ए�शयाई�वा�तुकला�के��वचार��को�भी
पेश��कया।�इसने�स�तनत�वा�तुकला�और�इ�लामी�और�भारतीय�शैली�का�समामेलन��कया।�

अ�धकांश��ारं�भक�इमारत���पछली�संरचना��को�न��करने�और�उनसे�क�चे�माल�का�उपयोग�करने�के�बाद�बनाई�गई�थ�।�कहा�जाता�है��क
�द�ली�म���वात-उल-इ�लाम�म��जद�को�एक��ह���मं�दर�को��व�त�करके�कुतुब-उद-द�न�ऐबक��ारा�बनवाया�गया�था�, �जसे�जैन�मं�दर�के�ऊपर

IASbaba
Score:
Web: http://ilp.iasbaba.com/
47.00 /
Email: ilp@iasbaba.com
Page 87 200
2019 - Test 28-
Exam Title :
History & C...
Email : narenips6@gmail.com
Contact :
बनाया�गया�था।�इसी�तरह�built अदाई-द�न-का�झोपड़ा�' अजमेर�म��एक�म��जद�है�, �जसे�2 �दन��म��बनाया�गया�था�, जो�एक��ह���इमारत�के
खंडहर�म��आया�था।�

मेहराब�और�मीनार��को�स�तनत�शासक���ारा�पेश��कया�गया�था।��पछली��वदे शी�वा�तुकला�बीम�और�को�क�पर�आधा�रत�थी��जसे��ै �ड
’शैली�कहा�जाता�था।��सरी�ओर�, इ�ला�मक�आ�क�टे�चर�‘चापाकार’ था��जसने��र���थान�को�पाटने�के��लए�मेहराब��और�वा�ट��का
इ�तेमाल��कया�और�सपाट�छत��के�बजाय�सुंदर�गुंबद��के��नमा�ण�का�समथ�न��कया।�

QUESTION 21.
�न�न�ल�खत�म��से�कौन�सही�सुमे�लत�है�?

संरचना��वशेषता�

1. बलबन�का�मकबरा�A) पहला�वा�त�वक�आच��

2. जमात�खाना�म��जद�B) पहला�वै�ा�नक�गुंबद�

3. �सकंदर�लोधी�का�मकबरा�C) पहला�डबल�गुंबद�

सही�कूट�का�चयन�क��जए�

a) केवल�1 तथा�2
b) केवल�2 तथा�3
c) केवल�1 तथा�3
d) उपरो��सभी�
Correct Answer: D
Your Answer:
Explanation

Solution (d)

बलबन�का�मकबरा�- बलबन�ने�अपना�मकबरा�बनाया�था��जसे�पहला�स�चा�मेहराब�माना�जाता�है�

अला�उद�द�न��खलजी�ने��सरी�(�द�ली�म�) नामक�नया�शहर�बनाया�और�जमात�खाना�म��जद�का��नमा�ण��कया।�उ�ह�ने�अलाई�दरवाजा�भी
बनवाया।�अलाई�दरवाजा�, जमात�खाना�म��जद�का��वेश��ार�है।�यह�पहला�वै�ा�नक�गुंबद�है।�भवन�म��हॉस��शू�आक��का�उपयोग��कया�गया
था।�

लॉ�डस�का�काय�काल�ब�त���थर�था�और�इस�लए�उ�ह���नमा�ण�का�मौका��मला।�

मह�वपूण���वशेषताएं:

इमारत��एक�उभरे��ए��लेटफ़ॉम��पर�बनी��ई�थ��, �जसके�चार��ओर�बगीचे�थे।�यह��वग��के��वषय�क��नकल�करने�के��लए�बनाया�गया�था�, बाद


म��, मुगल��ने�अपनी�इमारत��म��उसी�अवधारणा�का�उपयोग��कया।�

क���को�अ�कोणीय�आकार�म��बनाया�गया�था।�इस��वशेषता�को�मुगल��ने�भी�अपनाया�था।�

पहली�बार��सकंदर�लोधी�के�मकबरे�म��डबल�गुंबद�पेश��कया�गया�था।�

QUESTION 22.
�न�न�ल�खत�कथन��पर��वचार�क��जए�

1. क़�तुब�मीनार�का��नमा�ण�स�पूण���प�से�लाल�बलुआ�प�थर�का�उपयोग�करके��कया�गया�था�

2. तुगलकाबाद��कला�स�पूण��तरह�से��ेनाइट�से�बना�था��य��क�यह�आसानी�से�उपल�ध�था�

सही�कूट�का�चयन�क��जए�

IASbaba
Score:
Web: http://ilp.iasbaba.com/
47.00 /
Email: ilp@iasbaba.com
Page 88 200
2019 - Test 28-
Exam Title :
History & C...
Email : narenips6@gmail.com
Contact :

a) केवल�1
b) केवल�2
c) 1 तथा�2 दोन��
d) इनम��से�कोई�नह��
Correct Answer: B
Your Answer:
Explanation

Solution (b)

कुतुब�मीनार�स�तनत�काल�क��संभवतः�सबसे�मह�वपूण��संरचना�है।�इसे�सूफ��संत�कुतुब�उद�द�न�ब��तयार�काक��के�स�मान�म��बनाया�गया
था।��नमा�ण�कुतुब�उद�द�न�ऐबक��ारा�शु���कया�गया�था�, अगली�तीन�मं�जल��इ�तुत�मश��ारा�बनाई�गई�थ��और�अं�तम�दो�मं�जल��को
�फरोज�शाह�तुगलक��ारा��फर�से�बनाया�गया�था�, जब�शीष��मं�जल�को��बजली��ारा�न��कर��दया�गया�था।�

मीनार�73 मी�ऊँची�है�और�5 मं�जला�है।�उनके�पास�सुंदर��या�मतीय��डजाइन�और�अरबी�ह�।�

��येक�मं�जल�पर�बालक�नय��को�फूल��और��या�मतीय��डजाइन��के�साथ�ज�टल��प�से�उकेरा�गया�है।�वे�भी�इस�तरह�से��डज़ाइन
�कए�गए�ह��जैसे��क�वे�अगली�मं�जल�को�पकड़�रहे�ह�।�

पहले�तीन�मं�जला�का��नमा�ण�लाल�बलुआ�प�थर��ारा��कया�गया�है�और�अं�तम�दो�संगमरमर�से�बने�ह�।�

घायस�उद�द�न�तुगलक�ने�तुगलकाबाद�के�नए�शहर�का��नमा�ण��कया।�इस�अव�ध�म��तुगलकाबाद��कला�बनाया�गया�था।�यह��ेनाइट�से�बना�है
�य��क�यह�आसानी�से�उपल�ध�था।�चूं�क��ेनाइट�क��इमारत�को�चमकाना�मु��कल�है�, इस�लए�भवन�क��बनावट�खुरदरी�है।�इस��कले�क�
चारद�वारी�सीधी�क��बजाय�ढलान�वाली�है।�

QUESTION 23.
मुगल�वा�तुकला�के�संदभ��म���न�न�ल�खत�कथन��पर��वचार�क��जए�

1. मुगल�वा�तुकला�क��सभी�संरचना��वषयव�तु�आधा�रत�थे�

2. मुग़ल�इमारत��इंडो�- गो�थक�शैली�का��म�ण�थ��

3. �ारंभ�म��संगमरमर�का�उपयोग�इसक��आसान�उपल�धता�के�कारण��कया�गया�था।�बाद�म��लाल�बलुआ�प�थर�का�बड़े�पैमाने�पर�उपयोग
�कया�गया�

सही�कूट�का�चयन�क��जए�

a) केवल�1
b) केवल�2 तथा�3
c) केवल�1 तथा�3
d) उपरो��सभी�
Correct Answer: A
Your Answer:
Explanation

Solution (a)

मुगल�वा�तुकला�क��मु�य��वशेषताएं�

1) मुगल�इमारत��इंडो�- इ�ला�मक�शैली�का��म�ण�थ�।�भारतीय�शै�लय��म��, राजपूत�और�बौ���भाव�अ�धक��भावी�ह�।�इ�लामी�शैली�म��, ईरा


नी�और�म�य�ए�शयाई�शैली��मुख�है।�

2) संरचनाएं�आम�तौर�पर�बड़ी�और�बुलंद�होती�ह��जो�सा�ा�य�के�धन�और�ताकत�को��द�श�त�करती�ह�।�

IASbaba
Score:
Web: http://ilp.iasbaba.com/
47.00 /
Email: ilp@iasbaba.com
Page 89 200
2019 - Test 28-
Exam Title :
History & C...
Email : narenips6@gmail.com
Contact :

3) राजा��डजाइन�और��नमा�ण�से����गत���च�लेते�थे।�इस�लए�आमतौर�पर�एक�क���कृत�योजना�बनाई�गई�थी��क��कन�कलाकार��को�बीच
म��बदलने�क��अनुम�त�नह��थी।�

4) खंभे�, गुंबद�, मेहराब�, ���वंच�और���लंथ�मु�य��वशेषताएं�थ�।�

5) सभी�इमारत��थीम�आधा�रत�थ�।�उ�ान�, �या�म�त�और��डजाइन�क��सम�पता�पर��वशेष�जोर��दया�गया�था।�

6) �ारंभ�म��लाल�बलुआ�प�थर�का�उपयोग�इसक��आसान�उपल�धता�के�कारण��कया�गया�था।�बाद�म��संगमरमर�था�

7) बड़े�पैमाने�पर�इ�तेमाल��कया।�चूने�का�उपयोग�बंधन�साम�ी�के��प�म���कया�जाता�था।�

8) सजावट�के��लए�क�मती�और�अ�� �क�मती�प�थर��का�इ�तेमाल��कया�गया�था।�उ�ह��ईरान�और�म�य�ए�शया�से�लाया�गया�था।�

9) द�वार��पर�सुलेख�काय��और�अरबी�का�उपयोग�अ�सर��कया�जाता�था।�ताजमहल�पर�एक�सबसे���स��है।�फारसी�शैली�के�दोहरे�गुंबद
(�याज�के�आकार�के) बनाए�गए�थे।�

QUESTION 24.
गंधार�मू�त�कला�के�बारे�म���न�न�ल�खत�कथन��पर��वचार�क��जए�

1. यह��ीक�कला�मक��भाव�और�मानव-दे वता�क��अवधारणा�को��द�श�त�करता�है�जो�अ�नवाय���प�से��ीक�पौरा�णक�सं�कृ�त�से��े�रत�था।�

2. गंधार�कला�म��मु�य��प�से�बौ���वषय��को�दशा�या�गया�है�

3. गंधार�मू�त�कला�को�कुषाण�शासक��का�उदार�संर�ण��ा�त�था�

सही�कूट�का�चयन�क��जए�

a) केवल�1
b) केवल�2 तथा�3
c) केवल�1 तथा�3
d) उपरो��सभी�
Correct Answer: D
Your Answer:
Explanation

Solution (d)

गांधार��कूल�ऑफ�आट� �ने�कुषाण�शासक���वशेष��प�से�क�न�क�के�उदार�संर�ण�का�आनंद��लया।�गंधार�कला�म��मु�य��प�से�बौ���वषय�
को�दशा�या�गया�है।�यह��ीक�कला�मक��भाव�को��द�श�त�करता�है�और�"मानव-दे वता" क��अवधारणा�अ�नवाय���प�से��ीक�पौरा�णक
सं�कृ�त�से��े�रत�थी।�कहा�जाता�है��क�गंधार��कूल�ऑफ��क�पचर�म��लहराते�बाल��, कंध��को�ढं कने�वाले�जूते�और�स�डल�आ�द�का�योगदान
है।�

QUESTION 25.
�न�न�ल�खत�कथन��पर��वचार�क��जए�

1. गुं�डचा�बंधु��ह���तानी�शा��ीय�संगीत�क����पद�शैली�के�अ�छे �जानकार�ह��

2. �यागराज�, शमशा��ी�और�मुथु�वामी�द���त�को�कना�टक�संगीत�के�"��मू�त�" के��प�म��जाना�जाता�है�

सही�कूट�का�चयन�क��जए�

a) केवल�1
b) केवल�2

IASbaba
Score:
Web: http://ilp.iasbaba.com/
47.00 /
Email: ilp@iasbaba.com
Page 90 200
2019 - Test 28-
Exam Title :
History & C...
Email : narenips6@gmail.com
Contact :

c) 1 तथा�2 दोन��
d) इनम��से�कोई�नह��
Correct Answer: C
Your Answer:
Explanation

Solution (c)

गुंडेचा�बंधु��ह���तानी�शा��ीय�संगीत�क����पद�शैली�के�अ�छे �जानकार�ह��

�ुपद�भारतीय�उपमहा��प�के��ह���तानी�शा��ीय�संगीत�म��एक�शैली�है।�यह��ह���तानी�शा��ीय�संगीत�, हवेली�संगीत�और�द��ण�भारतीय
कना�टक�परंपरा�से�संबं�धत�संगीत��मुख�मुखर�शै�लय��क��सबसे�पुरानी�शैली�है।�यह�श�द�क�वता�के�उस��प�और�उस�शैली�के�दोन���प��को
दशा�ता�है��जसम��इसे�गाया�जाता�है।�यह�आ�या��मक�, वीर�, �वचारशील�, सदाचारी�, अंत�न��हत�नै�तक��ान�या�गीत-संगीत�संयोजन�का
एकमा���प�है।�एक��ुपद�म��कम�से�कम�चार��ोक�ह��, �ज�ह��Sthayi ( या�Asthayi), Antara, Sancari और�Abhoga कहा
जाता�है।�

कना�टक�संगीत�या�कना�टक�संगीत�द��ण�भारतीय�शा��ीय�संगीत�है।�कना�टक�संगीत�का�एक�समृ��इ�तहास�और�परंपरा�है�और�यह��व�
संगीत�के�र�न��म��से�एक�है।�कना�टक�संगीत�द��ण�भारतीय�रा�य��त�मलनाडु �, केरल�, आं���दे श�और�कना�टक�म���वक�सत��आ�है।�ये�रा�य
��वड़�सं�कृ�त�क��अपनी�मजबूत���तु�त�के��लए�जाने�जाते�ह�।�प�नीतस�( 1480-1564) कना�टक�संगीत�के�जनक�माने�जाते�ह�।�उसे
कना�टक�संगीत�क��प��त�के�कोडकरण�का��ेय�जाता�है।�उ�ह��कई�हज़ार�गीत��के��नमा�ण�का��ेय�भी��दया�जाता�है।�कना�टक�संगीत�से�जुड़ा
एक�और�बड़ा�नाम�व�कट�मुखी��वामी�का�है।�उ�ह��कना�टक�संगीत�का�भ���स�ांतकार�माना�जाता�है।�उ�ह�ने�द��ण�भारतीय�राग��को�वग�कृत
करने�क���णाली�"मेलनकारा" भी��वक�सत�क�।�

यह�18 व��शता�द��म��था��क�कना�टक�संगीत�ने�अपना�वत�मान��व�प�हा�सल�कर��लया।�यह�वह�अव�ध�थी��जसम��कना�टक�संगीत�क�
"��मू�त�" दे खी�गई�थी�; �यागराज�, शमशा��ी�और�मुथु�वामी�द���त�अपनी���स��रचना��का�संकलन�करते�ह�।�

QUESTION 26.
�न�न�ल�खत�म��से�कौन���तीय�बु��के��प�म��पूजनीय�है�?

a) प�संभव�
b) सामंथाभ��
c) अवलो�कते�र�
d) व�पाणी�
Correct Answer: A
Your Answer:
Explanation

Solution (a)

प�संभव�को�गु���रनपोछे �के�नाम�से�भी�जाना�जाता�है�, जो�भारतीय�उपमहा��प�के�8 व��शता�द��के�बौ��गु��थे।�य��प�एक�ऐ�तहा�सक


प�संभव�था�, कम�ही�उसे���श�ग�ड् तेसन�के�कहने�पर�, साये�म���त�बत�म��पहले�बौ��मठ�के��नमा�ण�म��मदद�करने�के�अलावा�जाना�जाता�है�,
और�कुछ�ही�समय�बाद�राजदरबार�क��सा�ज़श��के�कारण��त�बत�छोड़��दया�गया।�

प�संभव�के�जीवन�और�कम��के�आसपास�कई��क�वदं �तयां�बढ़��ह��, और�उ�ह���त�बत�, नेपाल�, भूटान�, भारत�के��हमालयी�रा�य��, और�अ�य


जगह��पर��त�बती�बौ��धम��के�अनुया�यय���ारा�"�सरे�बु�" के��प�म���ापक��प�से�स�मा�नत��कया�जाता�है।�

�त�बती�बौ��धम��म��, वह�सा�ह�य�क��एक�शैली�क��एक��वशेषता�है��जसे�टमा��कहा�जाता�है�, अ�मताभ�का�एक�उ�रण�जो��रदश��मुठभेड़��म�


तृतीयक�और�गु��योग�अ�यास�का��यान�क���त�करने�के��लए�कहा�जाता�है�, �वशेष��प�से�चूने�वाले��कूल��म�।�द��न�ग�मा��कूल�प�संभव�को
अपनी�परंपरा�का�सं�थापक�मानता�है।�

QUESTION 27.
�न�न�ल�खत�कथन��पर��वचार�क��जए�

IASbaba
Score:
Web: http://ilp.iasbaba.com/
47.00 /
Email: ilp@iasbaba.com
Page 91 200
2019 - Test 28-
Exam Title :
History & C...
Email : narenips6@gmail.com
Contact :

1. अथव�वेद��व�भ��ब�लदान��के��दश�न�के��लए�अनु�ान��का�वण�न�करता�है�

2. भरत�का�ना�शा���ग�धव�वेद�पर�आधा�रत�है�जो��क�यजुव�द�का�एक�उपवेद�है�

3. यजुव�द�म���वन��लोक( humble folk ) क��लोक��य�मा�यता��और�अंध�व�ास��का�वण�न�है�

सही�कूट�का�चयन�क��जए�

a) केवल�1 तथा�2
b) केवल�2 तथा�3
c) केवल�1 तथा�3
d) इनम��से�कोई�नह��
Correct Answer: D
Your Answer:
Explanation

Solution (d)

" यजुस" का�अथ��है�"ब�लदान�का�सू�" और�यजुव�द�ब�लदान��क���ाथ�ना�है।�इसम��य���का�अनु�ान�होता�है।�अनुमान�है��क�इसक��रचना�1,


400 से�1000 ईसा�पूव��के�बीच��ई�होगी।�

यह��व�भ��ब�लदान��के��दश�न�के��लए�अनु�ान��का�वण�न�करता�है।�

गंधव�वेद�सैम�वेद�का�उपवेद�है�, संगीत�, नृ�य�और�नाटक�पर�एक�तकनीक���ंथ�है।�भरत�का�ना�शा���तो�गंधव�वेद�पर�आधा�रत�है�

अथव�-वेद�अ�य�तीन�वेद��से�पूरी�तरह�से�अलग�है�और�कालानु��मक��प�से�चार�म��से�अं�तम�है।�

यह�मह�वपूण��और��दलच�प�है��य��क�इसम���वन��लोक�क��लोक��य�मा�यता��और�अंध�व�ास��का�वण�न�है।�

अथव�वेद�म��जा��मं��शा�मल�ह��, �च�क�सा�और�जा��क���ारं�भक�परंपरा��को�शा�मल�करता�है�जो�अ�य�इंडो-यूरो�पयन�परी�ा��म��समान
ह�।�

QUESTION 28.
�न�न�ल�खत�कथन��पर��वचार�क��जए�

1. �श�ा�एक�वेदांग�है��जसका�उ�े �य�वै�दक�भजन��और�मं���के�सही�उ�चारण�का��श�ण�है�

2. �ौतसू��, जो��ु�त�पर�आधा�रत�ह��, महान�ब�लदान��के��दश�न�का��श�ण�ह��, �जसम��तीन�या�पाँच�ब�ल�क��आव�यकता�होती�है�

3. छं द�वेदांग�को�मापता�है�और�एक�छं द�म��वै�दक�मं���को�सं�या�के�आधार�पर��वभा�जत�करता�है�

सही�कूट�का�चयन�क��जए�

a) केवल�1 तथा�2
b) केवल�2 तथा�3
c) केवल�1 तथा�3
d) उपरो��सभी�
Correct Answer: D
Your Answer:
Explanation

Solution (d)

�मृ�त�सा�ह�य�म��वेदांग��क��पहली��ृंखला�है।�ये�वेद��के�अ�ययन�और�समझ�से�जुड़े�छह�सहायक��वषय��को�संद�भ�त�करते�ह�।�वे�इस��कार�ह�:

IASbaba
Score:
Web: http://ilp.iasbaba.com/
47.00 /
Email: ilp@iasbaba.com
Page 92 200
2019 - Test 28-
Exam Title :
History & C...
Email : narenips6@gmail.com
Contact :

1. �शखा�(�व�न-�व�ान) - इसका�उ�े �य�वै�दक�भजन��और�मं���के�सही�उ�चारण�का��श�ण�है।�

2. क�प�(अनु�ान�कैनन) - इसम��ब�ल��था�और��व��थत�सू��शा�मल�ह�।��ौतसू��, जो��ु�त�पर�आधा�रत�ह��, और�महान�ब�लदान��के


�दश�न�को��सखाते�ह��, �जसम��तीन�या�पांच�ब�ल�क��आव�यकता�होती�है।�

�माट� सू��, या��मृ�त�या�परंपरा�पर�आधा�रत��नयम।�

धम�सू��धम�शा���परंपरा�के�पहले�चार��ंथ�ह��और�वे�धम��के��वचार�पर��यान�क���त�करते�ह��, मु�य�माग�दश�क��जसके��ारा��ह���अपना�जीवन
जीने�का��यास�करते�ह�।�

3. �ाकरण�(�ाकरण) - �ाकरण�म��पा�ण�न�के�आ�यययी�शा�मल�ह�।�8 व��शता�द��ईसा�पूव��से�लेकर�ब�त��ारं�भक�भारतीय��ाकरण�के


अ�धकांश�काय��खो�गए�ह�।�

4. �न���(�प�ीकरण) - यह�परंपरागत��प�से�या�का�, एक��ाचीन�सं�कृत��ाकरण�के��लए��ज�मेदार�है।�यह��ु�प���से�संबं�धत�है�, �वशेष


�प�से�अ�प��श�द��म��, �वशेष��प�से�वेद�म��होने�वाली।�

5. छ�दा�(वै�दक�मीटर) - यह�एक�छं द�म��वै�दक�मं���क��सं�या�को�मापता�है�और�उ�ह��पेड��क��सं�या�से��वभा�जत�करता�है।�पाद��क��सं�या


��येक�पद�, भजन�, या�मं��को��वभा�जत�करती�है�और�श�दांश�क��सं�या���येक�पद�को��वभा�जत�करती�है।�

6. �यो�तष�(�यो�तष) - यह�सूय��और�चं�मा�क��ग�त�और�वै�दक��यो�तष�क��न�व�पर�नज़र�रखने�के��नयम��का�वण�न�करता�है।�

QUESTION 29.
�न�न�ल�खत�म��से�कौन�सही�सुमे�लत�है�?

कला��प��थान�

1. सौर�Saura A) अ�णाचल��दे श�

2. कालबे�लया�B) ओ�डशा�

3. द�म�दा�C) राज�थान�

4. �सला�बम�D) केरल�

सही�कूट�का�चयन�क��जए�

a) केवल�1 तथा�2
b) केवल�2 तथा�4
c) केवल�3 तथा�4
d) इनम��से�कोई�नह��
Correct Answer: D
Your Answer:
Explanation

Solution (d)

सौरा�ओ�डशा�म��सौरा�जनजा�त��ारा��च�लत��च�कला�का�एक��प�है�जो�वल��के�समान�है�ले�कन�द�घ��और�ल�बी�है।��ाकृ�तक�रंग��का
उपयोग��कया�जाता�है�

�सलंबम�त�मलनाडु �म���च�लत�माश�ल�आट� �का�एक��कार�है�- ह�थयार�आधा�रत�माश�ल�आट� ।�बाघ�, साँप�, बाज�के��प�और�पैर��के�काम�के


पैटन��का�पशु�आंदोलन��म��एक�मह�वपूण��भू�मका�है�; कु�टुवरसई�- �सलंबम�का��ह�सा�- �नह�था�सं�करण।�

IASbaba
Score:
Web: http://ilp.iasbaba.com/
47.00 /
Email: ilp@iasbaba.com
Page 93 200
2019 - Test 28-
Exam Title :
History & C...
Email : narenips6@gmail.com
Contact :

कालबे�लया�, राज�थानी�का�नृ�य��प�है�, �जसम��घूमने�वाले�, सुशो�भत�मूवम�ट�होते�ह��जो�इस�नृ�य�को��नहारने�के��लए��े�रत�करते�ह�।


म�हलाएं�नृ�य�करती�ह��जब�क�पु�ष��दश�न�के��लए�संगीत��दान�करते�ह�।�कालबे�लया�नृ�य�और�गीत�मानवता�के�अमूत��सां�कृ�तक��वरासत�क�
यूने�को�क����त�न�ध�सूची�का�एक��ह�सा�ह�।�संगीत�वा�यं��का�उपयोग�- पुंगी�- सपेर���ारा�इ�तेमाल��कया�जाने�वाला�

दा�म�डा�अ�णाचल��दे श�म��अपातानी�जनजा�त�का�एक�कला��प�है�

QUESTION 30.
�न�न�ल�खत�म��से�कौन�भारत�से�मानवता�क��अमूत��सां�कृ�तक��वरासत�क��यूने�को�क����त�न�ध�सूची�म��शा�मल�ह��?

1. कु�टया�म�, सं�कृत��थएटर�

2. मु�दये��, अनु�ान��थएटर�और�केरल�का�नृ�य�नाटक�

3. नवरौज़�

4. कुंभ�मेला�

5. कृ�णाअ�म�

6. संक�त�न�, अनु�ान�गायन�, म�णपुर�का�ढोलक�और�नृ�य�

सही�कूट�का�चयन�क��जए�

a) केवल�1,2,3 तथा�5
b) केवल�1,2,3,4 तथा�6
c) केवल�1,3,4,5 तथा�6
d) केवल�1,2,4,5 तथा�6
Correct Answer: B
Your Answer:
Explanation

Solution (b)

भारत�के�कुल�13 अमूत��सां�कृ�तक��वरासत�( ICH) त�व��को�मानवता�क��अमूत��सां�कृ�तक��वरासत�क��यूने�को�क����त�न�ध�सूची�म��आज


तक�अं�कत��कया�गया�है।�

अमूत��सां�कृ�तक��वरासत�क��यूने�को�क����त�न�ध�सूची�म��एक�त�व�को�शा�मल�करने�के��लए�, रा�य�दल��को�यूने�को�स�म�त�के�मू�यांकन
और�परी�ा�के��लए�संबं�धत�त�व�पर�नामांकन�डो�जयर���तुत�करना�आव�यक�है।�

सं�कृ�त�मं�ालय�ने�सं�कृ�त�मं�ालय�के�तहत�एक��वाय��संगठन�, संगीत�नाटक�अकादमी�को�, अमूत��सां�कृ�तक��वरासत�से�संबं�धत�मामल�


के��लए�नोडल�काया�लय�के��प�म���नयु���कया�है�, �जसम��यूने�को�क����त�न�ध�सूची�के��लए�नामांकन�डो�जयर�तैयार�करना�शा�मल�है।
संगीत�नाटक�अकादमी�(एसएनए) तदनुसार�नामांकन�के��लए�पहचाने�गए�त�व�के�संबंध�म��डो�जयर�को�अं�तम��प�दे ने�से�पहले��हतधारक��,
�वशेष���/ अ�धका�रय��आ�द�के�साथ�आव�यक�बातचीत�करता�है।�

Representative List of the Intangible Cultural Heritage of Humanity

ICH Element - Year of Inscription

मानवता�क��अमूत��सां�कृ�तक��वरासत�क����त�न�ध�सूची�

आईसीएच�त�व�- �शलालेख�का�वष��

1) वै�दक�जप�क��परंपरा�- 2008

IASbaba
Score:
Web: http://ilp.iasbaba.com/
47.00 /
Email: ilp@iasbaba.com
Page 94 200
2019 - Test 28-
Exam Title :
History & C...
Email : narenips6@gmail.com
Contact :

2) रामलीला�, रामायण�का�पारंप�रक�मंचन�- 2008

3) कु�टया�म�, सं�कृत��थयेटर�- 2008

4) रा�मम�गढ़वाल��हमालय�, भारत�का�धा�म�क�उ�सव�और�धा�म�क�रंगमंच, 2009

5) मु�दये�टु�, अनु�ान��थएटर�और�केरल�का�नृ�य�नाटक�- 2010

6) राज�थान�के�कालबे�लया�लोक�गीत�और�नृ�य�- 2010

7) ) छऊ�नृ�य�- 2010

8) ल�ाख�का�बौ��जप: �ांस��हमालयन�ल�ाख��े��, ज�मू�और�क�मीर�, भारत�म��प�व��बौ���ंथ��का�पाठ�- 2012

9) संक�त�न�, अनु�ान�, म�णपुर�का�गायन�, नृ�य�और�नृ�य�- 2013

10) 1 जं�डयाला�गु��, पंजाब�, भारत�के�थेथरस�के�बीच�बत�न�बनाने�का�पारंप�रक�पीतल�और�तांबे�का��श�प�- 2014

11) योग�- 2016

12) नवरोज़�, नोव�ज़�, नोवरोज़�, नाउरोज़�, नौरोज़�, नौरोज़�, नू�ज़�, नोव�ज़�, नव�ज़�, नेव�ज़�, नोवरोज़�, नवरोज़�- 2016

13) कुंभ�मेला�- 2017

QUESTION 31.
�न�न�ल�खत�म��से�कौन�सा�काय��अ�घोष�हेतु�उ�रदायी�है�?

1. बु��च�रत�

2. सा�रपु���वचन�

3. सौ�दरानंद�का��

4. �व�वासवद�ा�

सही�कूट�का�चयन�क��जए�

a) केवल�1,2 तथा�3
b) केवल�2,3 तथा�4
c) केवल�1,3 तथा�4
d) केवल�1,2 तथा�4
Correct Answer: A
Your Answer:
Explanation

Solution (a)

अ�घोष�(लगभग�80 – 150 AD ) को���नया�का�पहला�सं�कृत�नाटककार�माना�जाता�है।�वह�एक�ज�म�से��ा�ण�थे�, ले�कन�बाद�म��बौ�


�भ�ु�बन�गए।�

वह�कुषाण�राजा�क�न�क�के�दरबारी�क�व�थे।�

अ�घोष�ने�शा��ीय�सं�कृत�म���लखा�था।�उनक��कृ�त�, बु��चा�रता�एक�महाका��शैली�सं�कृत�कृ�त�है।�यह�मु�य��प�से�बु��के�जीवन�से
संबं�धत�है।�

IASbaba
Score:
Web: http://ilp.iasbaba.com/
47.00 /
Email: ilp@iasbaba.com
Page 95 200
2019 - Test 28-
Exam Title :
History & C...
Email : narenips6@gmail.com
Contact :

असवघोसा�ने�एक�सं�कृत�नाटक�"सा�रपु���वचन" भी��लखा�है�जो�बु��के��श�य�सा�रपु��या�सा�रपु��के�बारे�म��है।�

असवघोष�ने�सौ�दरानंदका��भी��लखा�, नंद�के��पांतरण�के��वषय�के�साथ�एक�का��क�वता�, बु��के�सौतेले�भाई�, ता�क�वे�मो��तक�प�ंच


सक�।�

QUESTION 32.
यह�एक�महाका�ा�मक�क�वता�है��जसम��17 �सरग�ह��, �जनम��से�केवल�8 को�ही�सा�ह��यक�काय��के��प�म���वीकार��कया�जाता�है।
राजकुमार�का�त�केय�है�और�यह��शव�और�पाव�ती�के�पु��का�त�केय�के�ज�म�को�संद�भ�त�करता�है�, �शव�को�जीतने�के��लए�अ�य�धक�तप�या�के
बाद�जो�पहले�से�ही�कामदे व�(�ेम�के�ई�र) को�जीता�था।�बाद�म��का�त�केय�ने�तारकासुर�रा�स�को�मार�डाला�, �जसे�आशीवा�द��दया�गया�था
�क�वह��शव�और�पाव�ती�के�पु��के�अलावा��कसी�और��ारा�नह��मारा�जाएगा।�

�न�न�ल�खत�म��से�कौन-सी�रचना�को�यहाँ�संद�भ�त��कया�जा�रहा�है�?

a) रघुवंशम्�
b) कुमारसंभवम्�
c) ऋतुसंहार�
d) मृ�छक�टकम्�
Correct Answer: B
Your Answer:
Explanation

Solution (b)

रघुवंश�- रघुवंश�एक�सं�कृत�महाका��क�वता�है�जो�भगवान�राम�क��रघु�वामा�क��वंशावली�क��एक�लंबी�( 19 सरगास) कथा�है�जो�राजा


�दलीप�से�अ��नवण��तक�शु��होती�है।�

कुमारसंभव�- कुमारसंभव�एक�महाका��क�वता�है��जसम��17 सरग�ह��, �जनम��से�केवल�8 को�ही�उनके�लेखन�के��प�म���वीकार��कया�जाता


है।�कुमारा�या�राजकुमार�का�त�केय�है�और�यह��शव�को�जीतने�के��लए��शव�और�पाव�ती�के�पु��का�त�केय�के�ज�म�को�संद�भ�त�करता�है�, जो
�शव�को�जीतने�के��लए�ब�त�पहले�तप�या�के�बाद�कामदे व�(भगवान�का��ेम) जीता�था।�

बाद�म��का�त�केय�ने�तारकासुर�रा�स�को�मार�डाला�, �जसे�आशीवा�द��दया�गया�था��क�वह�पु��ओस��शव�और�पाव�ती�के�अलावा��कसी�और�के
�ारा�नह��मारा�जाएगा।�

�रतसमारा�- �रतसमारा��फर�से�सं�कृत�म��एक��मनी�महाका��क�वता�है��जसम��6 सग��ह�।�इन�सरगा��म��6 सीज़न�(�रतु) अथा�त�, ���मा


(�ी�म) , वरशा�(रे�स) , शरद�(शरद�ऋतु) , हेमंत�(कूल) , �स�सरा�(शीतकालीन) और�वसंत�(बसंत) का�उ�लेख�है।�इसम��6 मौसम��म�
�े�मय��क��भावना��, भावना��और�अनुभव��का�उ�लेख��कया�गया�है।��रतसमशरा�को�का�लदास�का�सबसे�पहला�लेखन�माना�जाता�है।�

QUESTION 33.
चोल�क��कां�य�नटराज���तमा�के�बारे�म���न�न�ल�खत�कथन��पर��वचार�क��जए�

1. इस���तमा�म���शव�को�अपने�बाएं�पैर�से��वयं�को�संतु�लत�करते��ए�तथा�इसी�पैर�से� अक�मा�को�दबाते��ए��दखाया�गया�है�, जो��क�अ�ान


का��तीक�है�

2. उनक��चार��भुजाएँ�बाहर�क��ओर��नकली��ई�ह��और�मु�य�दा�हना�हाथ�अभय�मु�ा�म��है�

3. ऊपरी�दाएं�हाथ�म��डम��पकड़ा��आ�है�जो�अपने�पसंद�दा�संगीत�वा�यं��को�ताल�पर�रखता�है।�

सही�कूट�का�चयन�क��जए�

a) केवल�1 तथा�2
b) केवल�2 तथा�3
c) केवल�1 तथा�3

IASbaba
Score:
Web: http://ilp.iasbaba.com/
47.00 /
Email: ilp@iasbaba.com
Page 96 200
2019 - Test 28-
Exam Title :
History & C...
Email : narenips6@gmail.com
Contact :

d) उपरो��सभी�
Correct Answer: B
Your Answer:
Explanation

Solution (b)

�शव�लौ�कक���नया�के�अंत�के�साथ�जुड़ा��आ�है��जसके�साथ�यह�नृ�य�क����थ�त�जुड़ी��ई�है।�

इस�चोल�काल�क��कां�य�मू�त�कला�म��वे�अपने�दा�हने�पैर�म��खुद�को�संतु�लत�करते��ए��दखाई�दे ते�ह��और�उसी�पैर�के�पैर�के�साथ�अ�सरा�, अज्


ञानता�या��व�मृ�त�के�दानव�को�दबाते�ह�।�

उसी�समय�वह�भुजंग��सता�के��ख�म��अपना�बायाँ�पैर�उठाता�है�, जो��तरोभाव�का���त�न�ध�व�करता�है�, जो�भ��के�मन�से�माया�या��म�को


�र�कर�रहा�है।�

उसक��चार�भुजाएँ��नकली��ई�ह��और�मु�य�दा�हना�हाथ�अभय�ह�ते�या�इशारे�से�संकेत�दे ने�वाला�है।�

ऊपरी�दायां�डम��को�अपने�पसंद�दा�संगीत�वा�यं��को�ताल�ताल�पर�रखने�के��लए�रखता�है।�

ऊपरी�बाएँ�हाथ�म���वाला�होती�है�, जब�क�मु�य�बाएँ�हाथ�को�डोला�ह�ता�म��रखा�जाता�है�और�दा�हने�हाथ�के�अभय�ह�ते�से�जोड़ता�है।�

उसके�बाल��के�ताले�गोलाकार��वाला�माला�या�आग�क��लपट��को�छू ते��ए�दोन��तरफ�से�उड़ते�ह��, जो�पूरे�नाचते��ए�को�घेरे�रहते�ह�।�

QUESTION 34.
एलोरा�क��गुफा��क���न�न�ल�खत�म��से�कौन�सी��वशेषताएँ�अ��तीय�ह��?

1. इसम��तीन�धम��से�जुड़े�मठ�ह��

2. एक�ही��थान�पर�कई�शै�लय��का�संगम�है�

3. तीन�मं�जला�गुफाएँ�है�

4. एकल�च�ान�को�काटकर�मं�दर�का��नमा�ण��कया�गया�है�

सही�कूट�का�चयन�क��जए�

a) केवल�1,2 तथा�3
b) केवल�2,3 तथा�4
c) केवल�1,3 तथा�4
d) उपरो��सभी�
Correct Answer: D
Your Answer:
Explanation

Solution (d)

एलोरा�अजंता�से�सौ��कलोमीटर�क���री�पर���थत�है�और�इसम��चौबीस�बौ��, �ा�ण�और�जैन�गुफाएँ�ह�।�

यह�दे श�म��एक�अ��तीय�कला-ऐ�तहा�सक��थल�है��य��क�इसम��पांचव��शता�द��से�लेकर��यारहव��शता�द��तक�के�तीन�धम��से�जुड़े�मठ�ह�।�

यह�शैलीगत�पा�र��थ�तकवाद�के�संदभ��म��भी�अ��तीय�है�, अथा�त�, एक�ही��थान�पर�कई�शै�लय��का�संगम।�एलोरा�और�औरंगाबाद�क��गुफाएँ


दो�धम��- बौ��धम��और��ा�णवाद��के�बीच�चल�रहे�मतभेद��को�दशा�ती�ह�।�व�ायण�बौ��धम��से�संबं�धत�बारह�बौ��गुफाएँ�ह��, जैसे�तारा�, म
हामायुरी�, अ�ो�य�, अवलो�कते�र�, मै�य�, अ�मताभ�, इ�या�द।�बौ��गुफाएँ�अजंता�ने�भी�दो�मं�जला�गुफा��क��खुदाई�क��है�, ले�कन
एलोरा�म��, ��पल�मं�जल�एक�अनूठ��उपल��ध�है।�सभी�गुफा��को�समतल�और��च��त��कया�गया�था�ले�कन�कुछ�भी��दखाई�नह��दे ता�है।�

IASbaba
Score:
Web: http://ilp.iasbaba.com/
47.00 /
Email: ilp@iasbaba.com
Page 97 200
2019 - Test 28-
Exam Title :
History & C...
Email : narenips6@gmail.com
Contact :

गुफा�सं�या�16 को�कैलाश�लेन�के�नाम�से�जाना�जाता�है।�एक�रॉक-कट�मं�दर�को�एकल�च�ान�से�उकेरा�गया�है�, जो�कारीगर��क��एक�अनूठ�


उपल��ध�है।�

QUESTION 35.
मो�हनीअ�म�के�बारे�म���न�न�ल�खत�कथन��पर��वचार�क��जए�

1. यह�म�हला�कलाकार���ारा��कया�जाने�वाला�एक�एकल�नृ�य�है�, �जसम��गीत�म�ण�वाल�म��होता�है�जो�सं�कृत�और�मलयालम�भाषा�का
�म�ण�है।�

2. परंपरागत��प�से�मो�हनीअ�म�के��दश�न��क��सूची�म��दो��दश�न��े�णयां�' नृ��' और�' नृ�य�' ह�।�

3. यह�नृ�य�के�ला�य��कार�का�अनुसरण�करता�है�जो�नृ�य�के�अ�धक�सुंदर�, कोमल�और���ी��प�को��द�श�त�करता�है�और�‘एकाहाय�
अ�भनव’ �प�म��उ�कृ�ता��द�श�त�करता�है।�

सही�कूट�का�चयन�क��जए�

a) केवल�1 तथा�2
b) केवल�2 तथा�3
c) केवल�1 तथा�3
d) उपरो��सभी�
Correct Answer: D
Your Answer:
Explanation

Solution (d)

मो�हनीअ�म�या�मो�हनीअ�म�एक�भारतीय�शा��ीय�नृ�य��प�है�, जो�भारत�के�केरल�रा�य�म���वक�सत��आ�है�, और�रा�य�क��दो�लोक��य


नृ�य�कला��म���गना�जाता�है�, �सरा�कथकली�है।�य��प�इसक��जड़��ना��शा���नाम�क��कला�पर�आधा�रत�पुराने��ह���पाठ�पर�आधा�रत�ह��,
अ�य�भारतीय�शा��ीय�नृ�य��प��के�समान�, मो�हनीअ�म�ला�य��कार�का�पालन�करता�है�जो�नृ�य�के�अ�धक�सुंदर�, कोमल�और���ी��प�को
�द�श�त�करता�है।�मो�हनीअ�म�का�नाम�' मो�हनी�' श�द�से��लया�गया�है�, जो�भगवान��व�णु�का�एक�अवतार�है।�पारंप�रक��प�से�म�हला
कलाकार���ारा��कया�जाने�वाला�एकल�नृ�य�, यह�नृ�य�और�गायन�के�मा�यम�से�एक�नाटक�को��द�श�त�करता�है�जहां�गीत�म�ण�वाल�म��पूरी
तरह�से�होता�है�जो�सं�कृत�और�मलयालम�भाषा�का��म�ण�है�और�पाठ�या�तो��वयं�नत�क��ारा�या�गायक��ारा��कया�जा�सकता�है।�संगीत�शैली
कना�टक�है।�

परंपरागत��प�से�मो�हनीअ�म�के��दश�न��क��सूची�म��दो��दश�न��े�णयां�ह��, �जनका�नाम�' नृ�य�' और�' नृ�य�' ' ना��शा���' म��बताया�गया


है।�यह�नृ�य�के�ला�सया��कार�का�अनुसरण�करता�है�जो�नृ�य�के�अ�धक�सुंदर�, सौ�य�और���ी��प�को�दशा�ता�है�और�अ�य�श�द��म��एकल�और
अ�भ�ंजक�नृ�य�कला�के��प�म��संगीत�और�गीत��के�साथ��शंसा�म��in एकहारा�अ�भनव�’�प�म��उ�कृ�ता��द�श�त�करता�है।�

QUESTION 36.
�न�न�ल�खत�कथन��पर��वचार�क��जए।�

1. नृ�य�के�इस��ा�प�म��वै�णव�एवं��शव, सूय��व�श���आ�द��ह���दे वी-दे वता��से�जुड़े��वषय�शा�मल�ह�।�

2. ‘ना��शा��’ म��बताए�गई�सभी�108 मौ�लक�इकाईयाँ�इस�कला��प�से��मलती-जुलती�ह�।�

3. इस�नृ�य��ा�प�म��द��ण�और�उ�र�दोन��के�भारतीय�राग�शा�मल�ह���जससे�भारत�के�इन�दो�भाग��के�बीच�अवधारणा��एवं�अ�भनय�के
�व�नमय�का�संकेत��मलता�है।�

�न�न�ल�खत�म��से�कौन�से�नृ�य��ा�प�का�यहाँ�संदभ���दया�जा�रहा�है�?

a) कु�चपुड़ी�
b) म�णपुरी�

IASbaba
Score:
Web: http://ilp.iasbaba.com/
47.00 /
Email: ilp@iasbaba.com
Page 98 200
2019 - Test 28-
Exam Title :
History & C...
Email : narenips6@gmail.com
Contact :

c) ओ�डसी�
d) क�थक�
Correct Answer: C
Your Answer:
Explanation

Solution (c)

ओ�डसी�भारत�के��े��शा��ीय�नृ�य�का�एक��कार�है��जसका�उ�म�भारत�म��पूव��तट�पर�ओ�डशा�रा�य�के��ह���मं�दर��म���आ�ह�।�इसका
सै�ां�तक�आधार�‘ना��शा��’ (जो��क��न�पादन�कला��पर�र�चत��ाचीन�सं�कृत��ह����ंथ�है) म��खोजा�जा�सकता�है।�ओ�डसी�क��वष�
पुरानी�परंपरा�ओ�डशा�के��ह���मं�दर��तथा��ह���धम�, जैन�धम��व�बौ��धम��से�जुड़े��व�भ��पुराता��वक�मह�व�के�अनेक��थल��म���दखाई�दे ती�थी,
�जनक��मू�त�याँ�इस�कला��प�क��मु�ा��को�सुशो�भत�करती�ह�।�यह��मथक�य�एवं�धा�म�क�कथा��के��ा�या�मक��क�स�, भ���क�वता�
एवं�नृतक��ारा�अपनी��व�श��शारी�रक�ग�त�व�धय���ारा�उभारे�गए�आ�या��मक��वचार�, भाव�, �भावी�संकेत��और�सांके�तक�भाषा��का
एक��कार�है�तथा�इसके��न�पादन�म��आ�ान, नरीता, नृ�य, ना��व�मो��शा�मल�ह��।�नृ�य�के�इस��ा�प�म��वै�णव�एवं��शव, सूय��व�श��
आ�द��ह���दे वी-दे वता��से�जुड़े��वषय�शा�मल�ह�।�

इस�नृ�य��ा�प�क���ाचीनता�इसक��जड़��से��प��है��ज�ह���ाचीन�सं�कृत��ह����ंथ�‘ना��शा��’ म��दे खा�जा�सकता�है�जो��व�भ���न�पादन


कला��से�संबं�धत�है।�‘ना��शा��’ म��बताए�गई�सभी�108 मौ�लक�इकाईयाँ�इस�कला��प�से��मलती-जुलती�ह�।�इसम��हजार��प��ह���ज�ह�
�व�भ��अ�याय��म��रखा�गया�है।�

इस�नृ�य��ा�प�क���व�श���वशेषता�यह�है��क�इसम��द��ण�और�उ�र�दोन��के�भारतीय�राग�शा�मल�ह���जससे�भारत�के�इन�दो�भाग��के�बीच
अवधारणा��एवं�अ�भनय�के��व�नमय�का�संकेत��मलता�है।�‘ शोकाबरद��’, ‘ कन��’, ‘ भैरवी�’, ‘ धन�ी�’, ‘ पंचम�’, ‘ �ी�गौड़�’, ‘ नट�’, ‘
बरद��’ तथा�‘ क�याण�’ ओ�डसी�के�मु�य�राग�ह�।�इसके�संगीत�वा�यं�ो�म��तबला, पखावज, हारमो�नयम, झांझ, वाय�लन, बंसी, �सतार�तथा
�वण�मंडल�शा�मल�है�।

QUESTION 37.
�न�न�ल�खत�कथन��पर��वचार�क��जए।�

1. अंगकोरवाट�मं�दर�का��नमा�ण�12 व��सद��म��थाईल�ड�म���आ�था।�

2. यह�वह�अकेला�सबसे�बड़ा��व��धरोहर��थल�है, �जसका��ड�जटल�खाका�ख�चा�गया�है।�

3. यह�भगवान��शव�को�सम�प�त�है।�

4. इसे�खमेर�वंश�के�राजा�सूय�वम�न�II �ारा�बनवाया�गया�था।�

उपरो��म��से�कौन�से�कथन�स�य�ह��:

a) उपयु���सभी�
b) 1,2 और�3
c) 2 और�4
d) 1 और�3
Correct Answer: C
Your Answer:
Explanation

Solution (c)

अंगकोरवाट�मं�दर�का��नमा�ण�खमेर�वंश�के�राजा�सूय�वम�न�II �ारा�12 व��सद��म��क�बो�डया�(क�बोज) म��करवाया�गया�था।�यह�भगवान


�व�णु�को�सम�प�त�है।�

IASbaba
Score:
Web: http://ilp.iasbaba.com/
47.00 /
Email: ilp@iasbaba.com
Page 99 200
2019 - Test 28-
Exam Title :
History & C...
Email : narenips6@gmail.com
Contact :

गूगल����ट��ू�अंगकोरवाट�मं�दर�क��90000 से�अ�धक�इमेजेज�दशा�ती�है�।�आप�घर�बैठे�इसका�360 �ड�ी�दश�न�कर�सकते�ह��( माउंट


�फजी�तथा�ताजमहल�के��लए�भी�यह�उपल�ध�है�) ।�यह�अब�तक�का�अकेला�सबसे�बड़ा��व��धरोहर��थल�है, �जसका��ड�जटल�खाका�ख�चा
गया�है।�

QUESTION 38.
‘ भागवतवाद’ के�बारे�म���न�न�ल�खत�कथन��पर��वचार�क��जए�:

1. यह��व�णु�क��पूजा�के�इद� �गद� �संके��त�था�।

2. इसने�मू�त��पूजा�को�हतो�सा�हत��कया�।

3. इसने�अवतारवाद�के��स�ांत�का��चार��कया�।

4. इसने�पशु��क��ब�ल�को�बढ़ावा��दया�।

सही��वक�प�चु�नए�:

a) 1 और�3
b) 1,2 और�3
c) 2,3 और�4
d) उपयु���सभी�
Correct Answer: A
Your Answer:
Explanation

Solution (a)

· भागवतवाद��व�णु�अथवा�भागवत�क��पूजा�के�इद� �गद� �संके��त�था�तथा�मौय��र�काल�म��उ��मत��आ�।�उ�ह��भगवत�तथा�उनके�अनुया�यय�


को�भागवत�कहा�जाता�था�।

· भागवतवाद�म��भ���और�अं�हसा�क���धानता�थी�।�भ���का�आशय�है��ेमपूण��समप�ण।�अ�ह�सा�अथवा�पशु��को�न�मारने�का��स�ांत�कृ�ष
समाज�के��लए�उपयु��था।�पशु��को�मारने�से��र�रहने�के�कारण�उनम��से�कई�केवल�शाकाहारी�भोजन�लेते�थे।�

· इसने�अवतारवाद�के��स�ांत�का��चार��कया�। इ�तहास�को��व�णु�के�दस�अवतार��के�एक�च��के��प�म����तुत��कया�गया�। यह�माना�गया


�क�जब�कभी�सामा�जक��व�था�पर�संकट�आया, �व�णु�एक�उ�चत��प�म��इसे�बचाने�के��लए��कट��ए�।

· भागवतवाद�के�प�ात�मू�त�पूजा�एक�आम��वशेषता�बन�गई�।�आधु�नक�तरीके�के�मं�दर�गु�त�काल�म��बनने�आरंभ��ए।�

QUESTION 39.
�न�न�ल�खत�कथन��पर��वचार�क��जए।�

1. ' माउंटेन�इकोज��लटरेचर�फे��टवल�' के�2018 वाले�सं�करण�का�आयोजन��हमाचल��दे श�म���आ।�

2. इंटरनेशनल�वाटर�कलर�फे��टवल�, 2018 का�आयोजन�रांची�म���आ।�

सही�कूट�चु�नए�

a) केवल�1
b) केवल�2
c) 1 और�2 दोन��
d) उपरो��म��से�कोई�नह��
Correct Answer: B
Your Answer:

IASbaba
Score:
Web: http://ilp.iasbaba.com/
47.00 /
Email: ilp@iasbaba.com
Page 100 200
2019 - Test 28-
Exam Title :
History & C...
Email : narenips6@gmail.com
Contact :
Explanation

Solution (b)

भूटान�म��, ' माउंटेन�इकोज��लटरेचर�फे��टवल�' के�नौव��वाले�सं�करण�का�आयोजन��थ�पू�म���कया�गया।�यह�उ�सव�इं�डया-भूटान�फाउंडेशन


और�अ�णी�सा�ह��यक�एज�सी��याही�क��एक�पहल�है।�उ�सव�का�2018 का�सं�करण�भारत�और�भूटान�के�बीच�राजन�यक�संबंध��के�50 वर्
ष��को�मनाता�है।�

इंटरनेशनल�वाटर�कलर�फे��टवल�, 2018 का�आयोजन�रांची, झारख�ड�म���आ।�इसे�पय�टन, कला-सं�कृ�त, खेल�एवं�युवा�मामले��वभाग


�ारा�आयो�जत��कया�गया।�

इंटरनेशनल�वाटर�कलर�फे��टवल�का�उ�े �य��थानीय�कलाकार��को�अंतरा��ीय�कला�के�साथ�जोड़ना�है�और�झारखंड�क��समृ��कला�और
सं�कृ�त�को�बढ़ावा�दे ना�है।�

QUESTION 40.
भारतीय�दश�न�के�बारे�म���न�न�ल�खत�कथन��पर��वचार�क��जए।�

1. सां�य�दश�न�के�अनुयायी�मानते�ह���क�स�चे��ान�क���ा��त�ही��ख��से�छु टकारे�का�एकमा��उपाय�है।�

2. �याय�दश�न�का�मानना�है��क�वा�त�वकता�म��सब�कुछ�पु�ष�और��कृ�त�से��नकला�है�और�मु���यह�समझने�म���न�हत�है��क�आ�मा�और
पदाथ��क��चाह�एक�ह�।�

3. पूव��मीमांसा�दश�न�वेद��म���न�व�वाद�आ�था�तथा���ांड�क��सभी�ग�त�व�धय��को�बनाए�रखने�हेतु��नय�मत��प�से�वै�दक�अ��न-आ��तय��क�
बात�करता�है।�

सही�कूट�चु�नए�

a) केवल�1 और�2
b) केवल�2 और�3
c) केवल�3
d) केवल�1 और�3
Correct Answer: C
Your Answer:
Explanation

Solution (c)

भारतीय�दश�न�दाश��नक��वचार�क��उन�कई�पर�परा��म��से�एक�को�संद�भ�त�करता�है��जनका�उ�म�भारतीय�उपमहा��प�म���आ।�इनम���ह��
दश�न, बौ��दश�न�और�जैन�दश�न�शा�मल�है।�

सां�य���ढ़वाद��दाश��नक��णा�लय��म��सबसे��ाचीन�है�और�मानता�है��क�वा�त�वकता�म��सब�कुछ�पु�ष�(�व�अथवा�आ�मा�अथवा�म��त�क)
और��कृ�त�(पदाथ�, सृजक�उजा�, उजा�) से��नकला�है।�यह�एक��ै त�दश�न�है�और��व�एवं�पदाथ��के�बीच�है�न��क�म��त�क�व�शरीर�के�बीच�जैसा
�क�प��मी��ै त�परंपरा�म��होता�है।�इसके�अंतग�त�मु���यह�समझने�म���न�हत�है��क�आ�मा�और�पदाथ��क��चाह�(��थरता, ग�त�व�ध�एवं�मंदता)
अलग-अलग�ह�।�

�याय�दश�न��याय�सू��पर�आधा�रत�है, �जसे��सरी�शता�द��ईसा�पूव��अ�पाद�गौतम��ारा��लखा�गया�था�।�इसक��काय��णाली�तक��क��एक
�णाली�पर�आधा�रत�है��जसे�बाद�म��अ�धकांश�भारतीय��वचार��म��लगभग�उसी��कार�अपना��लया�गया, जैसे�अर�तु�के�तक��ने�प��मी�दश�न
को��भा�वत��कया�है।�इसके�अनुया�यय��का�मानना�है��क�स�चे��ान�(�जसके�चार���ोत�ह�- ���कोण, �न�कष�, तुलना�और�गवाही) क���ा��त
ही��ख��से�छु टकारे�का�एकमा��उपाय�है।��याय�ने��व�भ��कसौ�टयां��वक�सत�क���जनके��ारा��ा�त��ान�स�चा�अथवा�झूठा�(एक�हद�तक
प��मी��व�े�ण�दश�न�जैसा) माना�जाता�था।�

पूव��मीमांसा�दश�न�का�मु�य�उ�े �य�वेद��के��ा�धकार�क���ा�या�और�उसे��था�पत�करना�है।�यह�वेद��म���न�व�वाद�आ�था�तथा���ांड�क��सभी
ग�त�व�धय��को�बनाए�रखने�हेतु��नय�मत��प�से�वै�दक�अ��न-आ��तय��क��बात�करता�है।�य��प�मीमांसा�अ�य�दश�न��क��ता�क�क�एवं�दाश��नक
�श�ा��को�सामा�यत: �वीकार�करता�है, तथा�प�यह�आ�ह�करता�है��क�मो��केवल�वेद��म��बताए�माग��पर�अमल�करने�से�ही��ा�त��कया�जा

IASbaba
Score:
Web: http://ilp.iasbaba.com/
47.00 /
Email: ilp@iasbaba.com
Page 101 200
2019 - Test 28-
Exam Title :
History & C...
Email : narenips6@gmail.com
Contact :
सकता�है।�इस�दश�न�ने�आगे�चलकर�अपना�मत�बदलना�आरंभ�कर��दया�और�यह��ा�ण�और��वतं�ता�के��स�ांत�को��सखाने�लगा, �जसम�
�बु��ग�त�व�ध�के�मा�यम�से�आ�मा�क��बाधा��से�मु���क��अनुम�त�है।�

QUESTION 41.
�वासी�भारतीय��दवस�के�आयोजन�क���त�थ�बदलकर��याग�कुंभ�मेले�के�साथ�कर�द��गई�थी।��न�न�ल�खत�म��से��कस��थान�पर�कुंभ�मेले�का
आयोजन�नह���कया�जाता�?

a) ह�र�ार�
b) ना�सक�
c) उ�जैन�
d) पुरी�
Correct Answer: D
Your Answer:
Explanation

Solution (d)

कुंभ�मेला�एक��वशाल��ह���तीथ�या�ा�है��जसम���ह���एक�प�व��नद��म���नान�हेतु�एक��होते�ह�।�

पारंप�रक��प�से�, चार�मेल��को��मुखत: कुंभ�मेले�के�तौर�पर�मा�यता�द��जाती�है�:, �यागराज�कुंभ�मेला, ह�र�ार�कुंभ�मेला�, ना�सक-


���बके�र��स�ह�थ�, तथा�उ�जैन��स�ह�थ।�

ये�चार�मेले�आव�धक��प�से�अ��ल�खत�म��से��कसी�एक��थान�पर�च��य�आधार�पर�आयो�जत��कए�जाते�ह��: इलाहाबाद�( �यागराज�), ह�रद्


वार�, ना�सक��जला�( ना�सक�और����बक�), तथा�उ�जैन।�मेले�का�मु�य��थल�एक�नद��के��कनारे�अव��थत�है�: ह�र�ार�म��गंगा�; गंगा, यमुना
और�अ��य�सर�वती�का�संगम�इलाहाबाद�म��; ना�सक�म��गोदावरी�; तथा�उ�जैन�म�����ा।�इन�न�दय��म���नान�से�एक�����के�सभी�पाप��के
धुल�जाने�क��मा�यता�है।�

�कसी�भी��थान�पर�कुंभ�मेला���त�12 वष��आयो�जत�होता�है।�ह�र�ार�और�ना�सक�म��होने�वाले�कुंभ�मेले�के�बीच�लगभग�3 वष��का�अंतर


रहता�है।�ना�सक�और�उ�जैन�के�मेले�एक�वष��या�एक�वष��के�अंतराल�पर�आते�ह�।�इसक��सट�क��त�थ�का��नधा�रण��व�म�संवत�तथा��यो�तष
के��नयम��के�अनुसार�बृह�प�त, सूय��एवं�चं�मा�क��रा�शगत�अव�था�के�अनुसार��कया�जाता�है।�

QUESTION 42.
�न�न�ल�खत�म��से��कस�नेता�को�रहबर-ए-आजम�क��उपा�ध��मली�?

a) सर�सैयद�अहमद�खां�
b) सर�छोटू �राम�
c) बाबा�रामचं��
d) द�न�बंधु��म��
Correct Answer: B
Your Answer:
Explanation

Solution (b)

�धानमं�ी�नर���मोद��ने�जाट�नेता�सर�छोटू राम�(1881-1945) क����तमा�का�अनावरण�ह�रयाणा�म���कया।�

उनके�सुधार�काय��

· �वभाजन�पूव��क��पंजाब��वधानप�रषद�के�सद�य�के�तौर�पर�उनक��पहली��मुख�उपल��ध�पंजाब�ल�ड�रेवे�यु�(अम�डम�ट) ए�ट�, 1929 का


पा�रत�होना�थी�जो�आज�तक�भी�एक�मह�वपूण��सामा�जक��वधान�माना�जाता�है।�

· सा�कार���ारा�का�तकार��का�शोषण��व�भ��उपाय��के�मा�यम�से�समा�त��कया�गया��जसक��शु�आत�पंजाब�रेगुलेशन�ऑफ़�एकाउंट्स�ए�ट�
, 1930 से��ई।�

IASbaba
Score:
Web: http://ilp.iasbaba.com/
47.00 /
Email: ilp@iasbaba.com
Page 102 200
2019 - Test 28-
Exam Title :
History & C...
Email : narenips6@gmail.com
Contact :

· इसके�प�ात�पंजाब�डेटस���ोटे �शन�ए�ट�1936 तथा�पंजाब��रलीफ�ऑफ़�इ�डे�टड् �नस�ए�ट�, 1943 आया।�

· पंजाब�ए�ीक�चरल��ो�ूस�माक�ट् स�ए�ट�1939 म��पास��कया�गया�था, �जसे�मंडी�ए�ट�के�लोक��य�नाम�से�जाना�जाता�था।�इसने


अ�धसू�चत��े���म��मा�क�ट�कमे�टय��के�गठन�का��ावधान��कया�तथा�कृषक��को�शोषण�से�मु����दलाने�म��मदद�क�।�

· जोत��क��चकबंद��का�काय��कंसो�लडेशन�ऑफ़�ल�ड�हो��डंग�ए�ट�, 1936 के�बाद�शु���आ��जसे�1945 म��संशो�धत��कया�गया।�न�केवल


ये�सभी�क़ानून�पा�रत��कए�गए, ब��क�छोटू �राम�ने�इनके�काया��वयन�को�भी�सु�न��त��कया।�

· आभारी�कृषक��ने�उ�ह��द�नबंधु�तथा�रहबर-ए-आजम�क��उपा�ध�से�नवाजा।�अं�ेज��ने�A 1937 म��उ�ह��नाईट�ड��दान��कया।�

QUESTION 43.
महारा��के�क�कण��े��के�र�ना�गरी�एवं�राजापुर��जल��और�उसके�आस-पास�लगभग�1000 पे�ो��ल�स�( शैलो�क�ण��) पाए�गए�ह�।
पे�ो��ल�स�के�बारे�म���न�न�ल�खत�म��से�कौन�से�कथन�स�य�ह��?

1. पे�ो��ल�स��ाचीन��शला�च��ह���ज�ह���ाकृ�तक�रंग��का��योग�कर��शला��पर��च��त��कया�जाता�है।�

2. पे�ो��ल�स��व��म���ल�भ�ही��मलते�ह��और�इनके�एकमा���ात�नमूने�भारत�और�पा�क�तान�म���मले�ह�।�

�न�न�ल�खत�म��से�कूट�का�चयन�क��जए:

a) केवल�1
b) केवल�2
c) 1 और�2 दोन��
d) न�तो�1 न�ही�2
Correct Answer: D
Your Answer:
Explanation

Solution (d)

�पछले�दो�अथवा�तीन�वष��के�दौरान�र�ना�गरी�एवं�राजापुर��जल��से�पे�ो��ल�स�वाले��व�भ���थल�पाए�गए�ह�।�

पे�ो��ल�स�ऐसे��च��ह���ज�ह��च�ान�क��सतह�को�छे दकर, हटाकर, उ�क�ण��कर�अथवा�घीसकर�एक�शैल�कला�के��प�म��उकेरा�जाता�है।�

पे�ो��लप�श�द�को�पे�ो�ाफ�के�साथ�नह��जोड़ा�जाना�चा�हए�जो��क�च�ान�क��सतह�पर��च��त�क��गई�एक�छ�व�होती�है।�दोन��ही��कार�शैल
कला�अथवा�पा���क�कला�क��वृहत�तथा�अ�धक�सामा�य��ेणी�से�संबं�धत�ह��।�पे�ोफॉ�स��अथवा�अनेक�बड़ी�च�ान��तथा��शलाखंड��से�धरती
पर�बनाए�गए��ा�प�व�आकार�भी��बलकुल�अलग�होते�ह�।�

पे�ो��ल�स�काफ��सामा�य�ह��और��व��के�अनेक�भाग��म��पाए�जाते�ह�।�

QUESTION 44.
बतुक�मा�उ�सव�के�बारे�म���न�न�ल�खत�म��से�कौन�से�कथन�सही�ह��?

a) यह�एक�पु�पो�सव�है�जो�मु�यत: तेलंगाना�क���ह���म�हला���ारा�मनाया�जाता�है।�
b) यह�ना�कला�का�एक��प�है��जसम��रामायण�के�अनुसार�भगवान�राम�का�जीवन�दशा�या�जाता�है।�
c) यह�एक�माश�ल�आट� �उ�सव�है�जो�केरल�म��प��मी�घाट�के�आ�दवा�सय���ारा�मनाया�जाता�है।�
d) यह�महीने�भर�चलने�वाला�एक�उ�सव�है�जो�दे वी�ल�मी�के���त�स�मान��द�श�त�करने�हेतु�आं��दे श�म��मनाया�जाता�है।�
Correct Answer: A
Your Answer:
Explanation

Solution (a)

IASbaba
Score:
Web: http://ilp.iasbaba.com/
47.00 /
Email: ilp@iasbaba.com
Page 103 200
2019 - Test 28-
Exam Title :
History & C...
Email : narenips6@gmail.com
Contact :

बतुक�मा�एक�पु�पो�सव�है�जो�मु�यत: तेलंगाना�आं��दे श�के�कुछ�भाग��क���ह���म�हला���ारा�मनाया�जाता�है।�यह�उ�सव���तवष��सातवाहन


पंचांग�के�अनुसार�नौ��दन��के��लए�मनाया�जाता�है��जसक��शु�आत�भा�पद�पू�ण�मा�से�होती�है�और�यह��गा��मी�तक�चलता�है।�अं�ेजी�कैल�डर
के�अनुसार�यह�सामा�यत: �सतंबर-अ�ूबर�के�महीने�म��आता�है।�

QUESTION 45.
�व��क��सबसे�बड़ी�प�ी�मू�त��‘जटायु’ भारत�म��बनाई�गई�है।�यह��न�न�ल�खत�म��से��कस�रा�य�म����थत�है�?

a) त�मलनाडु �
b) उ�र��दे श�
c) केरल�
d) ओ�डशा�
Correct Answer: C
Your Answer:
Explanation

Solution (c)

केरल�पय�टन��त�वनंतपुरम�के�नेचर�पाक��म��सबसे�बड़ी�प�ी�मू�त��का�अनावरण�करेगा।�

‘जटायु’ (जो��क�रामायण�म��एक��करदार�था) प�ी�क����तमू�त��200 फ�ट�लंबी�, 150 फ�ट�चौड़ी�, तथा�65 फ�ट�ऊँची�है�तथा�समु��तल�से


1,000 फ�ट�ऊँची�एक�च�ान�पर���थत�है।�

एक�कहानी�के�अनुसार, ‘जटायुपारा’ नामक�यह�च�ान��जस�पर�यह�मू�त��बनाई�गई�है, वही��थान�है�जहाँ�पौरा�णक�प�ी�‘जटायु’ रावण��ारा


उसके�पंख�काटे �जाने�के�प�ात��गरा�था।�

QUESTION 46.

25 व��अंतरा��ीय�ऊंट�उ�सव�का�आयोजन�भारत�म���कया�गया।�इसके�बारे�म���न�न�ल�खत�म��से�कौन�से�कथन�स�य�ह��?

1. यह���त�वष��दो��दन��के��लए�क�छ�के�रन�म��आयो�जत��कया�जाता�है।�

2. इसे�राज�थान�के�पय�टन, कला�एवं�सं�कृ�त��वभाग��ारा�आयो�जत��कया�जाता�है।�

3. �व�भ���े���से�लोक�कलाकार�यहाँ�आते�ह��और���तुती�दे ते�ह�।�

�न�न�ल�खत�म��से�कूट�का�चयन�क��जए�:

a) केवल�1
b) 2 और�3
c) 1 और�3
d) उपयु���सभी�
Correct Answer: B
Your Answer:
Explanation

Solution (b)

अंतरा��ीय�ऊंट�उ�सव�

यह�ऊंट�के�स�मान�म��बीकानेर, राज�थान�म��आयो�जत��कया�जाने�वाला�एक�दो��दवसीय�उ�सव�है�।

इसे�राज�थान�के�पय�टन, कला�एवं�सं�कृ�त��वभाग��ारा�आयो�जत��कया�जाता�है।�

IASbaba
Score:
Web: http://ilp.iasbaba.com/
47.00 /
Email: ilp@iasbaba.com
Page 104 200
2019 - Test 28-
Exam Title :
History & C...
Email : narenips6@gmail.com
Contact :

फर�क�ट�ग, ऊंट�को�सजाना, �डजाइ�न�ग�जैसी�कई�ग�त�व�धयाँ�आयो�जत�क��जाती�ह��। �व�भ���े���से�लोक�कलाकार�यहाँ�आते�ह��और


��तुती�दे ते�ह�।�

QUESTION 47.
डॉ. �ग�रजा�दे वी�को���ांज�ल�दे ने�के��लए�कोलकाता�म���ए�वा�ष�क�शा��ीय�उ�सव�‘ सुर�उ�सव�’ का�नाम�बदलकर�‘ सुर��ग�रजा�दे वी�उ�सव’
कर��दया�गया�है�। �ग�रजा�दे वी�को��न�न�ल�खत�म��से��कसक��सा�ा�ी�के�तौर�पर�जाना�जाता�था-

a) दादर�क��
b) ठु मरी�क��
c) गजल�क��
d) भजन�क��
Correct Answer: B
Your Answer:
Explanation

Solution (b)

सुर�उ�सव�

· शहर�म��होने�वाले�एक�वा�ष�क�शा��ीय�उ�सव�का�नाम��दवंगत�ठु मरी�सा�ा�ी�को���ांज�ल�दे ने�के��लए�‘ सुर��ग�रजा�दे वी�उ�सव’ कर��दया


गया�है�।

· उ�सव, �जसका�आयोजन�4 फरवरी�को�होगा, �ग�रजा�दे वी�के��श�य��दे ब��य�अ�धकारी�और�सम�वय�सरकार��ारा��कया�जाता�है�।

· �पछले�तीन�सं�करण��म��, इसे�‘ सुर�उ�सव�’ कहा�जाता�था��क�तु�आगे�से�इसे�‘ सुर��ग�रजा�दे वी�उ�सव’ कहा�जाएगा�।

QUESTION 48.
‘ क�मीर�डेली�’ वह�पहली�फ�चर��फ�म�है��जसे�पूरी�तरह�क�मीर�घाट��म���फ�माया�और�बनाया�गया�है।�इसके�बारे�म���न�न�ल�खत�म��से�कौन�
सा/से�कथन�सही�है/ह��?

1. यह�क�मीर�म��आतंकवाद�और�सेना�के�अ�याचार��जैसे��वषय��का�अ�वेषण�करती�है।�

2. यह��फ�म�क�मीर�म���कसी��थएटर�म���रलीज़�नह��होगी��य��क�घाट��म��कोई�काय�शील��सनेमा�हाल�नह��है।�

�न�न�ल�खत�म��से�कूट�का�चयन�क��जए�:

a) केवल�1
b) केवल�2
c) 1 और�2 दोन��
d) न�तो�1 न�ही�2
Correct Answer: B
Your Answer:
Explanation

Solution (b)

45 वष��म��पहली�बार�, एक�क�मीरी�फ�चर��फ�म��जसे�पूरी�तरह�क�मीर�घाट��म���फ�माया�और�बनाया�गया�है, क�मीर�के�बाहर��सनेमा�घर�


म���द�श�त�क��जाएगी�।

क�मीर�डेली�क�मीर�म��नशीले�पदाथ��क��लत�और�बेरोजगारी�जैसे��वषय��का�अ�वेषण�करती�है।�

�वडंबना�है��क��फ�म�क�मीर�म���कसी��थएटर�म���रलीज़�नह��होगी��य��क�घाट��म��कोई�काय�शील��सनेमा�हाल�नह��है।�

IASbaba
Score:
Web: http://ilp.iasbaba.com/
47.00 /
Email: ilp@iasbaba.com
Page 105 200
2019 - Test 28-
Exam Title :
History & C...
Email : narenips6@gmail.com
Contact :
QUESTION 49.
जनजातीय�समुदाय��क��कला�और�सं�कृ�त�के�गुणगान�हेतु�‘आ�द�महो�सव’ मनाया�गया।��न�न�ल�खत�म��से��कस�रा�य�म��यह�मनाया�गया�?

a) म�य��दे श�
b) नागाल�ड�
c) असम�
d) राज�थान�
Correct Answer: C
Your Answer:
Explanation

Solution (c)

‘ आ�द�महो�सव�’ का�आयोजन�गुवाहाट�, असम�म��10 �दन��तक�जनजातीय�समुदाय��क��कला�और�सं�कृ�त�के�गुणगान�हेतु��कया�गया�।

QUESTION 50.
महाम�तका�भषेक�उ�सव���येक�12 वष��म��आयो�जत�होता�है।�महाम�तका�भषेक�के�बारे�म���न�न�ल�खत�म��से�कौन�से�कथन�स�य�ह��?

1. गोमेते�र, �ज�ह��बा�बली�के�नाम�से�भी�जाना�जाता�है, का�महाम�तका�भषेक�12 वष��म��एक�बार�होता�है।�

2. इसे�कना�टक�के��वणबेलगोला�म��आयो�जत��कया�जाता�है।�

3. यह��व��के�सबसे�बड़े�बौ��उ�सव��म��से�एक�है।�

�न�न�ल�खत�म��से�कूट�का�चयन�क��जए�:

a) 1 और�2
b) 2 और�3
c) 1 और�3
d) उपयु���सभी�
Correct Answer: A
Your Answer:
Explanation

Solution (a)

रा�प�त�राम�नाथ�को�वदं �कना�टक�के�हासन��जले�के��वणबेलगोला�म��भगवान�गोमेते�र�के�88 व��महाम�तका�भषेक�उ�सव�का�उ�ाटन�कर�गे�


गोमेते�र�का�महाम�तका�भषेक��ज�ह��बा�बली�के�नाम�से�भी�जाना�जाता�है, 12 वष��म��एक�बार�होता�है।�

यह�एक�मह�वपूण��जैन�तीथ��थल�है�और�लाख��पय�टक�57 फ�ट�लंबे�एका�म�प�थर�क��इस�मू�त��को�दे खने�हेतु��वणबेलगोला�आते�ह��।

QUESTION 51.
" ल��पी�ह�पई�पॉटरी�" को�इसके�ज�टल��डजाइन�एवं�इसे�बनाने�म��लगी��ौ�ो�गक��के�चलते�अंतरा��ीय�बाजार�म���मुखता��मलनी�आरंभ�हो
गई�है।�इसके�बारे�म���न�न�ल�खत�म��से�कौन�से�कथन�स�य�ह��?

1. यह��ाचीन�कला�म�णपुर�म��दो�ल��पी��ाम��अथा�त�ल��पी�खु�लेन�तथा�ल��पी�कजुई�से��नकली।�

2. यह�म�णपुर�के�पहाड़ी��जले�म���नवास�करने�वाले�तांगखुल�समुदाय��ारा�बनाई�जाती�है।�

�न�न�ल�खत�म��से�कूट�का�चयन�क��जए�:

IASbaba
Score:
Web: http://ilp.iasbaba.com/
47.00 /
Email: ilp@iasbaba.com
Page 106 200
2019 - Test 28-
Exam Title :
History & C...
Email : narenips6@gmail.com
Contact :

a) केवल�1
b) केवल�2
c) 1 और�2 दोन��
d) न�तो�1 न�ही�2
Correct Answer: C
Your Answer:
Explanation

Solution (c)

म�णपुर�क���वदे शी�ल��पी�ह�पई�पॉटरी�को�इसके�ज�टल��डजाइन�एवं�इसे�बनाने�म��लगी��ौ�ो�गक��के�चलते�अंतरा��ीय�बाजार�म���मुखता
�मलनी�आरंभ�हो�गई�है।�

यह��ाचीन�कला�म�णपुर�म��उख�ल��जले�के��नकट�दो�ल��पी��ाम��अथा�त�ल��पी�खु�लेन�तथा�ल��पी�कजुई�से��नकली।�यह�मु�यत: म�णपुर�के
पहाड़ी��जले�म���नवास�करने�वाले�तांगखुल�समुदाय��ारा�बनाई�जाती�है।�

ल��पी�पॉटरी�काले�स�प�लाकार�भूतल�प�थर�एवं��वशेष�भूरी��म���के��म��त��म�ण�से�बनाई�जाती�है�, जो�ल��पी��ाम�म��पाई�जाती�है�। पा��


को�हाथ�से�आकार��दया�जाता�है, पो�लश��कया�जाता�है�और�धूप�म��सुखाया�जाता�है�। त�प�ात�इसे�आग�म��तपाया�जाता�है��जसे�पूरा�होने�म�
कुल�छ: �दन�लगते�ह��।

QUESTION 52.
इंडो-इ�ला�मक��थाप�य�कला�के�बारे�म���न�न�ल�खत�कथन��पर��वचार�क��जए�:

1. मुगल�भारत�म��इ�ला�मक��थाप�य�कला�लेकर�आए।�

2. मु��लम��थाप�य�कला�के�अंतग�त��कसी�सतह�पर��कसी�जीवधारी�का��च�ण���तबं�धत�है।�

3. कुतुबमीनार�का��नमा�ण�कुतुब��न�ऐबक, इ�तुत�मश�तथा��फरोजशाह�तुगलक��ारा��कया�गया।�

उपयु���म��से�कौन�से�कथन�स�य�ह��?

IASbaba
Score:
Web: http://ilp.iasbaba.com/
47.00 /
Email: ilp@iasbaba.com
Page 107 200
2019 - Test 28-
Exam Title :
History & C...
Email : narenips6@gmail.com
Contact :

a) 1 और�2
b) 2 और�3
c) 1 और�3
d) उपयु���सभी�
Correct Answer: B
Your Answer:
Explanation

Solution (b)

हालां�क�इ�लाम�भारत�म��Though Islam came to India over a period of 600 वष��क��अव�ध�के�दौरान��ापा�रय�,


कारोबा�रय�, धम�गु���के�मा�यम�से�आया, �क�तु�13 व��सद��म��आकर�तुक��सा�ा�य�(�जसक���थापना�उ�र�भारत�म��तुक��क���वजय�के
प�ात��ई�थी) �ारा�बड़ी�इमारत��के��नमा�ण�का�काय��आरंभ�हो�सका�।

�ह����से��भ��जो�अपनी�धा�म�क�मा�यता��के�अंतग�त�अनेक��प��म��हर�जगह�भगवान�के�होने�का��वचार�रखते�ह�, मु��लम��म���कसी�सतह
पर��कसी�जीवधारी�का��च�ण���तबं�धत�है।�

1200 ई�वी�म��, �द�ली�स�तनत�के�सं�थापक�कुतुब��न�ऐबक�ने�कुतुबमीनार�का��नमा�ण��ारंभ��कया�। 1220 म��, ऐबक�के�उ�रा�धकारी


तथा�दामाद�इ�तुत�मश�ने�इस�मीनार�म��तीन�मं�जल��जोड़ी�। 1369 म��, ऊपरी�मं�जल��बजली��गरने�से�पूरी�तरह�न��हो�गई�। अत:,
�फरोजशाह�तुगलक�ने�इसके�जीण��ार�का�काय���कया�और���त��त�मं�जल�के��थान�पर���येक�वष��दो�नई�मं�जल��का��नमा�ण�करवा��दया�।

QUESTION 53.
‘तोरण�’ के�बारे�म���न�न�ल�खत�म��से�कौन�से�कथन�स�य�ह��?

1. ये��तूप�के��वेश��ारा�होते�ह�।�

2. ये�दो��तंभ��तथा�उ�ह��जोड़ने�वाले�तीन��ै�तज�शहतीर��वाले��वतं��ढाँचे�होते�ह�।�

3. उन�पर�बु��क��जीवन�कथा��को�बारीक��के�साथ�उकेरा�जाता�है।�

4. �तूप��के�चार�तोरण�(��येक��दशा�म��एक) होते�ह�।�

�न�न�ल�खत�म��से�कूट�का�चयन�क��जए�:

a) 1,2 और�3
b) 2,3 और�4
c) 1,3 और�4
d) उपयु���सभी�
Correct Answer: D
Your Answer:
Explanation

Solution (d)

IASbaba
Score:
Web: http://ilp.iasbaba.com/
47.00 /
Email: ilp@iasbaba.com
Page 108 200
2019 - Test 28-
Exam Title :
History & C...
Email : narenips6@gmail.com
Contact :

तोरण�, भारतीय��वेश�ार�जो�सामा�यत: प�थर�के�बने�होते�ह�, बौ��तीथ��थल�अथवा�एक��ह���मं�दर�के��वेश�माग��को��द�श�त�करते�ह��। तोर


ण��म��आमतौर�पर�दो��तंभ�होते�ह���जनम���त�भ��के�दोन��ओर�बाहर��नकले�दो�या�तीन�अनु��थ�शहतीर�होते�ह��। ये�लकड़ी�के��नमा�ण�से��काफ�
�मलते-जुलते�ह��तथा��ाय: ऊपर�से�नीचे�तक�उ�कृ��मू�त�य��से�ढके�होते�ह��। साँची�के�महान��तूप�के�चार�तोरण�इसका�शानदार�उदाहरण�ह��।

QUESTION 54.
महावीर�और�जैन�धम��के�बारे�म���न�न�ल�खत�कथन��पर��वचार�क��जए।�नीचे��दया�गया�कौनसा�कथन�सही�नह��है�?

a) महावीर�जो��क�24 व��तीथ�कर�थे, को�जैन�धम��का�सं�थापक�माना�जाता�है।�


b) महावीर�6 वष��तक�म�खली�घोषाल�पु��से�जुड़े�रहे��क�तु�आगे�चलकर�गंभीर�दाश��नक�मतभेद��के�चलते�अलग�हो�गए�।
c) तब�महावीर��न��थ�सं�दाय�से�जुड़े�जब�क�म�खली�घोषाल�पु��ने�आजी�वक�धम��आरंभ��कया।�
d) महवीर�के�प�ात, जैन�धम��महावीर�के�11 �श�य��(गणधर) के�अधीन�आया।�
Correct Answer: A
Your Answer:
Explanation

Solution (a)

IASbaba
Score:
Web: http://ilp.iasbaba.com/
47.00 /
Email: ilp@iasbaba.com
Page 109 200
2019 - Test 28-
Exam Title :
History & C...
Email : narenips6@gmail.com
Contact :

· पा��नाथ�, जो��क�जैन�धम��के�23 व��तीथ�कर�थे, को�जैन�धम��का�सं�थापक�माना�जाता�है�।

· पहले�कथन�को�छोड़कर�, बाक��सभी�कथन�सही�ह��।

S. No. Tirthankara Associated Symbol

पहले�तीथ�कर� ऋषभ/आ�दनाथ� बैल�

21 व��तीथ�कर� ने�मनाथ� नीलकमल�

22 व��तीथ�कर� अ�र�ने�म� शंख�

23 व��तीथ�कर� पा�ा�नाथ� सांप�

24 व��तीथ�कर� महावीर� �स�ह�

QUESTION 55.
�न�न�ल�खत�कथन��पर��वचार�क��जए�:

1. अशोक�के�शासनकाल�( तीसरी�शता�द��ईसा�पूव��) के�दौरान�, सं�कृत�दे श�क��बोलचाल�क��भाषा�थी।�

2. �वशेषकर�गु�तो�र�काल�म��पाली�भाषा�को�यही�दजा���मला�और�वह�राजक�य�भाषा�बन�गई।�

उपयु����दए�गए�कथन��म��से�कौन�सा/से�सही�है/ह��?

a) केवल�1
b) केवल�2
c) 1 और�2 दोनो�
d) न�तो�1 न�ही�2
Correct Answer: D
Your Answer:
Explanation

Solution (d)

· तीसरी�शता�द��ईसा�पूव��म���ाकृत�(न��क�सं�कृत) दे श�क��बोलचाल�क��भाषा�थी।�भारत�के��मुख��ह�स��म�, अशोक�के�लेख��ाकृत


भाषा�तथा��ा�ी��ल�प�म���लखे�गए�। �ा�ी�गु�त�काल�के�अंत�तक�मु�य��ल�प�बनी�रही�।

· आगे�चलकर�सं�कृत�( न��क�पा�ल�) को�वही�दजा���मला�और�वह�दे श�के�सु�र��ह�स��तक�राजक�य�भाषा�बन�गई।�इस����या�को


चौथी�शता�द��म��गु�त�काल�के�दौरान��मुखता��ा�त��ई�।

· हालां�क, राजनी�तक��प�से�दे श�म��गु�तो�र�काल�म��अनेक�छोटे �रा�य�उभरे, �क�तु�आ�धका�रक�द�तावेज़�सं�कृत�म���लखे�गए�।

QUESTION 56.

IASbaba
Score:
Web: http://ilp.iasbaba.com/
47.00 /
Email: ilp@iasbaba.com
Page 110 200
2019 - Test 28-
Exam Title :
History & C...
Email : narenips6@gmail.com
Contact :

हड़�पाई�सं�कृ�त�के�पशुप�त�महादे व, जो��क�तीन�स�ग��वाले�एक�पु�ष�दे वता�ह��तथा�एक�पंजे�को��सरे�के�ऊपर�रखकर�योगी�क��मु�ा�म��बैठे�ह�,


क��मुहर�(जो�मोह�जोदारो�म���मली) म��वह��न�न�ल�खत�से��कन�पशु��से��घरे�ह��?

1. भ�स�

2. हाथी�

3. ग�डा�

4. बाघ�

5. �हरन�

उ�चत�कूट�चु�नए�:

a) 1, 2 और�5
b) 1, 2, 4 और�5
c) 1, 3, 4 और�5
d) उपयु���सभी�
Correct Answer: D
Your Answer:
Explanation

Solution (d)

�ा�या�:

· �च�/मुहर�को��यान�से�दे ख��और�पशु��क��पहचान�कर��।

IASbaba
Score:
Web: http://ilp.iasbaba.com/
47.00 /
Email: ilp@iasbaba.com
Page 111 200
2019 - Test 28-
Exam Title :
History & C...
Email : narenips6@gmail.com
Contact :

QUESTION 57.
�न�न�ल�खत�म��से�कौन�से�जैन�पंचसू��का/के��स�ांत�है/ह��?

1. अ�ह�सा�

2. स�य�

3. �याय�

4. धम��

5. ��चय��

IASbaba
Score:
Web: http://ilp.iasbaba.com/
47.00 /
Email: ilp@iasbaba.com
Page 112 200
2019 - Test 28-
Exam Title :
History & C...
Email : narenips6@gmail.com
Contact :

उ�चत�कूट�चु�नए�:

a) उपयु���सभी�
b) केवल�1, 2 और�3
c) केवल�1, 2 और�5
d) केवल�1, 2, 4 और�5
Correct Answer: C
Your Answer:
Explanation

Solution (c)

जैन��स�ांत�

पंचसू��: जैन�धम��के�आधारभूत��स�ांत�अथा�त�अ�ह�सा�, स�य, अप�र�ह, अ�तेय, ��चय��।

��र�न�/ र�न�य�: जैन�धम��के�तीन�र�न-

1. स�यक���या�

2. स�यक��ान�

3. स�यक��व�ास�

QUESTION 58.
सा�ह��यक�लेखन�पर��वचार�क��जए�और��न�न�ल�खत�म��से�गलत�यु�म�छां�टए�:

सा�ह��यक�लेखन�लेखक�

a) हष�च�र��बाणभ��
b) रामच�र��सं�याकर�नंद��
c) �व�मांकदे वच�र��क�हण�
d) मू�षक�वंश�अतुल�
Correct Answer: C
Your Answer:
Explanation

Solution (c)

· �व�मांकदे वच�र�, �ब�हण��ारा��लखा�गया�है��जसम��वह�अपने�संर�क�चालु�य�नरेश��व�मा�द�य�VI क��उपल��धय��का�वण�न�करता�है�।

· राजतरं�गणी�अथवा�‘राजा��क��तरंगे�’ क�हण��ारा��लखी�गई�है�। यह�क�मीर�के�राजा��क��जीव�नय��क���ृंखला�है�।

QUESTION 59.
ऋ�वेद�के�बारे�म���न�न�ल�खत�म��से�कौन�सा/से�कथन�सही�है/ह��:

1. ऋ�वेद�इंडो-यूरोपीय�भषा��का�सबसे��ाचीन��ंथ�है।�यह�अ��न, इं�, �म�, व�ण�तथा�अ�य�दे वता��क��क�वय��अथवा�संत��के��व�भ�


कुल���ारा�क��गई��ाथ�ना��का�सं�ह�है।�

2. ऋ�वेद�क��कई�बात��अवे�ता�(ईरानी�भाषा�का�सबसे��ाचीन��ंथ) से��मलती-जुलती�ह�।�

�दए�गए�उपयु���कथन��म��से�कौन�सा/से�सही�है/ह��?

IASbaba
Score:
Web: http://ilp.iasbaba.com/
47.00 /
Email: ilp@iasbaba.com
Page 113 200
2019 - Test 28-
Exam Title :
History & C...
Email : narenips6@gmail.com
Contact :

a) केवल�1
b) केवल�2
c) 1 और�2 दोन��
d) न�तो�1 न�ही�2
Correct Answer: C
Your Answer:
Explanation

Solution (c)

· �दए�गए�दोन��कथन�सही�और��व-�ा�याकारी�ह��।

· ऋ�वेद�क��कई�बात��अवे�ता�(ईरानी�भाषा�का�सबसे��ाचीन��ंथ) से��मलती-जुलती�ह�।�दोन���ंथ��म��कई�दे वता��और�यहाँ�तक��क�सामा�जक


वग��के��लए�भी�एक�जैसे�नाम��योग��ए�ह��।

QUESTION 60.
नीचे�पंचमहाक�याण/बु��के�जीवन�क��पाँच�घटनाएं�द��गई�ह�।�नीचे�द��गई�सूची�से�सही�सुमे�लत�यु�म/यु�म��का�चयन�क��जए-

सूची�I ( घटना�का�नाम�) सूची�II ( उससे�जुड़े��तीक�)

1. महा�भ�न��मण���������������������������������������बो�ध/ पीपल�वृ��

2. धम�च�प�रवत�न�����������������������������������������24 ती�लय��वाला�धम�च��

3. महाप�र�नवा�ण���������������������������������������������तूप�

4. बु��का�ज�म���������������������������������������������������कमल�

उ�चत�कूट�चु�नए:

a) केवल�1, 3 और�4
b) केवल�3 और�4
c) केवल�2 और�4
d) केवल�4
Correct Answer: B
Your Answer:
Explanation

Solution (b)

पंचमहाक�याण/बु��के�जीवन�क��पाँच�घटनाएं�:

IASbaba
Score:
Web: http://ilp.iasbaba.com/
47.00 /
Email: ilp@iasbaba.com
Page 114 200
2019 - Test 28-
Exam Title :
History & C...
Email : narenips6@gmail.com
Contact :

QUESTION 61.
�न�न�ल�खत�म��से�सही�यु�म�का�चयन�क��जए�:

1. �स�क��का�अ�ययन�: : नू�म�मै�ट�स�

2. अ�भलेख��का�अ�ययन�: : इ�प�फ��

3. �शलालेख��म��पुरातन�लेखन�और�अ�भलेख��का�अ�ययन�: : पै�लआ�फ��

कूट�का�चयन�क��जए�:

a) उपरो��सभी�
b) इनम��म��से�कोई�नह��
c) केवल�1
d) केवल�2 तथा�3

IASbaba
Score:
Web: http://ilp.iasbaba.com/
47.00 /
Email: ilp@iasbaba.com
Page 115 200
2019 - Test 28-
Exam Title :
History & C...
Email : narenips6@gmail.com
Contact :
Correct Answer: A
Your Answer:
Explanation

Solution (a)

· त�या�मक����- �प�ीकरण�क��आव�यकता�नह��है�

QUESTION 62.
शा��ीय�संगीत�क���ह���तानी�और�कना�टक�शै�लय��के�म�य��न�न�ल�खत�समानता��और�अंतर��पर��वचार�क��जए।�इसके�बारे�म���न�न�ल�खत
म��से�कौन�सा�कथन�गलत�है�?

a) �ह���तानी�संगीत�क��उ�प���कना�टक�संगीत�से�पहले��ई�है।�यह�इ�ला�मक�परंपरा��और�फारसी�मु�सक-ए-अ�सल�शैली�का�सं�ेषण�है
तथा�इसका�धम��से�कोई�संबंध�नह��है।�
b) �ह���तानी�संगीत�क��उ�प���वै�दक�काल�म���ई�, जब�क�कना�टक�संगीत�क��उ�प���भ���आंदोलन�के�दौरान��ई।�
c) कना�टक�संगीत�म��मु�य�जोर�वोकल�संगीत�पर�है�और�अ�धकांश�रचना��को�गाया�जाता�है�, �जसम��गायक��शैली�शा�मल�है।�
d) य��प�शैलीगत��भ�ताएँ�ह��, ले�कन�कना�टक�और��ह���तानी�दोन��क��न�व�के��प�म���वर�, राग�और�ताल�के�मूल�त�व�समान�ह�।�
Correct Answer: A
Your Answer:
Explanation

Solution (a)

�प�ीकरण:

• जैसा��क�शा��ीय�संगीत�क���ह���तानी�और�कना�टक�शैली�का�धम��के�साथ�बड़ा�संबंध�है।�

कना�टक�संगीत�भारतीय�उप�महा��प�के�द��णी�भाग�से�संबं�धत�संगीत�क��एक��णाली�है।��ह���तानी�संगीत�उ�र�भारत�से�जुड़ा�है
और�फारसी�और�इ�लामी�संगीत��णाली�से�गहराई�से��भा�वत�है।�

समानताएँ�

(i) य��प�शैलीगत��भ�ताएँ�ह��, ले�कन�कना�टक�और��ह���तानी�दोन�के�आधार�के��प�म���वर�, राग�और�ताल�के�मूल�त�व�समान�ह�।�

(ii) �ह���तानी�संगीत�क��उ�प���वै�दक�काल�म���ई�, जब�क�कना�टक�संगीत�क��उ�प���भ���आंदोलन�के�दौरान��ई।�इस��कार�दोन��का�धम�


से�बड़ा�संबंध�है।�

(iii) कना�टक�शा��ीय�संगीत�क��दो�मु�य�उप-शै�लय��म��से�एक�है�, जो��ाचीन��ह���परंपरा��से��वक�सत��आ�है�, �सरी�उप-शैली��ह���तानी


संगीत�है�, जो�उ�र�भारत�म��फारसी�और�इ�लामी��भाव��के�कारण�एक��व�श���प�म��उभरी�है।�

(iv) दोन��संगीत�अपने�आप�म��और�वै�दक�परंपरा��के�मा�यम�से�सं�कृत�भाषा�क���ल�पय��के�साथ��वक�सत��ए।�

(v) इन�दोन���णा�लय��म��क���य�धारणा�एक�मधुर��वधा�या�राग�, शक�रा�से�एक�लयब��च��या�ताल�है।�

असमानताएँ�

(i) �ह���तानी�संगीत�क��उ�प���कना�टक�संगीत�से�पहले�क��है।�यह�वै�दक�मं���, इ�लामी�परंपरा��और�फारसी�मु�सक-ए-अ�सल�शैली�से


सं�े�षत��आ�है।�कना�टक�तुलना�मक��प�से�शु��है�और�15 व��16 व��शता�द��म��भ���आंदोलन�के�दौरान��वक�सत��आ�था�और�19 व��-
20 व��शता�द��म��भी�इसे�बढ़ावा��मला।�

(ii) कना�टक�संगीत�म��मु�य�जोर�वोकल�संगीत�पर�है�और�अ�धकांश�रचनाएँ�गाईक��शैली�को�शा�मल�करते��ए�गाई�जाती�ह�।��ह���तानी�संगीत
म��वोकल-क���त�समूह�क��आव�यकता�होती�है।�कई�उपकरण��को�गायक��ारा�उपयोग�के��लए��डज़ाइन��कया�गया�है।�

(iii) �ह���तानी�संगीत�के��मुख�वोकल��प��ुपद�, �याल�, तराना�, ठु मरी�, दादरा�और�गजल�ह�।�जब�क�कना�टक�संगीत�म��अ�पना�क��कई


�क�म��शा�मल�ह��, �जनम��अ�पना�, �नरावल�, क�पन�वरम�और�रागम�थाना�प�लवी�शा�मल�ह�।�

IASbaba
Score:
Web: http://ilp.iasbaba.com/
47.00 /
Email: ilp@iasbaba.com
Page 116 200
2019 - Test 28-
Exam Title :
History & C...
Email : narenips6@gmail.com
Contact :

�ह���तानी�शा��ीय�संगीत: भारतीय�शा��ीय�संगीत�स�पूण��उ�र�भारत�म��पाया�जाता�है।�इस�शैली�को�कभी-कभी�उ�र�भारतीय�शा��ीय
संगीत�या�शा��ी�संगीत�कहा�जाता�है।�यह�एक�परंपरा�है�जो�वै�दक�अनु�ान��म��उ�प���ई�और�12 व��शता�द��से�उ�र�भारत�और�कुछ�हद�तक
नेपाल�और�अफगा�न�तान�म���वक�सत��ई�है।�

कना�टक�संगीत: आमतौर�पर�भारतीय�उपमहा��प�के�द��णी�भाग�से�जुड़ा�संगीत�क��एक��णाली�है�, �जसका��े��लगभग�भारत�के�चार


आधु�नक�रा�य�: आं���दे श�, कना�टक�, केरल�और�त�मलनाडु �तक�सी�मत�है।�यह�भारतीय�शा��ीय�संगीत�क��दो�मु�य�उप-शै�लय��म��से�एक
है�जो��ाचीन��ह���परंपरा��से��वक�सत��ई�है�; अ�य�उप-शैली��ह���तानी�संगीत�है�, जो�उ�र�भारत�म��फारसी�और�इ�लामी��भाव��के�कारण
एक��व�श���प�म��उभरा।�

QUESTION 63.
' पंच�स�ां�तका�' के�बारे�म���न�न�ल�खत�म��से�कौन�सा�कथन�सही�है�?

1. यह�आय�भ���ारा��ल�खत�है।�

2. यह��ीक�और�रोमन��यो�तष�का��व�तृत��ववरण�है।�

�न�न�ल�खत�म��से�सही�कूट�का�चयन�क��जए�:

a) केवल�1
b) केवल�2
c) 1 तथा�2 दोन��
d) न�तो�1 और�न�ही�2
Correct Answer: D
Your Answer:
Explanation

Solution (d)

पंच�स�ा�त�वराह�म�हर��ारा��लखा�गया�था।�यह�ग�णतीय�खगोल��व�ान�पर�एक��ंथ�है।�

QUESTION 64.
�न�न�ल�खत�कथन��पर��वचार�क��जए�—

1. यह�नृ�य��प�महापु�ष�शंकरदे व��ारा��चलन�म��लाया�गया�था।�

2. नृ�य�के�साथ�संगीतमय�संयोजन�को�बोरगीत�कहा�जाता�है।�

3. उपयोग�क��जाने�वाली�वेशभूषा�पैट�से��न�म�त�होती�है।�

�न�न�ल�खत��वक�प��म��से�नृ�य-�प�क��पहचान�क��जए�:

a) मो�हनीअ�म�
b) थांग�
c) स���य�नृ�य�
d) कु�चपुड़ी�
Correct Answer: C
Your Answer:
Explanation

Solution (c)

महापु�ष�शंकरदे व: एक�महान�वै�णव�संत�और�असम�के�सुधारक�

IASbaba
Score:
Web: http://ilp.iasbaba.com/
47.00 /
Email: ilp@iasbaba.com
Page 117 200
2019 - Test 28-
Exam Title :
History & C...
Email : narenips6@gmail.com
Contact :

पैट:

· असम�म��उ�पा�दत�रेशम�

· शहतूत�के�पौधे�से��ु�प��

QUESTION 65.
पैतकर�प��ट�ग�है�- एक�संकट��त�कला�का��प��जसका�पतन�हो�रहा�है�और�यह�ज�द�ही��वलु�त�हो�सकती�है�के�संबंध�म���न�न�ल�खत�कथन�
म��से�कौन�सा�स�य�नह��है�?

a) पैतकर�प��ट�ग�असम�के�आ�दवासी�लोग���ारा�अ�य�त���ॉल�प��ट�ग�ह�।�
b) प��ट�ग�के�इस�पुराने��प�क��सां�कृ�तक��वरासत�म��बंगाली�घर��म��सबसे���स��दे वी�मा�मानसा�म��से�एक�है।�
c) पैतकर��च���म��य���को�रखने�और��भ�ा�दे ने�के�सामा�जक-धा�म�क�री�त-�रवाज��के�साथ�संबंध�ह�।�
d) �च���म��एक�सामा�य��वषय�है�- मानव�जीवन�मृ�यु�के�बाद��या�होता�है।�
Correct Answer: A
Your Answer:
Explanation

Solution (a)

· पैटकर�प��ट�ग�झारखंड�रा�य�के�आ�दवासी�लोग���ारा�बनाई�जाने�वाली���ॉल�प��ट�ग�ह�।�

पैटकर�प��ट�ग�भारत�म��आ�दवासी��च���के�शु�आती��प��म��से�एक�है�, और�इसे�अपने��कटन�के�कारण���ॉल�प��ट�ग�भी�कहा�जाता�है।�ये
प��ट�ग�कहानी�कहने�के�तरीके�को�दशा�ती�त�वीर��ह�।�अमा�बी�(झारखंड) के�पैटकर�प��ट�ग�रा�य�और�दे श�म��सबसे�अ�धक�मांग�वाले��च���म��से
एक�है।�अम�बी�को�पैटकर�गांव�के��प�म��भी�जाना�जाता�है।��च�कला�क��यह��वधा�झारखंड�और�उससे�सटे �रा�य��क��एक�रचना�मक�लोक
कला�है।�इस�कला�से�संबं�धत��च���म��मृ�यु�के�बाद�मानव�जीवन�के�साथ��या�होता�है�, इसका�एक�सामा�य��वषय�है।�जब�क��सरी�ओर�, इस
�कार�क��प��ट�ग�बंगाली�और�झारखंडी�क��रोजमरा��क���ज�दगी�को�भी��च��त�करती�है।�इन��च���के�सामा�य��वषय�ह�-

· वन�प�त�और�जीव�

IASbaba
Score:
Web: http://ilp.iasbaba.com/
47.00 /
Email: ilp@iasbaba.com
Page 118 200
2019 - Test 28-
Exam Title :
History & C...
Email : narenips6@gmail.com
Contact :

· �क�वदं ती�और�लोककथा�

· पारंप�रक��ह���महाका��

· �यौहार�और�मेले�

QUESTION 66.
�चकनकारी�है�

a) एक�ज�टल�और�सू�म�छाया-�कार�क��कढ़ाई�जो�पारंप�रक��प�से�रंगहीन�मलमल�पर�सफेद�यान��के�साथ�क��जाता�है��जसे�तंज़ेब�कहा
जाता�है।�
b) एक�कालीन�, ईरानी�मूल�के�फारसी�कालीन��के�समान�, जो�पूरी�तरह�से�हाथ�से�बनाया�गया�है�, हाथ�से�बुना��आ�है�और�मु�य��प�से
शु��ऊन�, शु��रेशम�और�कभी-कभी�ऊन�और�रेशम��म�ण��से��न�म�त��कया�जाता�है।�
c) व�तुतः�‘पेन��ा�ट’- एक�ऐसी�कला�है��जसम��साड़ी�पर�प��ट�ग�के�अलावा�ह�ड��लॉक�����ट�ग�शा�मल�है।�
d) अंबूर�, त�मलनाडु �का�एक��वा�द��खा��पदाथ��, �जसे�हाल�ही�म��सामान�का�भौगो�लक�संकेत�(पंजीकरण�और�संर�ण�अ�ध�नयम
1999) अ�ध�नयम�के�तहत�जीआई�टै ग��ा�त��आ�है।�
Correct Answer: A
Your Answer:
Explanation

Solution (a)

· �चकनकारी�एक�ज�टल�और�बारीक�छाया-�कार�क��कढ़ाई�है�जो�परंपरागत��प�से�रंगीन�धागे�के�साथ�सफेद�धागे�के�साथ��कया�जाता�है
�जसे�तंजेब�कहा�जाता�है।�

IASbaba
Score:
Web: http://ilp.iasbaba.com/
47.00 /
Email: ilp@iasbaba.com
Page 119 200
2019 - Test 28-
Exam Title :
History & C...
Email : narenips6@gmail.com
Contact :

QUESTION 67.
थंगका�कला�के�संबंध�म���न�न�ल�खत�कथन��पर��वचार�क��जए�:

1. थांगका�म�णपुर�क��सबसे��ाचीन�माश�ल�आट� �म��से�एक�है।�
2. थांगका�म��एक�तलवार�या�भाले�का�उपयोग�एक�या�एक�से�अ�धक��वरो�धय��के��खलाफ�करना�शा�मल�है।�

उपरो��कथन�म��से�कौन�सा/से�सही�है/ह��?

a) केवल�1
b) केवल�2
c) दोन��
d) कोई�भी�नह��
Correct Answer: D
Your Answer:
Explanation

Solution (d)

IASbaba
Score:
Web: http://ilp.iasbaba.com/
47.00 /
Email: ilp@iasbaba.com
Page 120 200
2019 - Test 28-
Exam Title :
History & C...
Email : narenips6@gmail.com
Contact :

· थांगका��स��कम�(और�साथ�ही��त�बत) के��व�श���च��ह��, मूल��प�से���ा�का�एक�मा�यम�है��जसके�मा�यम�से�बौ��धम��के�उ�चतम


आदश��का��वकास��आ।�

· थांगका�कपास�के�कैनवास�पर�बनाए�जाते�ह��और�अ�सर�रेशम�से�बनाए�जाते�ह�।�वे�बौ��धम��से�संबं�धत��व�भ��दे वता��और�दश�न�क�
छ�वय��को��च��त�करते�ह�।�

· थांगका�पारंप�रक��प�से�बौ��पुजा�रय��और��भ�ु��और��व�श��जातीय�समूह���ारा�बनाए�गए�थे�, इससे�संबं�धत�कौशल�पीढ़��दर�पीढ़�
तक�ह�तांत�रत��कए�जा�रहे�थे।�(अब�थांगका�एक�लु�त�ाय�कला�का��प�है��जसका�पतन�हो�रहा�है)

· थंगका�बनाने�म���जन�रंग��का�उपयोग��कया�जाता�है�, वे�सभी�वेज�या�ख�नज�रंग�ह���ज�ह���कृ�त�से��ा�त��कया�जाता�है।�

नोट: थांग-टा�म�णपुर�क��सबसे��ाचीन�माश�ल�आट� �म��से�एक�है।�थांग-ता�म��एक�या�अ�धक��वरो�धय��के��खलाफ�तलवार�या�गोले�का�उपयोग


करना�शा�मल�है।�

QUESTION 68.
तूती-नामा�के�संबंध�म���न�न�ल�खत�कथन��पर��वचार�क��जए�

1. यह�लघु��च���म��कहा�नय��का�स�च��संकलन�है।�

2. यह�काय��अकबर��ारा�मा�यता��ा�त�था।�

3. इसक���वषय-व�तु�और�कहा�नयाँ�शुकस�त�त( Śukasaptati ) नामक�12 व��शता�द��के�सं�कृत�संकलन�से��ु�प��ह�।�

उपरो��कथन��म��से�कौन�सा/से�सही�है/ह��?

a) केवल�1 तथा�2
b) केवल�2
c) 1, 2 तथा�3
d) केवल�1
Correct Answer: C
Your Answer:
Explanation

Solution (c)

तूतीनामा�को�मुगल��कूल�का�पहला�काय��माना�जाता�है।�तुतीनामा�का�शा��दक�अथ��है�"टे �स�ऑफ़�ए�पैरट"।�यह�250 लघु��च���म��52


कहा�नय��का�स�च��संकलन�है।�इस�काय��को�अकबर�का�संर�ण��ा�त�था।�इसके��वषय��और�कहा�नय��को�12 व��शता�द��के�सं�कृत
एंथोलॉजी�से��लया�गया�है��जसका�शीष�क�सुकस�त�त�या�“तोता�क��सातव��दा�तां” है।�तोता�लगातार�52 रात��म��52 कहा�नयाँ
सुनाता�है�और�इन�कहा�नय��म��वह�अपने�मा�लक�को�कुछ�नै�तक�कहा�नयाँ��सखाता�है।�मीर�सै�यद�अली�और�अ��स�समद�के�नेतृ�व�म��यह
काय��पाँच�वष��म��पूरा��आ।�यह���थ�न�बी�, एक�जातीय�फारसी��च�क�सक�और�एक�सूफ��संत��ारा��लखा�गया�था�, जो�बदायूं�चले�गए�थे।
इसे�फारसी�म���लखा�गया�था।�

�या�आप�जानते�ह��?

· हमजा-नाम�, �जसम��कपड़े�पर��च��शा�मल�ह��, मूल��प�से�स�ह�खंड��म��1400 प��य��से��मलकर�बना�है।���येक�प�ा�लगभग�27 20


x20 का�है।�ये�प��ट�ग�एक�फ़ारसी�हमज़ानामा�या�दा�तान-ए-अमीर�हमज़ा�पर�आधा�रत�थ�।�

QUESTION 69.
महाब�लपुरम�के�रथ�मं�दर��के�बारे�म���न�न�ल�खत�म��से�कौन�सा�कथन�सही�है�?

1. ये�प�लव�शासक���ारा��न�म�त�रॉक�कट�मं�दर�ह�।�

2. पाँच�रथ�, पाँच�पांडव��के��लए�एक�ह�।�

IASbaba
Score:
Web: http://ilp.iasbaba.com/
47.00 /
Email: ilp@iasbaba.com
Page 121 200
2019 - Test 28-
Exam Title :
History & C...
Email : narenips6@gmail.com
Contact :

3. यह�यूने�को��ारा�मा�यता��ा�त�महाबलीपुरम�म���मारक��के�समूह�के��प�म���व��धरोहर��थल�का�भाग�है।�

�न�न�ल�खत�म��से�चयन�क��जए�:

a) 1 तथा�2
b) 2 तथा�3
c) 1 तथा�3
d) उपरो��सभी�
Correct Answer: C
Your Answer:
Explanation

Solution (c)

पंच�रथ�

भारत�के�त�मलनाडु �रा�य�के�कांचीपुरम��जले�म��, बंगाल�क��खाड़ी�के�कोरोमंडल�तट�पर�महाब�लपुरम�म��पंच�रथ�(�जसे�पाँच�रथ�या�पांडव�रथ


भी�कहा�जाता�है) एक��मारक�प�रसर�है।�पंच�रथ�भारतीय�रॉक-कट�वा�तुकला�का�एक�उदाहरण�है।�डे�ट�ग�से�7 व��शता�द��के�उ�राध��म��इसे
राजा�मह��वम�न��थम�और�उनके�पु��नर�स�हवम�न��थम�( 630–680 ई�; �जसे�प�लव�सा�ा�य�का�माम�ला�या�"महान�यो�ा") भी�कहा
जाता�है।�नर�स�हवम�न�का�एक�नवाचार�, संरचना�भारतीय�वा�तुकला�क��एक�अ��त��मसाल�है।�यह�प�रसर�भारतीय�पुरात�व�सव��ण
(एएसआई) के�त�वावधान�म��है�और�यूने�को��ारा�महाबलीपुरम�म���मारक��के�समूह�के��प�म��यूने�को��ारा��ल�खत��व��धरोहर��थल�का
�ह�सा�है।�

पंच�रथ�प�रसर�म��पांच��मारक��म��से���येक�एक�रथ�(रथ) जैसा��दखता�है�, और���येक�को��ेनाइट�के�एकल�, लंबे�प�थर�या�मोनो�लथ�पर


उकेरा�गया�है�, जो�उ�र-द��ण��दशा�म��ढलान�के�साथ�थोड़ा�सा�झुका�है।�मं�दर��, संरचना��को�कभी�भी�संर��त�नह���कया�गया��य��क�वे
कभी�भी�नर�स�हवम�न�क��मृ�यु�के�बाद�पूरे�नह���ए�थे।�संरचना��का�नाम�महाका��महाभारत���स���क��पंच�पांडव��और�उनक��प�नी��ौपद�
के�नाम�पर�रखा�गया�है।�उनके�आकार�के��म�म��, वे�धम�राज�रथ�, भीम�रथ�, अजु�न�रथ�, नकुल�सहदे व�रथ�, और��ौपद��रथ�शा�मल�ह�।�

नोट: �सरा�कथन�गलत�है��य��क�5 पांडव��के��लए�चार�रथ�ह��और��ौपद��के��लए�एक�रथ�है।�

QUESTION 70.
चचनामा�अली�कुफ���ारा��लखा�गया�था।�यह��कसके�इ�तहास�से�संबं�धत�है�

a) �द�ली�स�तनत�
b) �स�ध�
c) क�मीर�
d) बहमनी�सा�ा�य�
Correct Answer: B
Your Answer:
Explanation

Solution (b)

चच�नामा�

चच�नामा�सातव��से�आठव��शता�द��ई�वी�म���स�ध�के�इ�तहास�के��लए�मु�य�ऐ�तहा�सक��ोत��म��से�एक�है�, जो�फारसी�म���लखा�गया�है।�

मुह�मद��बन�का�सम�के�8 व��शता�द��के�शु�आती��वजय�क��कहा�नय��के�साथ�पाठ�को�लंबे�समय�से�एक�मूल�, ले�कन�अनुपल�ध�अरबी�पाठ


के�` अली�कुफ���ारा�फ़ारसी�म��13 व��शता�द��का�अनुवाद�माना�जाता�है।�

नोट: �स�ध�के�राजा�दा�हर�, �ज�ह��मोह�मद��बन�का�सम�ने�हराया�था�, चच�वंश�के�थे।�

QUESTION 71.

IASbaba
Score:
Web: http://ilp.iasbaba.com/
47.00 /
Email: ilp@iasbaba.com
Page 122 200
2019 - Test 28-
Exam Title :
History & C...
Email : narenips6@gmail.com
Contact :

सूफ��दश�न�के�बारे�म���न�न�ल�खत�यु�म��पर��वचार�क��जए�:

दश�न� अथ��

1. वाह�दत-अल-वजूद� अ��त�व�क��एकता�

2. वाहदत�अंश-शु�द� उप��थ�त�क��एकता�

3. अल-वजूद�अल-मुनबा�सत� �व-�कट�होना�

उपरो��म��से�कौन-सा�यु�म�सही�सुमे�लत�है/ह��?

a) केवल�1
b) केवल�1 तथा�2
c) 1, 2 तथा�3
d) केवल�3
Correct Answer: C
Your Answer:
Explanation

Solution (c)

सूफ��त�वमीमांसा�के��मुख��वचार��ने�वीदाह�क��अवधारणा�को�"एकता" , या�अरबी�ता��हद�से�संबं�धत�है।�इस��वषय�पर�दो�मु�य�सूफ��दश�न
�च�लत�ह�।�वाहदत-अल-वुजद�का�शा��दक�अथ��"अ��त�व�क��एकता" या�"होने�क��एकता" है�, ले�कन�बेहतर�अनुवाद�अ��त�ववाद
का�एके�रवाद�होगा।�यहां�वजूद�(यानी�अ��त�व) अ�लाह�के�वजूद�को�संद�भ�त�करता�है�- �न�हताथ��अ�लाह�के�वुजूद�के�वहादत�/�वा�हद�है।
�सरी�ओर�, वाहदत-अश-शु�द�, �जसका�अथ��है�" अ�परे�ट�म" या�"गवाह�का�एके�रवाद" , यह�मानता�है��क�भगवान�और�उसक�
रचना�पूरी�तरह�से�पृथ�क�ह�।�

अल-वुजूद�अल-मु�ब�सत�(Self-unfolding Being)

शाह�वलीउ�लाह�दे हलवी�ने�इ�न�अरबी�के�वदाहत�अल-वुज�द�(एक�होने�क��एकता) और�वहादत�अश-शु�द(अंतरा�मा�म��एकता) के�दो


(जा�हरा�तौर�पर) �वरोधाभासी��स�ांत��को��मलाने�क��को�शश�क�।�शाह�वलीउ�लाह�ने�इन�मतभेद��को�' मौ�खक��ववाद�' बताते��ए�बड़े�ही
करीबी�तरीके�से�संघष��को�सुलझाया�, जो�अ�प��भाषा�के�कारण�आए�ह�।�अगर�हम�छोड़�द� �, तो�वे�कहते�ह��, �वचार��क��अ�भ����के��लए
इ�तेमाल��कए�गए�सभी��पक�और�उपमाएं�, दो��पक��के��प���प�से��वपरीत��वचार��से�सहमत�ह�गे।�

�या�आप�जानते�ह��?

· जब�क���ढ़वाद��मुसलमान�बाहरी�आचरण�पर�जोर�दे ते�ह��, सू�फय��ने�आंत�रक�शु�ता�पर�जोर��दया।�

· जब�क���ढ़वाद��लोग�अनु�ान��के�अंध�पालन�म���व�ास�करते�ह��, सूफ���ेम�और�भ���को�मो���ा�त�करने�का�एकमा��साधन�मानते�ह�।�

· सू�फय��के�अनुसार��कसी�को�एक�पीर�या�गु��का�माग�दश�न�होना�चा�हए�, �जसके��बना�आ�या��मक��वकास�असंभव�है।�

· सूफ�वाद�ने�भी�अपने�अनुया�यय��म��स�ह�णुता�क��भावना�पैदा�क�।�

QUESTION 72.
गोल�गुंबद�के�बारे�म���न�न�ल�खत�कथन��पर��वचार�क��जए:

1. यह�संगमरमर�से��न�म�त�है�तथा�इसका��ला�टरवक��अलंकृत�है।�

2. इसका�गुंबद�उपजाप-वी�थ( whispering gallery ) के��लए���स��है।�

उपरो��कथन��म��से�कौन-सा/से�सही�है/ह��?

IASbaba
Score:
Web: http://ilp.iasbaba.com/
47.00 /
Email: ilp@iasbaba.com
Page 123 200
2019 - Test 28-
Exam Title :
History & C...
Email : narenips6@gmail.com
Contact :

a) केवल�1
b) केवल�2
c) 1 तथा�2 दोन��
d) न�तो�1 और�नह��2
Correct Answer: B
Your Answer:
Explanation

Solution (b)

गुंबद�एक�वगा�कार�इमारत�है��जसम��एक�गोलाकार��म�होता�है��जसके�ऊपर�एक�राजसी�गुंबद�है�, जो�इमारत�को�यह�नामकरण�दे ता�है।�

यह�गहरे��े�बेसा�ट�से��न�म�त�है�तथा��ला�टरवक��से�अलंकृत�है।�

इमारत�म��एक�अ��त��व�नक��णाली�है।�गुंबद�के��म�के�साथ�, एक�फुसफुसाती�गैलरी�है�जहां��व�न�बढ़�जाती�ह��और�कई�बार�गूंज�उठती�ह�।�

गोल�गुंबद�म�यकालीन�भारत�म����थत�कई�शै�लय��का�एक�अ�छा�अ�भसरण�है।�

�या�आप�जानते�ह��?

�ह�डोला�महल�, मांडू�

· यह�एक�रेलवे�पुल�पुल�क��तरह��दखता�है�, �जसक��द�वार��का�समथ�न�करने�वाले�बड़े-बड़े�बु�रे�सेस( buttresses ) ह�।�

· यह�सु�तान�का�ऑ�डयंस�हॉल�था।�

· झूले�(�ह�डोला) क��द�वार��को�छाप�दे ने�के��लए�बैटर�का�उपयोग�ब�त��भावी�ढं ग�से��कया�गया�था।�

QUESTION 73.
�न�न�ल�खत�कथन��पर��वचार�क��जए�

1. चारबाग�पैटन��का�उपयोग��मायूँ�के�मकबरे�म���कया�गया�था।�

2. लाल�महलबाड़ी�म��सीढ़�दार�चारबाग�पैटन��का�उपयोग��कया�गया�है।�

3. शालीमार�उ�ान�म���रवर��ंट�चार�बाग�पैटन��का�उपयोग��कया�गया�है।�

सही�कूट�का�चयन�क��जए�

a) केवल�1
b) केवल�1 तथा�2
c) केवल�1 तथा�3
d) उपरो��सभी�
Correct Answer: A
Your Answer:
Explanation

Solution (a)

· बाबर�को�औपचा�रक�उ�ान��क��योजना�बनाने�और��बछाने�म���दलच�पी�थी, आयताकार�द�वार��वाले�बाड़��के�भीतर�रखा�गया�और
कृ��म�चैनल���ारा�चार��तमा�हय��म���वभा�जत��कया�गया।�

· इन�बाग��को�चार�बाग़�कहा�जाता�था, चार�बाग़�उनके�सम�मत��वभाजन�के�कारण।�अकबर�के�साथ�शु�, क�मीर, आगरा�और��द�ली


म��जहाँगीर�और�शाहजहाँ��ारा�कुछ�सबसे�खूबसूरत�चार�बाग�का��नमा�ण��कया�गया�था।�

IASbaba
Score:
Web: http://ilp.iasbaba.com/
47.00 /
Email: ilp@iasbaba.com
Page 124 200
2019 - Test 28-
Exam Title :
History & C...
Email : narenips6@gmail.com
Contact :

QUESTION 74.
�न�न�ल�खत�म��से�कौन�सही�ढं ग�से�सुमे�लत�है�?

1. अकबरनामा�- अबुल�फजल�

2. �कताब�उल��ह�द�- अल�ब�नी�

3. तबक़ात�ई�ना�सरी�- �म�हाज��सराज�

�न�न�ल�खत�म��से�कूट�का�चयन�क��जए�:

a) 1 तथा�2
b) 2 तथा�3

IASbaba
Score:
Web: http://ilp.iasbaba.com/
47.00 /
Email: ilp@iasbaba.com
Page 125 200
2019 - Test 28-
Exam Title :
History & C...
Email : narenips6@gmail.com
Contact :

c) 1 तथा�3
d) उपरो��सभी�
Correct Answer: D
Your Answer:
Explanation

Solution (d)

अ��ल�फ़जल�

· अबुल�फ़ज़ल�मुग़ल�बादशाह�अकबर�का�वज़ीर�था�, और�अकबरनामा�का�लेखक�, 3 खंड��म��अकबर�के�शासनकाल�का�इ�तहास�है�, ( तीस


रे�वॉ�यूम�को�आइने�-ए-अकबरी�के��प�म��जाना�जाता�है) और�बाइ�बल�का�फारसी�अनुवाद।�

· वे�अकबर�के�राज�के�नौ��वे�स�(�ह�द�: नवर�न�) म��से�एक�भी�थे।�

अल�ब�नी�

· 973 म��वत�मान�के�उ�बे�क�तान�म���वा�र�म�ज�म।�

· 1017 म��, जब�सु�तान�महमूद�ने��वा�र�म�पर�आ�मण��कया�, तो�उ�ह�ने�कई��व�ान��और�क�वय��को�अपनी�राजधानी�गजनी�म��वापस�ले


�लया�; अल-�ब�नी�उनम��से�एक�था।�

· अल-ब�नी�ने��ा�ण�पुजा�रय��और��व�ान��क��कंपनी�म��वष���बताए�, सं�कृत�सीखने�और�धा�म�क�और�दाश��नक��ंथ��का�अ�ययन��कया।�

· उ�ह�ने�“�कताब-उल-�ह�द" �लखा।�

· यह�अरबी�म���लखा�गया�था�

· यह�भारत�के�धम��और�दश�न�, �यौहार��, खगोल��व�ान�, क��मया�, �श�ाचार�और�री�त-�रवाज��, सामा�जक�जीवन�, वजन�और�उपाय��, �


�तमा�, कानून��और�मे�ोलॉजी�जैसे��वषय��पर�80 अ�याय��म���वभा�जत�एक�बड़ा�पाठ�है।�

�म�हाज-ए-�सराज�

· 13 व��शता�द��का�एक�फ़ारसी�इ�तहासकार�घु�रड�क��राजधानी��फ़रोज�ख�म��पैदा��आ�था।�

· वह�श�सु��न�इ�तुत�मश�के�समकालीन�थे।�

उ�ह�ने�' तबक़ात-ए-ना�सरी�’�लखा।�

QUESTION 75.
सारंगी�और�रबाब�का�आ�व�कार��कसने��कया�था�?

a) अमीर�ख़ुसरो�
b) पीर�भोड़न�
c) अबुल�फजल�
d) अबुल�फैजी�
Correct Answer: A
Your Answer:
Explanation

Solution (a)

अ�मर�खुसरो�

· इस�काल�के���स��फ़ारसी�लेखक।�उ�ह�ने�कई�क�वताएँ��लख�।�

IASbaba
Score:
Web: http://ilp.iasbaba.com/
47.00 /
Email: ilp@iasbaba.com
Page 126 200
2019 - Test 28-
Exam Title :
History & C...
Email : narenips6@gmail.com
Contact :

· उ�ह�ने�कई�का���प��के�साथ��योग��कया�और�फारसी�क�वता�क��एक�नई�शैली�का��नमा�ण��कया�, �जसे�‘ सबाक-ए-�ह�द�या�भारतीय


शैली�’कहा�जाता�है।�

· उ�ह�ने�कुछ��ह�द��छं द�भी��लखे।�

· अमीर�ख़ुसरो�खज़ैन-उल-फ�तुह�और�तुगलक�नामा�के�लेखक�थे।�

· वे�सूफ��संगीतकार�, क�व�और��व�ान�थे।�

· अलाउ��न��खलजी�ने�अमीर�खुसरो�और�अमीर�हसन�जैसे�क�वय��को�संर�ण��दया।�

· अमीर�खुस��को�उ�� �भाषा�का��पता�माना�जाता�है�और�उ�ह��' भारत�के�तोते�' के��प�म��भी�जाना�जाता�है।�

· अ�मर�ख़ुसरो�सारंगी�और��सतार�के�आ�व�कारक�थे।�

· लैला�मजनू�और�तुगलक�नामा�अमीर�खुस��क����स��रचनाएँ�ह�।�

· अमीर�खुसरो�ने�घोरा�और�सनम�जैसे�कई�नए�राग�पेश��कए।�

· उ�ह�ने��ह���और�ईरानी��णा�लय��को�स��म��त�करके�"कवाली" के��प�म��जाने�जाने�वाले�ह�के�संगीत�क��एक�नई�शैली��वक�सत�क�।�

· �सतार�के�आ�व�कार�को�भी�उनके��लए��ज�मेदार�ठहराया�गया�था।�

पीर�भोडन�या�बुदन�

· उ�ह��बाबा�बुधन�अली�शाह�भी�कहा�जाता�था�

· वह�एक�स�मा�नत�सूफ��पीर�थे��ज�ह�ने�रावल�प�डी�म��गु��नानक�के�साथ�धा�म�क��वचन��कया�था�और�बाद�म��गु��हर�गो�ब�द�के�समय�म�
गुरमत��वचार�को��वीकार��कया।�

· वह��द�ली�स�तनत�काल�के�महान�संगीतकार��म��से�एक�थे।�

अ��ल�फ़जल�तथा�अबुल�फैजी�

· शेख�मुबारक�, �ीस�के�दश�न�और�सा�ह�य�के�साथ-साथ�इ�लामी�धम�शा���म��एक��व�ान�थे।�वह�धा�म�क�चचा���म��अकबर�से�जुड़े�थे।�

· अबुल�फजल�और�अबुल�फैजी�शेख�मुबारक�के�पु��थे।�

· अबुल�फैज़ी�अकबर�के�इ�तहासकार�अबुल�फ़ज़ल�के�बड़े�भाई�थे।�अकबर�ने�फैजी�क����तभा�को�अ�य�धक�मा�यता�द��और�उ�ह��अपने�बेट�
के��लए��ूटर��नयु���कया�और�उ�ह��अपने�सजावट��' नवर�न��' के�बीच�जगह�द�।�महाभारत�का�फारसी�भाषा�म��अनुवाद�उनक��दे खरेख�म�
�कया�गया�था।�

· अबुल�फ़ज़ल�मुग़ल�बादशाह�अकबर�का�वज़ीर�था�, और�अकबरनामा�का�लेखक�, 3 खंड��म��अकबर�के�शासनकाल�का�इ�तहास�है�, ( तीस


रे�वॉ�यूम�को�आइने-ए-अकबरी�के��प�म��जाना�जाता�है) और�बाइ�बल�का�फारसी�अनुवाद��कया।�वह�उनके�दो�त�और�आ�मकथाकार�थे�, उ
�ह�ने�अकबर�के�शासन-�शासन�के�बारे�म��एक��वचार���तुत�करने�म��मदद�क�।�

· अबुल�फ़ज़ल�ने�ग��लेखन�क��एक�शैली��नधा��रत�क��और�इसके�बाद�कई�पी�ढ़य��ने�अनुसरण��कया।�

· वे�अकबर�के�राज�के�नौ��वे�स�(�ह�द�: नवर�न�) म��से�एक�भी�थे।�

QUESTION 76.
‘�शरोका�पो�याना( Shiroka Polyana )’ हाल�ही�म��खबर��म��था।�यह�कहाँ���थत�है�?

a) �स�
b) �ीस�

IASbaba
Score:
Web: http://ilp.iasbaba.com/
47.00 /
Email: ilp@iasbaba.com
Page 127 200
2019 - Test 28-
Exam Title :
History & C...
Email : narenips6@gmail.com
Contact :

c) बु�गा�रया�
d) यू�ेन�
Correct Answer: C
Your Answer:
Explanation

Solution (c)

XVI व�ड��आइस��फ�श�ग�च��पयन�शप��शरोका�पॉ�लयाना�म��आयो�जत�क��गई�थी।�

यह�बु�गा�रया�म��प��मी�रोडो�स�पहाड़��म����थत�एक�झील�है।�

QUESTION 77.
‘ H5N1 ’ के�संबंध�म���न�न�ल�खत�कथन��पर��वचार�क��जए।�

1. यह�मनु�य��को��भा�वत�नह��करता�है�

2. यह�इ��लूएज
ं ा�A वायरस�का�एक�उप�कार�है�

3. यह�ताप�के���त�संवेदनशील�है�

सही�कूट�का�चयन�क��जए�:

a) 1 तथा�2
b) 2 तथा�3
c) 1, 2 तथा�3
d) केवल�2
Correct Answer: B
Your Answer:
Explanation

Solution (b)

इ��लुएज
ं ा�A वायरस�उप�कार�H 5 N 1, �जसे�A ( H 5 N 1) या�H 5 N 1 के��प�म��भी�जाना�जाता�है�, इ��लूएज
ं ा�A वायरस�का
उप�कार�है�जो�मनु�य��और�कई�अ�य�जानवर��क���जा�तय��म��बीमारी�का�कारण�बन�सकता�है।�

H5N1 एक��कार�का�इ��लूएज ं ा�वायरस�है�जो�ए�वयन�इ��लूएज ं ा�(या�"बड���लू") नामक�प��य��म��एक�अ�य�धक�सं�ामक�, गंभीर��सन


रोग�का�कारण�बनता�है।�H5N1 ए�वयन�इ��लूएज ं ा�के�मानवीय�मामले�कभी-कभी�होते�ह��, ले�कन�सं�मण�को�एक�����से��सरे����
तक�प�ंचाना�मु��कल�होता�है।�जब�लोग�सं��मत�हो�जाते�ह��, तो�मृ�यु�दर�लगभग�60% होती�है।�

यह�वायरस�ताप�के���त�संवेदनशील�है।�खाना�पकाने�के��लए�उपयोग��कए�जाने�वाले�सामा�य�तापमान�(ता�क�भोजन�सभी�भाग��म��70 �ड�ी
से��सयस�तक�प�ंच�जाए) वायरस�को�समा�त�कर�दे ता�है।�एक�मानक�सावधानी�के��प�म��, ड��यूएचओ�क��सलाह�है��क�पो����उ�पाद��और
जंगली�खेल�प��य��को�सदै व�अ�छ���व�छता��था��के�बाद�तैयार��कया�जाना�चा�हए�, और�यह��क�मुग��का�मांस�ठ�क�से�पकाया�जाना
चा�हए।�

आज�तक�, H5N1 वायरस�के�साथ�बड़ी�सं�या�म��मानव�सं�मण�खाना�पकाने�से�पहले�घर�म��इनके�वध�और�बाद�म��रोग��त�या�मृत�प��य�


से��नपटने�के�साथ�जोड़ा�जाता�है।�ये�अ�यास�मानव�सं�मण�के�उ�चतम�जो�खम�का���त�न�ध�व�करते�ह��और�बचने�के��लए�सबसे�मह�वपूण�
ह�।�

QUESTION 78.
अंतरा��ीय�छा��मू�यांकन�के��लए�काय��म�( PISA) के�संबंध�म���न�न�ल�खत�कथन��पर��वचार�क��जए�

1. इसे�वष��2012 म��आ�थ�क�सहयोग��वकास�संगठन�( OECD) �ारा��ार�भ��कया�गया�था�

IASbaba
Score:
Web: http://ilp.iasbaba.com/
47.00 /
Email: ilp@iasbaba.com
Page 128 200
2019 - Test 28-
Exam Title :
History & C...
Email : narenips6@gmail.com
Contact :

2. यह�पठन�, ग�णत�और��व�ान�म��15 वष�य�ब�च��के��ान�के��तर�का�परी�ण�करता�है�

3. यह���त�तीन�साल�म��आयो�जत��कया�जाता�है�

सही�कूट�का�चयन�क��जए�:

a) 1 तथा�2
b) 2 तथा�3
c) केवल�1
d) 1, 2 तथा�3
Correct Answer: B
Your Answer:
Explanation

Solution (b)

2021 म��अंतरा��ीय�छा��मू�यांकन�काय��म�( PISA) म��भारत�क��भागीदारी�हेतु�HRD मं�ालय�ने�आ�थ�क�सहयोग��वकास�संगठन�( OE


CD) के�साथ�एक�समझौते�पर�ह�ता�र��कए।�

पीआईएसए�- ओईसीडी��ारा�वष��2000 म���ार�भ��कया�गया�- पठन�, ग�णत�और��व�ान�म��15- वष�य�ब�च��के��ान�के��तर�का�परी�ण


करता�है।�यह�टे �ट���त�तीन�साल�म��आयो�जत��कया�जाता�है।�भारत�2009 म��अपने��नराशाजनक��दश�न�के�कारण�2012 और�2015 म�
पीआईएसए�म��भागीदार�नह��बना�रहा�, जब�इसे�74 दे श��के�दे श��म��72 वां��थान��ा�त��आ�था।�त�कालीन�यूपीए�सरकार�ने�2009 म��खराब
�दश�न�के��लए�"संदभ��के�बाहर" ����को��ज�मेदार�ठहराया�था।�दे श�ने�बाद�म��2012 और�2015 के�च��म��भाग�नह���लया।�

QUESTION 79.
‘ �वरकता�( Virakta )’ श�द��न�न�ल�खत�म��से��कसके�साथ�संबं�धत�है�?

a) जैन�धम��
b) बु��धम��
c) �ल�गायत�धम��
d) वै�णव�
Correct Answer: C
Your Answer:
Explanation

Solution (c)

Read More - https://en.wikipedia.org/wiki/Lingayatism

QUESTION 80.
‘ NGARM ’ के�संबंध�म���न�न�ल�खत�कथन��पर��वचार�क��जए�

1. यह��वदे शी��प�से�DRDO �ारा��वक�सत��कया�गया�है�

2. इसे��व�भ���कार�क���नगरानी�और�रडार�को�न��करने�के��लए��न�म�त��कया�गया�है�

सही�कथन�का�चयन�क��जए�

a) केवल�1
b) केवल�2
c) 1 तथा�2 दोन��
d) न�तो�1 और�न�ही�2

IASbaba
Score:
Web: http://ilp.iasbaba.com/
47.00 /
Email: ilp@iasbaba.com
Page 129 200
2019 - Test 28-
Exam Title :
History & C...
Email : narenips6@gmail.com
Contact :
Correct Answer: C
Your Answer:
Explanation

Solution (c)

इस��मसाइल�को�सुखोई�- 30 एमकेआई�लड़ाकू�से�दागे�जाने�के�बाद�जमीन�पर��व�भ���कार�क���नगरानी�और�रडार�के�ल�य��को�न��करने
के��लए�बनाया�गया�है�

�स�के�साथ�संयु���प�से��वक�सत�सुपरसो�नक���ोस��ूज��मसाइल�के�बाद�, डीआरडीओ��ारा��वक�सत�क��जाने�वाली�यह�नई�पीढ़��क�
�व�करण-रोधी��मसाइल�( NGARM), लगभग�100- �कमी�क����ाइक�र�ज�वाली�पहली��वदे शी�वायु�से�सतह�पर�वार�करने�वाली��मसाइल
है।�

Source: https://timesofindia.indiatimes.com/india/india-tests-new-anti-radiation-
missile-to-destroy-enemy-radars/articleshow/67676064.cms

QUESTION 81.
�न�न�ल�खत�म��से�कौन�सा�सही�सुमे�लत�है/ह��?

बंदरगाह�दे श�

1. �वादर�- पा�क�तान�

2. हंबनटोटा�- मालद�व�

3. �यूक�यू�- �यांमार�

सही�कूट�का�चयन�क��जए�:

a) 1 तथा�2
b) 1 तथा�3
c) 1, 2 तथा�3
d) केवल�1
Correct Answer: B
Your Answer:
Explanation

Solution (b)

�वादर�- पा�क�तान�

हंबनटोटा�- �ीलंका�

�यूक�यू�- �यांमार�

QUESTION 82.
‘ �ेट�इं�डयन�ब�टड��’ के�संबंध�म���न�न�ल�खत�कथन��पर��वचार�क��जए।�

1. यह�केवल�भारत�म��पाया�जाता�है�

2. यह�महारा��का�रा�य�प�ी�है�

3. इसे�IUCN �ारा�‘गंभीर��प�से�संकट��त�’ के��प�म��सूचीब���कया�गया�है�

सही�कथन�का�चयन�क��जए�:

IASbaba
Score:
Web: http://ilp.iasbaba.com/
47.00 /
Email: ilp@iasbaba.com
Page 130 200
2019 - Test 28-
Exam Title :
History & C...
Email : narenips6@gmail.com
Contact :

a) 1 तथा�2
b) 2 तथा�3
c) केवल�3
d) इनम��से�कोई�नह��
Correct Answer: C
Your Answer:
Explanation

Solution (c)

�ेट�इं�डयन�ब�टड��(अड��टस�नाइ��से�स) या�भारतीय�ब�टड��भारतीय�उपमहा��प�म��पाया�जाने�वाला�एक�ब�टड��है।�यह�राज�थान�का�एक
रा�य�प�ी�है।�

IUCN ��थ�त�– गंभीर��प�से�संकट��त�

QUESTION 83.
�न�न�ल�खत�म��से�कौन�सा�रा�ीय�उ�ान�तैरती��ई�वन�प�त�के�साथ�एक�दलदल�होने�के�कारण�अ��तीय�है�जो�एक�समृ��जैव��व�वधता�का
समथ�न�करता�है�?

a) �भतरक�नका�रा�ीय�उ�ान�
b) केइबुल�लामजाओ�नेशनल�पाक��
c) केवलादे व�घाना�रा�ीय�उ�ान�
d) सु�तानपुर�नेशनल�पाक��
Correct Answer: B
Your Answer:
Explanation

Solution (b)

केइबुल�लामजाओ�रा�ीय�उ�ान�भारत�म��म�णपुर�रा�य�के��ब�णुपुर��जले�म��एक�रा�ीय�उ�ान�है।�यह�40 वग���कमी��े��म��है�, जो��व��का


एकमा��तैरता��आ�पाक��है�, जो�उ�र�पूव��भारत�म����थत�है�, और�लोकटक�झील�का�अ�भ��अंग�है।�

QUESTION 84.
‘ �चन( Chin ) ’ लोग�मु�य��प�से�ह��

a) चीन�
b) �यांमार�
c) बां�लादे श�
d) भूटान�
Correct Answer: B
Your Answer:
Explanation

Solution (b)

�चन�लोग�बमा��म���मुख�नृजातीय�लोग��म��से�एक�ह�।��चन�, बमा��संघ�के�सं�थापक�समूह��(�चन�, का�चन�, शान�और��यांमार) म��से�एक�ह�।


�चन�लोग�रा�य�का��ाथ�मक�जातीय�समूह�है�, �जनसे�संबं�धत�कई�भाषाएं�, सं�कृ�तयां�और�परंपराएं�ह�।��चन�लोग��का�सबसे�बड़ा�जातीय
समूह�ज़ोमी�है।�ये�लोग�मु�य��प�से��चन�रा�य�, राखीन�रा�य�और��यांमार�के�सा�ग�ग��े��म���नवास�करते�ह��, ले�कन�पूरे�बमा��, बां�लादे श�और
भारत�म��शरणाथ��के��प�म��फैले��ए�ह�।�2014 के�बम��जातीय�जनगणना�म��, �चन�जातीयता�को��फर�से��चन�रा�य�के�लोग���ारा�खा�रज�कर
�दया�गया�था।�

IASbaba
Score:
Web: http://ilp.iasbaba.com/
47.00 /
Email: ilp@iasbaba.com
Page 131 200
2019 - Test 28-
Exam Title :
History & C...
Email : narenips6@gmail.com
Contact :
QUESTION 85.
‘ अ�खल�भारतीय��ामीण��व�ीय�समावेशन�सव��ण�2016-17’ के�संबंध�म���न�न�ल�खत�कथन��पर��वचार�क��जए।�

1. इसम��ऋण�, बचत�, �नवेश�, प�शन�, �ेषण�और�बीमा�से�लेकर��व�भ���व�ीय�समावेश�को�कवर��कया�गया�है।�

2. यह�नेशनल�ब�क�फॉर�ए�ीक�चर�एंड��रल�डेवलपम�ट( NABARD ) �ारा�संचा�लत��कया�गया�था�

3. 50% से�अ�धक�कृ�ष�प�रवार��को�बीमा�के�तहत�कवर��कया�गया�था�

सही�कथन�का�चयन�क��जए�

a) 1 तथा�2
b) 2 तथा�3
c) 1 तथा�3
d) उपरो��सभी�
Correct Answer: A
Your Answer:
Explanation

Solution (a)

एनएएफआईएस�को�2016-17 म��एक�रा�ीय��तर�के�सव��ण�के��प�म��लॉ�च��कया�गया�था�जो��ामीण�आबाद��क��आजी�वका�क����थ�त
और��व�ीय�समावेशन�के��तर�के�संदभ��म��एक��ापक�अवलोकन��दान�करता�है।�

NAFIS का�कवरेज�ऋण�, बचत�, �नवेश�, प�शन�, �ेषण�और�बीमा�से�लेकर��व�भ���व�ीय�समावेश�पहलु��तक�है।�सव��ण�म�����य��के


�व�ीय��ान�, ���कोण�और��वहार�का�आकलन�करना�भी�शा�मल�था�तथा��व�ीय�उ�पाद��और�सेवा��के�साथ�उनके�अनुभव�को�कै�चर
�कया�था�, �जसका�उ�ह�ने�उपयोग��कया�था।�

ऋण��तता�क��घटना( IOI), जो�सव��ण�क���त�थ�पर�बकाया�ऋण�वाले�प�रवार��का�अनुपात�है�, कृ�ष�और�गैर-कृ�ष�प�रवार��के��लए�52.5


��तशत�और�42.8 ��तशत�थी।�

अ�खल�भारतीय�IOI �ामीण�प�रवार��म��47.4 ��तशत�था�।�

जब�क�88.1 ��तशत��ामीण�प�रवार��और�55 ��तशत�कृ�ष�प�रवार��के�पास�ब�क�खाता�होने�क��सूचना�है�, ��त�प�रवार�औसत�बचत�173


488 थी�।�

यह�कहा�गया�है��क�लगभग�26 ��तशत�कृ�ष�प�रवार��और�25 ��तशत�गैर-कृ�ष�प�रवार��को�बीमा�के�तहत�कवर��कया�गया�था।�

इसी�तरह�, 18.9 ��तशत�गैर-कृ�ष�घरान��के�मुकाबले�20.1 ��तशत�कृ�ष�प�रवार��ने�प�शन�योजना��क��सद�यता�ली।�

Read More - https://www.nabard.org/auth/writereaddata/tender/1608180417NABARD-


Repo-16_Web_P.pdf

Source: https://economictimes.indiatimes.com/news/economy/agriculture/farm-loan-
waiver-how-to-nip-it-in-the-bud/articleshow/67410788.cms

QUESTION 86.
‘तेलंगाना�क��रायथु�बंधु�योजना’ और�ओ�डशा�क��का�लया�( KALIA ) योजना’ �ाय: समाचार��म��आती�है।�यह��कससे�संबं�धत�है�?

a) �कसान��के��लए�मु�त�सोलर�पंप�
b) �कसान��के��लए�मु�त��बजली�
c) �कसान��के��लए�स��सडी�वाले�उव�रक�और�क�टनाशक�
d) �कसान��के��लए�आय�/ �नवेश�सहायता�योजना�

IASbaba
Score:
Web: http://ilp.iasbaba.com/
47.00 /
Email: ilp@iasbaba.com
Page 132 200
2019 - Test 28-
Exam Title :
History & C...
Email : narenips6@gmail.com
Contact :
Correct Answer: D
Your Answer:
Explanation

Solution (d)

रायथु�बंधु�योजना�जो��क�एक��कसान��नवेश�सहायता�योजना�( FISS) है, यह�तेलंगाना�सरकार��ारा�एक�वष��म��दो�फसल��के��लए��कसान�


को��नवेश�समथ�न�करने�के��लए�एक�क�याणकारी�काय��म�है।�सरकार�रबी�और�खरीफ�सीजन�के��लए�58.33 लाख��कसान��को�, ��त
एकड़�4000 �पये�वष��म��दो�बार�कृ�ष��नवेश�का�समथ�न�करने�के��लए��दान�कर�रही�है।�यह�भारत�म��पहली���य���कसान��नवेश�सहायता
योजना�है�, जहाँ�सीधे�नकद�भुगतान��कया�जाता�है।�

आजी�वका�और�आय�संवध�न�के��लए�कृषक�सहायता�( KALIA )

यह�उड़ीसा�सरकार�क��एक�योजना�है��जसम��सभी��कसान��को�खेती�के��लए�सहायता�के��प�म����त�प�रवार�10,000 �पये��दान��कए
जाएंगे।�2018-19 और�2021-22 के�बीच�पांच�फसली�स���के��लए���येक�प�रवार�को�खरीफ�और�रबी�मौसम�म��अलग�से�5,000 �पये
�मल�गे।�

Read More - https://indianexpress.com/article/explained/kalia-how-odisha-new-scheme-


supports-farm-community-with-payments-5540259/

https://en.wikipedia.org/wiki/Rythu_Bandhu_scheme

QUESTION 87.
‘�लोबल�टै ल�ट�कॉ��प�ट�टवनेस�इंडे�स�( GTCI) ’ कौन-सी�सं�था��ारा�जारी��कया�जाता�है�

a) इनसीड( INSEAD )
b) �व��आ�थ�क�मंच�
c) यूएनडीपी�
d) अंतरा��ीय�मु�ा�कोष�
Correct Answer: A
Your Answer:
Explanation

Solution (a)

वष��2013 म��पहली�बार�लॉ�च��कया�गया�, �लोबल�टै ल�ट�कॉ��प�ट�टवनेस�इंडे�स�( GTCI) एक�वा�ष�क�ब�चमा�क�ग��रपोट� �है�जो�दे श��क�


��तभा�हेतु���त�पधा��क���मता�को�मापता�है।�

�लोबल�टै ल�ट�कॉ��प�ट�टव�इंडे�स�2019 म��भारत�80 व���थान�पर�है।�

QUESTION 88.
�न�न�ल�खत�म��से�कौन�सा�कथन�‘कर�अपवंचन’ श�द�को�सबसे�उपयु���प�से�संद�भ�त�करता�है�?

a) यह�आय�और�राइट�ऑफ�क��कलापूण��और��ानवध�क�फाइ�ल�ग�और��रपो�ट�ग�है�
b) यह�उपयु��कर�दे नदारी�का�भुगतान�करने�से�बचाव�करने�का�एक�सो�े �य��यास�है�
c) यह�ऐसी�संप��य��के��पांतरण�(चोरी) के��लए�एक�या�एक�से�अ�धक����य���ारा�संप���को�वापस�लेने�का�काय��है�, �ज�ह��संप��
स�पी�गई�थी�, या�तो�आयो�जत��कया�जाना�था�या��व�श���योजन��के��लए�उपयोग��कया�जाना�था।�
d) यह�गैरकानूनी��प�से��ा�त�धन�क��उ�प���को�ब��क�ग�ह�तांतरण�या�वा�ण��यक�लेनदे न�के�ज�टल�अनु�म�के�मा�यम�से��छपाने�क�
���या�है।�
Correct Answer: B
Your Answer:
Explanation

IASbaba
Score:
Web: http://ilp.iasbaba.com/
47.00 /
Email: ilp@iasbaba.com
Page 133 200
2019 - Test 28-
Exam Title :
History & C...
Email : narenips6@gmail.com
Contact :

Solution (b)

कर�अपवंचन�एक�गैरकानूनी�काय��है��जसम��कोई�����या�सं�था�जानबूझकर�कर�क��सही�दे नदारी�दे ने�से�बचती�है।�

QUESTION 89.
‘बेव�रज�व�( Beveridge curve )’ �कसके�साथ�संबं�धत�है�

a) आय��वतरण�
b) बेरोजगारी�
c) कर�से�जीडीपी�अनुपात�
d) मौ��क�नी�त�
Correct Answer: B
Your Answer:
Explanation

Solution (b)

बेव�रज�व��( Beveridge curve )

· यह�एक��ा�फकल���त�न�ध�व�को�संद�भ�त�करता�है�जो�एक�अथ��व�था�म��बेरोजगारी�दर�(�ै�तज�अ��पर) और�रोजगार��र���दर
(ऊ�वा�धर�अ��पर) के�म�य�संबंध�को�दशा�ता�है।�

· इसका�नाम����टश�अथ�शा��ी��व�लयम�बेव�रज�के�नाम�पर�रखा�गया�है।�

· बेव�रज�व��क���वणता�आमतौर�पर�नीचे�क��ओर�होता�है��य��क�ऐसे�समय�म��जब�अथ��व�था�म��उ�च�नौकरी�क���र���भी�अपे�ाकृत�कम
बेरोजगारी��ारा��च��त�होती�है��य��क�कंप�नयां�वा�तव�म��नए�लोग��को��नयु��करने�के��लए�स��य�होती�ह�।�

· इसी�तक��से�, कम�रोजगार��र���दर�आमतौर�पर�उ�च�बेरोजगारी�के�साथ�मेल�खाती�है��य��क�कंप�नयां�कई�लोग��को�नई�नौक�रय��म���नयु�
नह��कर�सकती�ह�।�

QUESTION 90.
�न�न�ल�खत�म��से�कौन�से�संभा�वत�कारण�ह���जससे�जीएसट��से�राज�व�ल�य��ा�त�नह���कए�जा�रहे�है�?

1. 28 ��तशत�के��लैब�म��व�तु��क��सं�या�कम�करना�

2. उ�च�इनपुट�टै �स��े�डट�का�दावा�करने�के��लए�नकली�चालान�बनाने�वाले��वसाय�

सही�कूट�का�चयन�क��जए�:

a) केवल�1
b) केवल�2
c) 1 तथा�2 दोन��
d) न�तो�1 और�न�ही�2
Correct Answer: C
Your Answer:
Explanation

Solution (c)

जीएसट��राज�व�ल�य��ा�त�नह��कर�रहा�है�- कारण�

· जीएसट��प�रषद�ने�अपनी�31 व��बैठक�म��लगभग�20 �े�णय��क��व�तु��और�कई�सेवा��क��दर��म��कटौती�क��है।�नतीजतन�, केवल�एक


सामा�य�उपयोग�वाली�व�तु�- सीम�ट�- को�28% के�उ�चतम�कर��लैब�म��छोड़�गया�है।�

IASbaba
Score:
Web: http://ilp.iasbaba.com/
47.00 /
Email: ilp@iasbaba.com
Page 134 200
2019 - Test 28-
Exam Title :
History & C...
Email : narenips6@gmail.com
Contact :

· �वाभा�वक��प�से�, 28% �लैब�म��व�तु��क��घटती�सं�या�के�प�रणाम�व�प�कम�कर�सं�ह�होगा।�कम�दर��को�ऑफसेट�करने�का�तरीका


टै �स�दे ने�वाले�लोग��क��सं�या�म��वृ���करना�है।�यानी�अनुपालन�बढ़ाना�होगा।�

· उ�च�इनपुट�टै �स��े�डट�का�दावा�करने�के��लए�नकली�चालान�बनाने�वाले��वसाय।�

QUESTION 91.
‘द��यूचर�ऑफ�रेल( The Future of Rail )’ �रपोट� ��ारा�जारी�क��गई�है�

a) �व��आ�थ�क�मंच�
b) अंतरा��ीय�रेल�प�रवहन�संघ�
c) अंतरा��ीय�ऊजा��एज�सी�
d) रेलवे�का�अंतरा��ीय�संघ�
Correct Answer: C
Your Answer:
Explanation

Solution (c)

द��यूचर�ऑफ़�रेल�

· अंतरा��ीय�ऊजा��एज�सी�( IEA) �ारा�"द��यूचर�ऑफ़�रेल" �रपोट� �जारी�क��गयी�है�

· यह�ऊजा��और�पया�वरणीय��न�हताथ��के�प�र�े�य�के�मा�यम�से���नया�भर�म��रेल�के�वत�मान�और�भ�व�य�के�मह�व�का��व�ेषण�करने�वाली
अपनी�तरह�क���थम��रपोट� �है।�

· �रपोट� �म��रेल�के�भ�व�य�पर�मौजूदा�योजना��और��नयम��के��भाव�क��समी�ा�क��गई�है�, और�उन��मुख�नी�तय��क��पड़ताल�क��गई�है�,


जो�भ�व�य�म��रेल�के��भाव�को�वा�त�वक�बनाने�म��सहायता�कर�सकती�ह�।�

· इस�पहले�वै��क��रपोट� �म��भारत�पर��यान�क���त��कया�गया�है�, भारत�म��रेल�क��अनूठ��सामा�जक�और�आ�थ�क�भू�मका�पर��व�तार�से�, सा


थ�ही�साथ�इसक��महान��थायी��मता�के�साथ�, यह��दखाने�के��लए��क�कैसे�भारत�एक�दायरे�और�पैमाने�पर�रेल�का��व�तार�करने�के��लए
अपने�नेटवक��का��व�तार�और�अ�तन�कर�सकता�है�अ��तीय�है।�

QUESTION 92.
‘ गो�डन�लंगूर�’ के�संबंध�म���न�न�ल�खत�कथन��पर��वचार�क��जए।�

1. यह�प��मी�घाट�म��पाया�जाता�है�

2. इसे�IUCN �ारा�' गंभीर��प�से�संकट��त�' के��प�म��सूचीब���कया�गया�है�

सही�कूट�का�चयन�क��जए�

a) केवल�1
b) केवल�2
c) 1 तथा�2 दोन��
d) न�तो�1 और�नह��2
Correct Answer: D
Your Answer:
Explanation

Solution (d)

IASbaba
Score:
Web: http://ilp.iasbaba.com/
47.00 /
Email: ilp@iasbaba.com
Page 135 200
2019 - Test 28-
Exam Title :
History & C...
Email : narenips6@gmail.com
Contact :

गी�का�गो�डन�लंगूर�( Trachypithecus geei), या�केवल�गो�डन�लंगूर�, प��मी�असम�, भारत�के�एक�छोटे �से��े��म��और�भूटान�के


�लैक�माउंटेन�क��पड़ोसी�तलहट��म��पाया�जाने�वाला�एक�पुराना�बंदर�है।�यह�भारत�क��सबसे�संकट��त��ाइमेट��जा�तय��म��से�एक�है।�इसे�कई
�हमालयी�लोग���ारा�लंबे�समय�से�प�व��माना�जाता�है�, गो�डन�लंगूर�को�पहली�बार�1950 के�दशक�म���कृ�तवाद��ई�पी�गी��ारा�प��मी
��नया�के��यान�म��लाया�गया�था।�भूटान�के�एक�भाग�म��, इसने�कै�ड�लंगूर�ट��पाइलटस�के�साथ�संकरण��कया�है।�

IUCN - संकट��त�

QUESTION 93.
‘कोलेसी�टो�क�नन( Cholecystokinin )’ एक�हाम�न�है�, �जसका�संबंध�है�

a) र���कंदन�
b) �तन�का��ध�
c) भूख�दमनक( Hunger suppressant )
d) सेल��जनन�और�सेल�पुनज�नन�
Correct Answer: C
Your Answer:
Explanation

Solution (c)

कोलेसी�टो�क�नन�( CCK)

· यह�जठरां���णाली�का�एक�पे�टाइड�हाम�न�है�जो�वसा�और��ोट�न�के�पाचन�को��ो�सा�हत�करने�के��लए�उ�रदायी�है।�

· कोलेसी�टो�क�नन�, �जसे�आ�धका�रक�तौर�पर�पै�ेकोज़ाइ�मन�कहा�जाता�है�, यह��हणी�, छोट��आंत�के�पहले�खंड�म��एंटरोएंडो�ाइन


को�शका���ारा�सं�े�षत�और��ा�वत�होता�है।�

· इसक��उप��थ�त��मशः�अ��याशय�और��प�ाशय�क��थैली�से�पाचन�एंजाइम��और��प��के��नग�मन�का�कारण�बनती�है�, और�एक�भूख
दमनकारी�के��प�म��भी�काय��करती�है।�

· यह�एक�तृ��त�हाम�न�है�जो��मृ�त�गठन�म��अ�य�धक�����कया�जाता�है�, उ�च��तर�पर�, �कसी�����के�अ�जाइमर�रोग�के��वकास�क�


संभावना�65% तक�कम�कर�सकता�है।�

· CCK छोट��आंत��और�म��त�क�दोन��म��पाया�जाता�है।�छोट��आंत��म��, CCK वसा�और��ोट�न�के�अवशोषण�क��अनुम�त�दे ता�है।�

QUESTION 94.
‘जमात-उल-मुजा�हद�न’ कौन-से�दे श�का�एक�आतंकवाद��संगठन�है�

a) बां�लादे श�
b) पा�क�तान�
c) अफ़ग़ा�न�तान�
d) �यांमार�
Correct Answer: A
Your Answer:
Explanation

Solution (a)

जमात�उल�मुजा�हद�न�

· यह�बां�लादे श�म��स��य�एक�इ�लामी�आतंकवाद��संगठन�है।�

IASbaba
Score:
Web: http://ilp.iasbaba.com/
47.00 /
Email: ilp@iasbaba.com
Page 136 200
2019 - Test 28-
Exam Title :
History & C...
Email : narenips6@gmail.com
Contact :

· यह�2018 म��बोधगया�मं�दर�और�बौ��धम��के�अ�य��तीक��पर�हमले�क��सा�जश�रचने�के��लए��ज�मेदार�है।�

QUESTION 95.
‘कारा�कटम( Karakkatam )’ �न�न�ल�खत�म��से�कौन-से�रा�य�से�संबं�धत�है�? ?

a) त�मलनाडु �
b) कना�टक�
c) तेलंगाना�
d) आं���दे श�
Correct Answer: A
Your Answer:
Explanation

Solution (a)

यह�त�मलनाडु �का�एक��ाचीन�लोक�नृ�य�है�, जो�वषा��दे वी�म�रय�मन�क���शंसा�म���कया�गया�था।��ाचीन�त�मल�महाका��कहता�है��क�इस


�कार�का�नृ�य�भारतम�से��लया�गया�है�और�त�मल�नृ�य�के�कई��प��का��म�ण�है�जैसे�भरतना�म�पो�चर�, मु�ाएँ।�इस�नृ�य�को�वषा��क��दे वी
का�आशीवा�द�लेने�के��लए��कया�जाता�है।�

QUESTION 96.
‘ रा�ीय�कृ�ष�उ�च��श�ा�प�रयोजना�( NAHEP) ’ के�संबंध�म���न�न�ल�खत�कथन��पर��वचार�क��जए।�

1. इसे�भारतीय�कृ�ष�अनुसंधान�प�रषद�( ICAR) �ारा�लॉ�च��कया�गया�है।�

2. इसे��व��ब�क�और�भारत�सरकार��ारा�50:50 के�आधार�पर��व��पो�षत��कया�जाएगा।�

सही�कथन�का�चयन�क��जए�

a) केवल�1
b) केवल�2
c) 1 तथा�2 दोन��
d) न�तो�1 और�नह��2
Correct Answer: C
Your Answer:
Explanation

Solution (c)

रा�ीय�कृ�ष�उ�च��श�ा�प�रयोजना�( NAHEP)

· यह�प�रयोजना�दे श�साझेदारी�रणनी�त�का�समथ�न�करती�है�और�एक�करण�, प�रवत�न�और�समावेश�के�तीन��े���क��संल�नता�को�संबो�धत


करती�है।�

· इन�संल�न��े���ने�कृ�ष�उ�पादकता�म��वृ���क��और�उ�च��श�ा�क��गुणव�ा�म��सुधार�के��लए�एक�और�अ�धक�कुशल�काय�बल�बनाने�म�
सहायता�करना�है�जो�कृ�ष�स�हत��मुख��े���क��उ�पादकता�म��लगातार�सुधार�करता�है।�

· ��ता�वत�प�रयोजना�एक�ब�-वै��क�अ�यास�सहयोग�(कृ�ष�और��श�ा) भी�है�और�यह�जलवायु�प�रवत�न�, नौक�रय��और��ल�ग�के


रणनी�तक��े���से�सीधे�संबं�धत�ग�त�व�धय��और�प�रणाम��का�समथ�न�करने�क��उ�मीद�है।�

· आईसीएआर�ने�हाल�ही�म��दे श�म����तभा�को�आक�ष�त�करने�और�उ�च�कृ�ष��श�ा�को�मजबूत�करने�के��लए�1100 करोड़��पये�क�


मह�वाकां�ी�रा�ीय�कृ�ष�उ�च��श�ा�प�रयोजना�(एनएएचईपी) �ार�भ�क��है।�

IASbaba
Score:
Web: http://ilp.iasbaba.com/
47.00 /
Email: ilp@iasbaba.com
Page 137 200
2019 - Test 28-
Exam Title :
History & C...
Email : narenips6@gmail.com
Contact :

· इस�प�रयोजना�को��व��ब�क�और�भारत�सरकार��ारा�50:50 के�आधार�पर��व��पो�षत��कया�जाएगा।�

· इसके�अलावा�, कृ�ष�, बागवानी�, म��य�और�वा�नक��म��चार�साल�क���ड�ी�को�पेशेवर��ड�ी�घो�षत��कया�गया�है।�

उ�े �य�

· प�रयोजना�का�उ�े �य�कृ�ष��व��व�ालय�के�छा���को�अ�धक��ासं�गक�और�उ�च�गुणव�ा�क���श�ा��दान�करने�म��भाग�लेने�वाले�कृ�ष
�व��व�ालय��और�आईसीएआर�का�समथ�न�करना�है।�

· NAHEP तकनीक��एओयू�का���ताव�और�काया��वयन�के��लए�इ�छु क�कृ�ष��व��व�ालय�का�समथ�न�करके�गुणव�ा�को�संबो�धत�करता�है


जो�संकाय��दश�न�को�बढ़ाता�है�, इन��व��व�ालय��के��लए�बेहतर�छा���को�आक�ष�त�करता�है�, छा��सीखने�के�प�रणाम��म��सुधार�करता�है
और��वशेष��प�से��नजी��े��म��भ�व�य�क��रोजगार�के��लए�उनक��संभावना��को�बढ़ाता�है।�

· एनएएचईपी�उन�सं�थान��को�ल��त�करेगा�जो�आईसीएआर-एयू��स�टम�का��नमा�ण�करते�ह��, �जसम��रा�य�कृ�ष��व��व�ालय�, डी�ड


�व��व�ालय�, क���य��व��व�ालय�कृ�ष�संकाय�और�क���य�कृ�ष��व��व�ालय�शा�मल�ह�।�

�या�आप�जानते�ह��?

· कृ�ष��वसाय�म��छा���क��भागीदारी�को�बढ़ावा�दे ने�के��लए�, छा��अ�ययन�(�ामीण�उ��मता�जाग�कता��वकास�योजना) चला�रहा�है�, �ज


सके�तहत��नातक�छा���को�कृ�ष�और�उ��मता�का��ावहा�रक�अनुभव��दान��कया�जाता�है।�

QUESTION 97.
�न�न�ल�खत�म��से�कौन-से�टाइगर��रज़व��को�संर�ण�आ�ासन�| बाघ�मानक�( CA | TS) �वीकृत���थ�त��दान�क��गयी�है�

a) बांद�पुर�टाइगर��रजव��
b) ब�सा�टाइगर��रजव��
c) काजीरंगा�टाइगर��रजव��
d) इनम��से�कोई�भी�नह��
Correct Answer: D
Your Answer:
Explanation

Solution (d)

भारत�म��उ�राखंड�के�ल�सडाउन�वन��भाग�, नेपाल�म���चतवन�नेशनल�पाक��और��स�म���सखोट-ए�लन�नेचर��रजव��को�CA | TS �वीकृत


दजा���दया�गया�है।�

QUESTION 98.
�न�न�ल�खत�म��से�कौन�सा�टू ल�, �वकासशील�दे श��को�आयात�वृ���या�क�मत��म���गरावट�से��नपटने�के��लए�अ�थायी��प�से�टै �रफ�बढ़ाने�क�
अनुम�त�दे ता�है�?

a) �वशेष�आहरण�अ�धकार�( Special Drawing Rights )


b) सामा�य�जो�खम�शमन�तं��( Common Risk Mitigation Mechanism)
c) रा�ीय�उपचार�दा�य�व�( National Treatment Obligation)
d) �वशेष�सुर�ा�तं��( Special Safeguard Mechanism)
Correct Answer: D
Your Answer:
Explanation

Solution (d)

IASbaba
Score:
Web: http://ilp.iasbaba.com/
47.00 /
Email: ilp@iasbaba.com
Page 138 200
2019 - Test 28-
Exam Title :
History & C...
Email : narenips6@gmail.com
Contact :

ड��यूट�ओ�का��वशेष�सुर�ा�तं��(एसएसएम) एक�सुर�ा�उपाय�है�, जो��वकासशील�दे श��को�कृ�ष�आयात��के��खलाफ�आक��मक���तबंध


लगाने�क��अनुम�त�दे ता�है�, �जससे�घरेलू��कसान��भा�वत�होते�है।�य�द�आक��मक�वृ���घरेलू�गरीब��कसान��के��लए�क�याणकारी�नुकसान�का
कारण�बनती�है�, तो�आक��मकता�शु�क�टै �रफ�है।�एसएसएम�का��डजाइन�और�उपयोग��व���ापार�संगठन�के�तहत�संघष��का�एक��े��है।�

ड��यूट�ओ�क��शत��म��, सुर�ा�उपाय��म���वशेष�प�र��थ�तय��जैसे��क�आयात�म��वृ���से��नपटने�के��लए�अ�थायी��प�से��लए�गए�आयात�पर
आक��मकता�या�आपातकालीन���तबंध�ह�।�आक��मक���तबंध�का�अथ��है�आयात�पर�रोक�लगाना�य�द�आयात�घरेलू�कृ�ष��े��पर�नकारा�मक
�भाव�पैदा�कर�रहा�है।�मूलभूत�GATT ही�घरेलू�अथ��व�था�क��सुर�ा�के��लए�इस�तरह�के���तबंध��क��अनुम�त�दे ता�है।�

दोहा�मं���तरीय�स�मेलन�म��, �वकासशील�दे श��को�मौजूदा�सुर�ा�उपाय��(जैसे��वशेष�कृ�ष�सुर�ा�या�एसएसजी) के�अलावा�एक��वशेष�सुर�ा


तं��(एसएसएम) अपनाने�क��छू ट�द��गई�थी।�इस�SSM ने�दोहा�म��दोहा��वकास�(�जसे�दोहा��वकास�एज�डा�के��प�म��जाना�जाता�है) और
दोहा�एमसी�को��वकास�के�दौर�म��पेश��कए�गए�वाद��का�एक�मह�वपूण���ह�सा�बनाया।�

जैसा��क�उ�लेख��कया�गया�है�, �वशेष�सुर�ा�तं��( SSM) ने��वकासशील�दे श��को�आयात��के�जवाब�म��कृ�ष�उ�पाद��पर�आयात�शु�क�बढ़ाने


क��अनुम�त�द��है।�

SSG सभी�दे श��के��लए�उपल�ध�था- �वकासशील�और��वक�सत�दोन��, जब�क�SSM केवल��वकासशील�दे श��के��लए��वीकाय��है।�यह


उ�लेख��कया�जाना�है��क�एसएसजी�उपल�ध�था��य��क�इसे�गैट�समझौते�के�तहत�शा�मल��कया�गया�था�; जब�क�एसएसएम�दोहा�एमसी�का
आ�व�कार�था।�

QUESTION 99.
दमन( DAMAN ) का�सं���त��प��ग�मा�आंचलारे�मले�रया��नरकरन�(�ग�म��े���म��मले�रया��नयं�ण) �न�न�ल�खत�म��से�कौन-से�रा�य��ारा
�ार�भ��कया�गया�है�?

a) असम�
b) ओ�डशा�
c) केरल�
d) कना�टक�
Correct Answer: B
Your Answer:
Explanation

Solution (b)

�ग�मा�अंचलारे�मले�रया��नराकरन�( DAMaN)

· यह�ओ�डशा�रा�य�सरकार��ारा�एक�पहल�है��जसका�उ�े �य�मले�रया��नयं�ण�ह�त�ेप��को�मजबूत�करना�है�

· इसका�नेतृ�व��ापक�मामले��बंधन�काय��म�( CCMP) �ारा��कया�जाता�है।�

· यह�काय��म�संयु���प�से�इं�डयन�काउं�सल�ऑफ�मे�डकल��रसच�-नेशनल�इं�ट��ूट�ऑफ�मले�रया��रसच��( ICMR-NIMR), नेशनल


वे�टर�बॉन���डजीज�कं�ोल��ो�ाम�( NVBDCP), ओ�डशा�तथा�मले�रया�व�चर�के��लए�दवा��( MMV) �ारा�संयु���प�से�लागू��कया
गया�है।�

· इस�काय��म�म��गहन��नगरानी�, म�छर��नयं�ण�के�उपाय�और�पूरे�वष���नय�मत��वा��य��श�ा�ग�त�व�धय��के�साथ-साथ�सकारा�मक�मामल�
के�उपचार�के�साथ�मले�रया�के��लए�सामू�हक�जांच�शा�मल�है।�

QUESTION 100.
�न�न�ल�खत�म��से�कौन�सा�सही�सुमे�लत�है/ह��?

रोग�����फसल�

1. येलो�र�ट�- गे�ँ�

IASbaba
Score:
Web: http://ilp.iasbaba.com/
47.00 /
Email: ilp@iasbaba.com
Page 139 200
2019 - Test 28-
Exam Title :
History & C...
Email : narenips6@gmail.com
Contact :

2. ��वक��व�ट- काली��मच��

3. ��ट�मटामाइको�सस( Stigmatomycosis ) - कपास�

सही�कूट�का�चयन�क��जए�:

a) 1 तथा�2
b) 2 तथा�3
c) 1 तथा�3
d) उपरो��सभी�
Correct Answer: D
Your Answer:
Explanation

Solution (d)

��वक��व�ट�, �जसे�फाइटो�थोरा�फुट�रोट�रोग�के��प�म��भी�जाना�जाता�है�, काली��मच��क��लता��क��अचानक�मृ�यु�का�कारण�बनता�है।�

��ट�मटामाइको�सस�एक�कवक�रोग�है�जो�कई�फसल��म��होता�है�, जैसे�कपास�, सोयाबीन�, पेकान�, अनार�, साइ�स�और��प�ता।�

गे�ँ�का�ये�लो�र�ट�, �जसे�गे�ँ����प�र�ट�के�नाम�से�भी�जाना�जाता�है�, गे�ँ�के�तीन�मु�य�रोग��म��से�एक�है�, जो�मु�यतः�ठं डे�वातावरण�म��उगाए


जाने�वाले�गे�ँ�म��पाया�जाता�है।�ऐसे��थान�आम�तौर�पर�उ�री�अ�ांश�या�ठं डे�मौसम�से�संबं�धत�होते�ह�।�

IASbaba
Score:
Web: http://ilp.iasbaba.com/
47.00 /
Email: ilp@iasbaba.com
Page 140 200

S-ar putea să vă placă și